Barbri MBE questions

Ace your homework & exams now with Quizwiz!

A husband and a wife were arrested by federal agents and charged with distributing obscene materials through the United States mails. When called before a grand jury, the wife refused to say anything, invoking her Fifth Amendment right to be protected from compelled self-incrimination. The husband was terrified of the grand jury and readily admitted under questioning that he sent obscene matter through the mail. He also incriminated his wife in the illegal activity. The thought of a trial and a prison term drove the husband over the edge, and he committed suicide two days before his trial was to begin. A month later, the wife was put on trial in federal district court. The federal prosecutor seeks to introduce a transcript of the husband's grand jury testimony into evidence against the wife. The defense attorney objects. How should the court rule on the admissibility of the grand jury transcript? A Admissible, as a vicarious admission. B Admissible, as former testimony. C Inadmissible, because the wife can invoke the testimonial privilege, even though her husband is now deceased. D Inadmissible, because the husband's testimony was not subject to cross-examination.

(D) The grand jury transcript is not admissible because the husband's testimony was not subject to cross-examination. The husband's testimony was hearsay because it was an out-of-court statement offered to prove the truth of the matter asserted. Under the former testimony exception to the hearsay rule, the testimony of a now unavailable witness given at another hearing is admissible in a subsequent trial as long as there is a sufficient similarity of parties and issues so that the opportunity to develop testimony or cross-examine at the prior hearing was meaningful. Thus, the grand jury testimony of an unavailable declarant is not admissible as former testimony against the accused at trial. This is because grand jury proceedings do not provide the opportunity for cross-examination. (A) is incorrect because the husband's testimony cannot be considered a vicarious admission. A statement by an opposing party is not hearsay. An admission is a statement made by a party and offered against that party. An admission does not have to be the statement of the party against whom the statement is being offered at trial if it qualifies as a vicarious admission. Here, however, the husband's grand jury testimony was not made in furtherance of a conspiracy. Because he was not a party here, and his testimony does not otherwise qualify as a vicarious admission of the wife, it cannot be considered an admission of a party-opponent. (B) is incorrect because the husband's grand jury testimony was not subject to cross-examination. The husband was an unavailable declarant because he was unable to testify because of death. However, as discussed above, his grand jury testimony is not admissible as former testimony because grand jury proceedings do not provide the opportunity for cross-examination.

A tenant entered into a written five-year lease to rent an office from a landlord for $6,000 per year beginning October 1. The lease required that rent in the amount of $500 be paid on or before the first of each month. Two months before the five-year term was up, the tenant received a new lease identical to the one he had already signed, except that the lease term began on the upcoming October 1 and the stated amount of rent per month was $600. The tenant returned the lease to the landlord unsigned, with a letter stating that he did not intend to renew the lease and would be moving out on September 30. The tenant did not move out on September 30. On October 1, the landlord received a check for $500 from the tenant. The notation on the check indicated that it was for the October rent. The landlord deposited the check in her account. She then sent a letter to the tenant stating that he was $100 in arrears in his rent. The tenant did not move out of the office during October, and the landlord did nothing to remove him. Most courts would hold that the tenant has what type of tenancy?

A year-to-year tenancy at $600 (R. Holdover Tenant liable under new terms of lease/money amount if tenant was notified of a rent increase prior to the end of the original lease; year-to year if commercial and month-to-month if residential)

A homeowner discovered that the siding on his house was defective and had allowed water to enter the structure, causing damage to the wood framing. The homeowner tried for some time to negotiate a settlement with the corporation that the homeowner believed had manufactured the defective siding. When no settlement was forthcoming, the homeowner filed an action in federal district court against the corporation one week before the SOL expired. Service of process was effected on the corporation several months later. After inspecting the home, the corporation filed and served its answer in which it denied manufacturing the siding used on the homeowners house. Upon examining the corporation's evidence, the homeowner conceded that the siding was manufactured by another company. With leave of court, the homeowner then filed an amended complaint substituting the actual manufacturer of the siding for the original incorrect defendant. The amended complaint was served on the manufacturer approximately seven months after the original complaint was filed and after the SOL had expired. The manufacturer was unaware of the action until it was served with the amended complaint. The manufacturer filed a motion for SJ on the grounds that the homeowner's claim against it is barred by the SOL. How should the court rule on the motion?

Grant the motion, because the amended complaint was filed after the SOL expired and the actual manufacturer did not receive timely notice of the action. (R. Amendments to claims substituting a new defendant are allowed and relate back if (1) claims arise from the same transaction or occurrence; (2) within the time allotted for serving the original complaint [90 days of filing]; (3) so that they received such notice that a claim was against them so that they would not be overly prejudiced; and (4) knew or should have known there would be an action against them.)

Auto workers went on strike in a town heavily reliant on the auto industry. While negotiations between the union and management were ongoing, a person intercepted and recorded a phone call between the union's president and management's chief negotiator. A state statute makes it illegal to record a phone call without the consent of the parties being recorded. The statute also make is illegal to play an illegally recorded conversation on television or radio. The person who recorded the call anonymously sent the recording to a local TV station. The TV station news anchor played the recording on air. Can the anchor who played the recording be prosecuted under the statute?

No, because the anchor did not record the conversation, and the information is truthful and about a matter of public significance. (R. Press has a right to publish info about a matter of public concern and it can only be restricted by a narrowly tailored sanction designed to further a state interest of the highest order; applies even if the information has been obtained unlawfully.)

A patient was scheduled to undergo non emergency surgery for the removal of her appendix by her family doctor. The day of the surgery, the doctor was called out of town because of a family illness. Even though the surgery could be postponed, the doctor asked the surgeon on call, who was an expert in appendectomies, to take his place. The patient was not informed of the switch in doctors. If the patient sues the surgeon on a battery theory, who will prevail?

The patient, regardless of whether she establishes damages at trial because there was no consent by patient. (There was no emergency and no consent)

On an icy day, a vehicle driven by the defendant struck the plaintiff's car in the rear, smashing a taillight and denting the plaintiff's bumper. Before the plaintiff could say anything, the defendant rushed out of his car and told the plaintiff, "Look, if you'll take $500 for the damage, I'm sure my insurance company will pay for it." The plaintiff refused and sued the defendant for damage to his car and minor personal injuries. The plaintiff wishes to testify as to the defendant's statement at the time of the accident. The defendant objects. Should the court allow the defendant's statement to be admitted?

Yes, because it is a statement by an opposing party. (R. Statement by opposing party being offered against him functions as an admission and is admissible.)

A balloonist sued the manufacturer of deflation panels for hot air balloons after one of the panels failed while his balloon was descending, causing the balloon to crash and the balloonist to suffer severe injuries. At trial, the balloonist calls as a witness a structural engineer who testifies that, common to industry practice, her opinion is based on several reports done by an independent laboratory on the burst strength and material composition of the deflation panel closures. The balloonist's attorney then asks the engineer whether, in her opinion, the closures caused the deflation panel to give way. The manufacturer objects. Should the court admit this testimony

Yes, because structural engineers reasonably rely on such reports in the course of their profession. (R. Experts do not have to have personal knowledge to make a statement/opinion; Experts do not have to offer into evidence reports used to make their statement/opinion.)

At the defendant's trial for grand theft auto and other offenses, the prosecution offers to introduce the testimony of a police officer. The officer will testify that he showed a photographic lineup containing the defendant's picture to a witness who had seen the defendant feeling form the stolen vehicle at the conclusion of a high-speed chase, and the witness selected the defendant's picture. The witness has left the state and she refuses to return. Should the court admit the evidence?

No, because it is inadmissible hearsay (Not admissible under prior identification because the witness is not available to testify, so we do not know if the witness is lying or not.)

A man purchased a large flat screen television and decided to mount it on the ceiling over his bed. The manual that came with the product included detailed instructions and illustrations on how to mount the television on different types of walls, along with all the required hardware, but contained neither instructions nor warnings regarding mounting on the ceiling. The man carefully followed the wall-mounting instructions and was satisfied that it would hold. In fact, however, the mounting was not appropriate for ceilings. The next night, a woman who was the man's overnight guest was seriously injured when the television came loose and fell on the bed. Will the woman prevail in a suit against the company that manufactured the television? A Yes, because the manufacturer had a duty to include warnings for all potential placements of its product. B Yes, if the manufacturer knew that its television was sometimes mounted on ceilings rather than walls. C No, if the manufacturer's manual had all of the customary warnings for this type of product. D No, because the man was negligent in mounting the television on the ceiling.

(B) Knowledge on the part of the manufacturer that its television was being mounted on the ceiling would give rise to a duty to include in the manual warnings against the practice or detailed instructions on how to safely mount it. The television hardware and instructions were appropriate for its intended mounting on the wall. However, courts in a strict liability case require a commercial supplier to anticipate reasonably foreseeable uses even if they are misuses of the product. If the manufacturer knew that members of the public were sometimes mounting the television on the ceiling, marketing the product without including either warnings against the practice or appropriate hardware and instructions on how to safely do so made the product so defective as to be unreasonably dangerous if it were improperly mounted. Under a strict liability theory, the manufacturer is liable for supplying a defective product. As a guest of a purchaser of the product, the woman is a foreseeable plaintiff; thus, the manufacturer may be liable to her. The defective product actually and proximately caused the woman to suffer serious injuries. Therefore, the manufacturer is liable to the woman in a strict products liability action. (A) is incorrect because the facts do not establish that the manufacturer was under a duty to include the warnings in its manual. Such a duty would exist if the manufacturer knew (as (B) states) or should have known that the television was being mounted on ceilings.

If more than _______ months' imprisonment is authorized, the offense is considered "serious" for determining whether a defendant has a constitutional right to a jury trial. A nine B six C three D twelve

(B) An offense is considered serious, making a jury trial a constitutional right, when more than six months' imprisonment is authorized.

A landowner owned two adjoining parcels of land containing a number of lakes. She conveyed the eastern parcel, which contained a campground, to a fisherman. The deed transferring the parcel granted to the fisherman "and to invited guests of the campground all hunting and fishing rights and use of the lakes on the western parcel for the benefit of the campground." Subsequently, the fisherman assigned his hunting and fishing rights to a hunter. When the landowner discovered the hunter hunting and fishing on her land, she brought an appropriate action to declare his rights void. If the court rules for the landowner, it will be because the fisherman's right to hunt and fish on the western parcel is.:

A profit appurtenant (R. Profit is a nonpossessory interest in land that entitles the profit holder to enter onto the servient land to take minerals and substance from the land [can be appurtenant or gross]; Can only transfer an appurtenant profit with the land itself; Can transfer gross profit without transferring the land itself; Easement in gross only allows you to use the land, not take stuff from the land.)

A motorist from State A struck and injured a pedestrian in State B. The pedestrian, a state B resident, brought an action in a State B federal court against the State A motorist, seeking $100,000 in damages. The summons and complaint were served on a receptionist at the motorist's place of business in State A. State A's rules permit service of process in this manner, while State B's rules do not. If the motorist moves to dismiss the complaint on the basis of improper service of process, is the court likely to dismiss the action?

No, because the federal rules permit service under the rules of the state in which service will be effected (R. Service can be made as allowed by the rules of the state where the fed. court sits or the state where service was to be affected.)

The plaintiff sued the defendant dry cleaner, claiming that it had permanently ruined her $10,000 mink coat by cleaning it with a solvent that left an extremely offensive odor that smelled like "skunk." Further attempts to have the odor removed by other cleaning services were unsuccessful. The odor was so bad that she could no longer wear the coat. At the trial, the plaintiff testified to the above facts. She then identified a mink coat as her coat that the defendant had ruined. She testified that it still smelled the same as it did after the defendant had cleaned it. The plaintiff's counsel offered to introduce the coat for the purpose of having the jury smell it. Defense counsel objected. How should the court rule?

The coat is admissible based on the plaintiff's testimony (R. Before an object is admissible at trial, the proponent of the evidence must present some additional evidence to establish that the object is what the proponent claims it is; you DO NOT HAVE to present extrinsic evidence.)

An investor rented his property to a pottery maker, who intended to use the back part of the building for living quarters, and the front part as a pottery studio. The pottery maker installed a kiln, some lights, and some storage units in the front part for her use. Sometime later, the investor mortgaged the property to a bank to secure a loan. The mortgage was recorded, but the investor did not personally tell the pottery maker that he had done so. In fact, she only learned of it when the investor defaulted on the loan and the bank foreclosed on the mortgage and told the pottery maker that she would have to quit the premises. The pottery maker began removing the equipment and fixtures that she had installed in the building. The bank objected and sought an injunction to prevent her from doing so. Under these circumstances, on what basis should the court deny the injunction

The equipment was installed for the pottery maker's exclusive benefit and she did not intend for it to stay. (R. Residential fixture: look at nature of chattel, intent for whether chattel will stay, and how much damage will be caused to the real property if the chattel is removed; Commercial fixture: we apply the trade fixtures doctrine, which allows a tenant to remove all trade fixtures prior to lease expiring. [Exception: accessions cannot be removed, which are structural additions to the property like a deck.]) (Here, fixtures are on the commercial part, so they can all be removed prior to the expiration of the lease.) (notice of there being a mortgage by landowner on the property is irrelevant.)

A small processor of specialized steel agreed in writing with a small manufacturer of children's toys that it would supply, and the manufacturer would buy, all of the manufacturer's specialized steel requirements over a period of years at a set price per ton of steel. Their contract did not include a nonassignment clause. Recently, the toy manufacturer decided to abandon its line of steel toys, so it made an assignment of its rights and delegation of its duties under the contract to a toymaker many times larger. The large toymaker notified the steel processor of the assignment and relayed to the processor its good faith belief that its requirements will approximate those of the assignor. Must the steel processor supply the requirements of the large toymaker? (A) Yes, because there was no nonassignment clause in the contract. (B) Yes, because the large toymaker acted in good faith to assure the steel processor that its requirements will approximate those of the small manufacturer into whose shoes it stepped. (C) No, because requirements contracts are not assignable under the UCC (D) No, because the steel processor did not give prior approval of the assignment.

(A) Because the large toymaker acts in good faith in setting its requirements to approximately those of the small manufacturer into whose shoes it stepped, the contract may be assigned. The contract in this question is a "requirements" contract: The steel processor must sell the small manufacturer of children's toys all the specialized steel it requires for its toys. Generally, the right to receive goods under a requirements contract is not assignable because the obligor's duties could change significantly. In fact, here, a significant change would seem possible because the large toymaker is a larger company than the small manufacturer and its needs could be greater. However, the UCC allows the assignment of requirements contracts if the assignee acts in good faith not to alter the terms of the contract. [UCC 2-306] (The UCC applies here because goods are involved.) Thus, assuming the large toymaker's requirements remain about the same as the small manufacturer's requirements, the steel processor would be required to honor its contract, now assigned to the large toymaker. (A) is wrong because requirements contracts may be nonassignable, even without a nonassignment clause. Thus, the clause would be irrelevant. The only thing that could allow assignment of a requirements contract is a good faith limitation, as addressed in choice (B). (C) is wrong because the UCC does allow requirements contracts to be assigned, as long as the good faith limitation is satisfied. (D) is similarly incorrect. The UCC would allow assignment without approval by the obligor if there is a good faith limitation on the requirements.

A landowner included in his will a provision giving "all of my property, both real and personal, wherever situated, to my widow for life, and after her death to any of our children who may survive her." What is the gift to the children? A A contingent remainder. B A vested remainder. C A shifting executory interest. D Void, as violating the Rule Against Perpetuities.

(A) The children have a contingent remainder. A remainder is a future interest created in a transferee that is capable of taking in present possession on the natural termination of the preceding estate created in the same disposition. Note that, as a rule of thumb, remainders always follow life estates. A remainder will be classified as contingent if its taking is subject to a condition precedent, or it is created in favor of unborn or unascertained persons. Here, the interest in the children follows a life estate and is a remainder because it is capable of taking in possession on the natural termination of the preceding estate. It is subject to the condition precedent of surviving the landowner's widow and, additionally, is in favor of unascertained persons Thus, the interest is a contingent remainder. (B) is incorrect because a vested remainder can be created in and held only by ascertained persons in being, and cannot be subject to a condition precedent. (C) is incorrect because a shifting executory interest is one that divests the interest of another transferee; i.e., it cuts short a prior estate created by the same conveyance. The gift to the children does not divest the interest of the widow; she retains a life estate in the property. The children's interest takes in possession only on the natural termination of the widow's estate (i.e., at her death).

A criminal suspect filed an action in federal district court against the police officer who arrested him and the city that employed the officer, claiming that the officer beat him even though he was not resisting arrest. The suspect's action asserted a battery claim against the police officer and a federal statutory claim against the city for violation of the suspect's civil rights. The suspect and the police officer are citizens of State A, and the city in which the arrest took place is in State A. The action seeks $50,000 each from the police officer and the city. Does the federal district court have subject matter jurisdiction over either of the claims? A The court has federal question jurisdiction over the federal statutory claim against the city and supplemental jurisdiction over the tort claim against the police officer. B The court has federal question jurisdiction over the federal statutory claim against the city, but it does not have subject matter jurisdiction over the tort claim against the police officer because the amount of that claim is too small. C The court has federal question jurisdiction over the federal statutory claim against the city, but it lacks subject matter jurisdiction over the tort claim against the police officer because it is a state law claim between two citizens of the same state. D The court does not have subject matter jurisdiction over either claim because the amount of each claim is too small.

(A) The court has subject matter jurisdiction over both claims. Federal question jurisdiction is available when the plaintiff, in his well-pleaded complaint, alleges a claim that arises under federal law. Here, the suspect claims that his civil rights were violated, which is a valid federal claim; thus, the court has federal question jurisdiction over that claim. The battery claim against the police officer does not arise under federal law, but rather under state law. However, when the federal court has subject matter jurisdiction over one claim, it has discretion to exercise supplemental jurisdiction over related claims that derive from the same common nucleus of fact and are such that a plaintiff would ordinarily be expected to try them in a single judicial proceeding. Here, the suspect's claim against the city is derived from the officer's battery, which is the basis of the state law claim. As a result, the two claims arise from the same nucleus of common fact. Thus, the battery claim may be heard in federal court under supplemental jurisdiction.

The criminal statutes of the state define manslaughter and murder as they were defined at common law. As to insanity, the state has the following provision: "Under the defense of insanity a defendant may be entitled to acquittal if, because of mental illness, the defendant was unable to control his or her actions or to conform his or her conduct to the law." The defendant was put on trial in the state for the murder of his wife and her co-worker. The evidence at trial established that the defendant's wife was having an affair with the co-worker, and that the defendant learned of it and killed the pair. The defendant did not take the stand in his own defense. In his closing statement to the jury, the defendant's attorney made a statement, "Ladies and gentlemen, you must consider that there are some things that would provoke any one of us to kill, and there are things that make one unable to control one's actions." The defendant's attorney requested that the judge give the jury instructions on manslaughter and on insanity, and the judge agreed to do so. The judge also issued the following instructions: "INSTRUCTION #6: In order to mitigate an intentional killing to voluntary manslaughter, the burden of proof is on the defendant to establish that adequate provocation existed." "INSTRUCTION #8: Insanity is an affirmative defense and the burden of proof is on the defendant to establish that such insanity existed at the time of the killing." The jury found the defendant guilty of murder, and he appealed. He asserts that the jury instructions violated his rights under the federal Constitution. How should the appeals court rule? A Reverse the defendant's conviction, because Instruction #6 was improper. B Reverse the defendant's conviction, because Instruction #8 was improper. C Reverse the defendant's conviction, because both Instructions #6 and #8 were improper. D Uphold the defendant's conviction, because neither Instruction #6 nor Instruction #8 was improper.

(A) The court should reverse the defendant's conviction because Instruction #6 requires the defendant to disprove one of the elements of murder. Due process requires in criminal cases that the state prove guilt beyond a reasonable doubt. The prosecution has the burden of proving all of the elements of the crime charged. Thus, if malice aforethought is an element of murder and voluntary manslaughter is distinguished from murder by the existence of adequate provocation, the defendant cannot be required to prove that he committed the homicide in the heat of passion (i.e., with adequate provocation). Such a requirement would impose on the defendant the burden of disproving the element of malice aforethought, because "heat of passion" negates malice. Although the defendant can be given the burden of going forward with some evidence on the provocation issue, once he has done so, the prosecution bears the burden of proving that the killing was not done in the heat of passion. In the case at issue, Instruction #6 requires a defendant to prove that he committed the intentional killing under adequate provocation. At common law, and consequently in the state, malice aforethought is an element of murder. Therefore, this instruction in effect requires the defendant to disprove the element of malice aforethought, thereby relieving the state of its burden of proving all elements of the crime. As discussed above, such an instruction cannot pass constitutional muster. On the other hand, for an affirmative defense such as insanity, it is permissible to impose the burden of proof on the defendant. Thus, Instruction #8 does not affect the state's obligation to prove all elements of the crime, and is permissible under the general principles mentioned above.

Based on recommendations of a state commission studying the effect of pornographic films on violent criminal activity, a state adopted legislation banning films intended for commercial distribution that appealed as a whole to the prurient interest in sex of the average person in the community, portrayed sex in a patently offensive way to citizens of the state, and which a reasonable person in the United States would find had no serious literary, artistic, political, or scientific value. In ruling on a constitutional challenge to the legislation from a film distributor in the state who was convicted of distributing films in violation of the legislation, will the federal court likely find the legislation to be constitutional? A Yes, because it uses a national "reasonable person" standard for determining the social value of the work. B Yes, because it uses a statewide standard rather than a community standard for determining whether the material is patently offensive. C No, because it uses a statewide standard rather than a national standard for determining whether the material is patently offensive. D No, unless the court finds that the legislation is necessary to advance the state's compelling interest in reducing violent criminal activity.

(A) The court will likely find the legislation to be a constitutional regulation of obscenity. Obscenity, which is not protected speech under the First Amendment, is defined by the Supreme Court as a description or depiction of sexual conduct that, taken as a whole, by the average person, applying contemporary community standards, appeals to the prurient interest in sex, portrays sex in a patently offensive way, and—using a national reasonable person standard—does not have serious literary, artistic, political, or scientific value. Thus, the legislation here is constitutional because it uses a reasonable person standard, rather than a community standard, for determining the value of the work. (B) is incorrect because while a statewide standard for determining whether the material is patently offensive is permissible, it is not mandatory. A state may use a "community standard" for making this determination. (C) is incorrect because, again, a statewide standard for determining whether the material is patently offensive is permissible. Only the "social value" element of the obscenity test requires a national standard. (D) is incorrect because the legislation is valid regardless of whether it is necessary to achieve the state's compelling interest in reducing violent crime. Speech that falls within the definition of obscenity is unprotected speech; the government does not need a specific compelling interest to ban it.

An acquaintance asked the defendant to give him a lift downtown because he did not have bus fare. While riding on the defendant's motorcycle, the acquaintance asked to stop at a convenience store to get a bottle of wine, showing the defendant a tire iron in his backpack that he was going to use. The defendant nodded in acknowledgment of what the acquaintance was planning to do. The defendant stopped at the store and waited in the parking lot while the acquaintance went in. He demanded money from the clerk, brandishing the tire iron. The clerk tried to grab a gun under the counter while he was filling a bag with money, and a struggle ensued. The gun discharged, killing the clerk. The defendant heard the gunshot and raced off, but was eventually apprehended. The jurisdiction's criminal code provides that a death caused during the commission of certain felonies, including robbery, is first degree felony murder, for which the death penalty is permitted. The code also permits cumulative penalties for first degree felony murder and for the underlying felony. The defendant was charged and convicted of both robbery and felony murder. After appropriate consideration of all relevant circumstances, the jury imposed the death penalty. On appeal, the defendant challenged both the convictions and the sentence. Assuming that the above facts were properly admitted into evidence, how should the appellate court rule? A The defendant's conviction for both offenses should be upheld, but imposition of the death penalty was not proper. B The defendant's conviction for both offenses should be upheld, and imposition of the death penalty was proper. C The defendant's conviction should be overturned under double jeopardy principles because robbery is a lesser included offense of felony murder. D The defendant's conviction for felony murder should be overturned because the circumstances do not establish the necessary degree of culpability.

(A) The defendant can be found guilty of robbery and felony murder, but the death penalty cannot be imposed. The defendant can be found guilty of robbery as an accomplice. The Supreme Court has held that, under the Eighth Amendment, the death penalty may not be imposed for felony murder where the defendant, as an accomplice, did not take or attempt or intend to take life, or intend that lethal force be employed. [Enmund v. Florida (1982)] Here, because the defendant's involvement in the crime was only to provide transportation, it cannot be said that he participated in such a major way that he acted with reckless indifference to human life; hence, the death penalty cannot constitutionally be imposed against him.

A plaintiff is suing a defendant in federal court for personal injuries arising out of an automobile accident. The defendant's automobile insurance policy covers the accident. Must the defendant disclose the existence of the insurance policy? A Yes, because it is required under the Federal Rules. B Yes, unless the discovery of the insurance coverage would not lead to other discoverable evidence. C No, unless the plaintiff submits an interrogatory. D No, because insurance coverage cannot be mentioned at trial.

(A) The defendant must disclose the existence of the insurance policy under the Federal Rules, which expressly permit discovery of insurance agreements as an initial disclosure. (B) is incorrect. There is no need to show that the discovery of insurance coverage would lead to other admissible evidence. (C) is incorrect. Initial disclosures must be made regardless of whether the opposing party submits a request for the information. (D) is incorrect. Even though insurance coverage is not relevant to the case, insurance coverage is nonetheless discoverable.

A plaintiff sued a defendant for defamation, asserting in her complaint that the defendant had called the plaintiff a thief in front of a number of business associates. The plaintiff calls two witnesses to the stand, both of whom testify that they heard the defendant refer to the plaintiff as a thief in front of the business associates. The plaintiff does not take the stand herself. The defendant pleads truth of the statement as an affirmative defense and calls a witness to the stand. The defense witness is prepared to testify that he was a co-worker of the plaintiff when the plaintiff supplemented her income by tending bar three nights a week. The witness will testify that he saw the plaintiff take a $20 bill from the tavern's cash register and secrete the money in her pocket. The plaintiff's attorney objects. May the defense witness's testimony be allowed? A Yes, as substantive evidence that the plaintiff is, in fact, a thief. B Yes, because theft is a crime indicating dishonesty. C No, because specific bad acts may not be used to show bad character. D No, because the plaintiff never took the stand.

(A) The defense witness's testimony is admissible character evidence because the plaintiff's character is directly in issue in the case. As a general rule, evidence of character to prove the conduct of a person in the litigated event is not admissible in a civil case. However, when proof of a person's character, as a matter of substantive law, is an essential element of a claim or defense in a civil action, character evidence is admissible because it is the best method of proving the issue. Under the Federal Rules, any of the types of evidence—reputation, opinion, or specific acts—may be used. Here, character is an issue in the plaintiff's defamation action because the defendant has pleaded as an affirmative defense that his statement claiming that the plaintiff is a thief is the truth. The defense witness's testimony that he saw the plaintiff take the money from the cash register is relevant because it tends to show that the defendant spoke the truth. Hence, it should be allowed. (B) is incorrect because the fact that the theft here could be considered a crime of dishonesty would be relevant only if the plaintiff's credibility were being impeached, and only then if proof of an actual conviction were provided. Here, the testimony is admissible because it is being offered as substantive evidence of an aspect of the plaintiff's character that is an essential element of a defense in the case. (C) is incorrect. One of the few cases where testimony as to specific acts of a person may be used to show that person's character is when character itself is one of the essential issues in the case, as it is here. (D) is incorrect because the fact that the plaintiff never took the stand only means that she has not placed her credibility in issue and become subject to impeachment. Here, however, the plaintiff's character is in issue and the testimony is being offered as substantive evidence of her character rather than to impeach her credibility.

A landowner owned a beachfront lot and home in a subdivision occupying several hundred acres near a lake. The recorded subdivision plan grants to each owner in the subdivision an easement to use the private roads therein for personal ingress and egress. Following seismic activity in the area, the level of the lake dropped substantially, exposing a considerable amount of land between the new shoreline and the old beachfront. It was judicially determined that this "new" land belonged to the county, which put portions of it up for sale. The landowner purchased the land extending from her old property line to the new shoreline, and constructed a boat launching ramp on the new property. She then permitted persons who did not own land in the subdivision to drive through her old property to reach the boat launching ramp on her new property, and thus to utilize the lake, for a small fee. The homeowners' association brought suit against the landowner, seeking to enjoin her from using or permitting nonresidents of the subdivision from traveling its streets to reach the boat launching ramp. How should the court rule? A For the homeowners, because the scope of the easement granted to the landowner as an owner in the subdivision does not extend to the use that she is making of the new property. B For the landowner, because she has an express easement over the streets of the subdivision. C For the landowner, because she has an easement by necessity as to the new property over the streets of the subdivision. D For the landowner, because she has an implied easement over the streets of the subdivision benefiting the new property since it abuts her old property.

(A) The express easement for the landowner's old property benefits that property only and cannot be used for the landowner's expanded access to the new property. An easement is a liberty, privilege, or advantage that one may hold in the lands of another. The holder of an easement has the right to use a tract of land (called the servient tenement) for a special purpose; e.g., laying utility lines, or for ingress and egress. An easement can be created, as in this question, by express grant. If the parties to the original creation of the use specifically state the location of the easement, its dimensions, and the special use or limits to such use, the courts will honor this expression of specific intent. Absent specific limitations, it will be assumed that the parties intend that the easement meet both present and future reasonable needs of the dominant tenement. However, a basic change in the nature of the use is not allowed. The landowner's easement by express grant merely allows her to use the private roads in the subdivision for her personal ingress and egress to and from her beachfront property. The use of the easement for access to a new boat launching ramp for which a fee is charged goes beyond the specific language of the grant (and arguably beyond the reasonable needs of the dominant tenement). Therefore, the homeowners will be able to prevent use of the subdivision streets to reach the boat launching ramp.

A brother and a sister held record title to a home as joint tenants with right of survivorship. The brother moved out of the home shortly after conveying his interest in the home to his friend by quitclaim deed. The friend did not record his deed. Several years later, the sister died, leaving her adopted daughter as her sole heir. Shortly after the sister died, the brother asked his friend to return his deed and give up his interest in the home. The friend agreed and returned the deed, which the brother destroyed. Who has title to the home? A The friend and the daughter as co-tenants. B The brother and the daughter as co-tenants. C The brother as sole owner. D The friend as sole owner.

(A) The friend and the daughter own the home as co-tenants. A joint tenancy is an estate between two or more co-tenants who have a right of survivorship—when one joint tenant dies, the property is freed from her concurrent interest and the survivor or survivors retain an undivided right in the property. An inter vivos conveyance by one joint tenant of his undivided interest destroys the joint tenancy so that the transferee takes the interest as a tenant in common and not as a joint tenant. Here, when the brother conveyed his interest to the friend, the joint tenancy between the brother and the sister was severed. At that point, the friend and the sister held title to the home as tenants in common. The adopted daughter then inherited the sister's interest upon the sister's death. Because delivery of a deed cannot be canceled, the friend's return and subsequent destruction of his deed has no effect.

A man and a woman lived together for many years but never got married. Although the state in which they reside does not recognize common law marriage, it has statutes that prohibit discrimination on the basis of marital status. The man and the woman purchased a large property, taking title as joint tenants. Subsequently, the woman accumulated a $20,000 debt. She was too embarrassed to tell the man but was able to convince a bank to hold a mortgage on the property in exchange for the money. The bank was also willing to accept the woman's signature alone, and the man never learned about the mortgage. Two years later, the woman died without having paid off the mortgage. She left no will, and her only heir at law is her sister. The state in which the property is located is a "lien theory" mortgage state. Who has title to the property? A The man. B The man and the bank. C The man and the sister. D The man, the sister, and the bank.

(A) The man takes sole title to the property under his right of survivorship. A joint tenancy carries the right of survivorship. Thus, when one joint tenant dies, the property is freed of her interest and the surviving joint tenant holds the entire property. Therefore, the man owns the property. (B) is wrong because the bank has no interest. Most states, like the one in this question, regard a mortgage as a lien on title. In these states, a mortgage of the property by one joint tenant does not, by itself, sever a joint tenancy until default and foreclosure proceedings have been completed. The bank's rights were lost when the woman died prior to foreclosure. When the woman died, her interest in the property evaporated, and with it the bank's security interest. On the other hand, in a title theory state, a mortgage is considered to be an actual transfer of title to the property, rather than just a lien on the property. Thus, a mortgage by a joint tenant transfers the legal title of the joint tenant to the mortgagee (the money lender). This action destroys the unity of title and thus severs the joint tenancy.

A local entertainment section of a newspaper published a story on the town's business district, accompanied by photos of various businesses in the district. A minister who happened to be walking on the sidewalk in front of an adult bookstore when a photo was taken for the story became very upset when he saw it in the newspaper, because the camera angle made it appear that he was exiting the bookstore. If the minister sues the newspaper for invasion of privacy and establishes the above facts, is he likely to prevail? A Yes, because the photo made it appear as if he was exiting an adult bookstore. B Yes, because the newspaper made a public disclosure of a private fact. C No, because he was on a public sidewalk when the photo was taken. D No, because he has not alleged any economic or pecuniary damages.

(A) The minister likely will prevail because unauthorized use of his picture that falsely makes him appear to be exiting the adult bookstore would be highly offensive to a reasonable person under the circumstances and constitute a false light invasion of privacy. To establish a prima facie case for invasion of privacy based on publication by defendant of facts placing plaintiff in a false light, the following elements must be proved: (i) publication of facts about plaintiff by defendant placing plaintiff in a false light in the public eye; and (ii) the "false light" must be something that would be highly offensive to a reasonable person under the circumstances. Here, the photo created the false impression that the minister was exiting an adult bookstore. Publication of the photo conveying this false impression of the minister's conduct would be highly offensive to a reasonable person under the circumstances.

During a presidential campaign, a candidate's campaign manager secretly engaged in activities that may have violated both state and federal laws. After the candidate was elected President, the FBI investigated the manager's activities as well as whether the President was involved. After the campaign manager was indicted in federal court, but before trial, the President granted a blanket pardon to the campaign manager for "all federal crimes that may have been committed in the past 20 years." Is the pardon valid? A Yes, because the pardon power is an unqualified power (except as to impeachment). B No, because a presidential pardon that interferes with an inquiry into the President's own actions constitutes an obstruction of justice. C No, presidential pardons must relate to specific crimes; the President cannot issue blanket pardons. D No, because a President's pardon power is limited to crimes that allegedly took place while the President is in office.

(A) The pardon is valid. Article II, Section 2 of the United States Constitution grants the President the power to grant reprieves and pardons for offenses against the United States, except in cases of impeachment. This pardon power is not subject to control by Congress, and it includes the power to commute a sentence on any conditions the President chooses (as long as the conditions do not offend some other constitutional provision). (B) is incorrect because even if the action of issuing the pardon amounted to the crime of obstruction of justice (a questionable assumption), the pardon itself would not be invalidated. The power to pardon is a constitutional power, superior to laws found in statutes. (C) is incorrect because the pardon power is not so limited. Blanket pardons are valid. (D) is also incorrect because the pardon power is not so limited. Presidents may pardon offenses that occurred before the President took office.

A man shopping for a leather jacket at a clothing store could not decide between two jackets, so the proprietor, who knew the man and his family well, let him take one of the jackets on approval. No mention was made by the proprietor of the method of payment he expected. The man wore the jacket on a visit to his grandfather, who liked it so much that when the man told him what the jacket cost and that he had taken it on approval, the grandfather said he would buy it for him if he promised to give some of his old clothes to a favorite charity for the poor at Christmastime. The man wholeheartedly agreed to donate the clothes to the charity at Christmas. Very pleased, the grandfather called the shop and told the proprietor to send the bill for the jacket to him, which he did. Before the bill was paid and before the Christmas season arrived, the grandfather fell ill and died. The grandfather's executor has refused to pay the bill, and the man has not yet given any old clothing to the charity. Will the proprietor be able to recover the price of the jacket from the estate? A Yes, because the proprietor was the intended beneficiary of the promise between the man and his grandfather. B Yes, because the man has no duty to give the clothing to the charity. C No, because the grandfather's implied promise to pay the proprietor arising from the phone call is unenforceable. D No, because a condition has not yet occurred.

(A) The proprietor can recover the cost of the jacket from the grandfather's estate because the proprietor is an intended third-party beneficiary and his right to enforce the contract has vested. The rights of an intended third-party beneficiary vest when the beneficiary (i) manifests assent to the promise in a manner invited or requested by the parties; (ii) brings suit to enforce the promise; or (iii) materially changes his position in justifiable reliance on the promise. Here, the proprietor qualifies as an intended beneficiary of the agreement between the man and his grandfather because the proprietor was expressly designated in the contract, he was to receive performance directly from the grandfather, and he stood in an existing contractual relationship with the man that required the man to either pay for the jacket or return it, making it likely that the young man's purpose in making the arrangement with his grandfather was to satisfy the obligation to the proprietor. The proprietor can enforce the contract because his rights vested when he sent the bill to the grandfather at the grandfather's request. Thus, the proprietor will prevail against the grandfather's estate. (D) is wrong because the man's giving the clothes to the charity is not a condition that must be fulfilled before the grandfather's estate must pay. The grandfather promised to pay for the jacket if the man promised to donate the clothes; i.e., the consideration for the grandfather's promise was the man's promise, not his actually donating the clothes. As soon as the man made the promise, the grandfather's duty to pay became absolute. (If the man does not donate the clothes, he will be in breach of his contract with his grandfather, but the grandfather's performance was not conditioned on the man's donating the clothes.)

A lawyer sued a client for his fee, based on an agreed hourly rate. The client subpoenaed the lawyer's time records for the days on which he purported to have worked for the client to show that the lawyer had billed an impossible number of hours to the client and others on those days. The client's subpoena provided that any information concerning the matters handled for other clients be deleted or masked. The lawyer moved to quash the subpoena on the ground of attorney-client privilege. Should the subpoena be upheld? A Yes, because the information about hours billed is not within the privilege. B Yes, because a lawyer has no right to invoke his client's privilege without instructions from the client. C No, because a lawyer is entitled to a right of privacy for the work product in his files. D No, because no permission was obtained from the other clients to divulge information from their files.

(A) The subpoena should be upheld because the information about hours billed is not within the privilege. There is no privilege regarding a communication that is relevant to an issue of breach of duty by the lawyer to his client or by the client to her lawyer. Thus, the billing data does not fall within the ambit of the privilege. (B) is incorrect because the information here is not covered by the privilege. Furthermore, if the privilege were applicable, the lawyer would be able to invoke the privilege on behalf of his clients. The lawyer's authority to do this is presumed in the absence of any evidence to the contrary. (C) is a correct statement of law, but the work product rule does not apply to these facts. Documents prepared by the lawyer for his own use in prosecuting his client's case are protected by this rule. Time records are not prepared for litigation purposes; they are not related to the substance of the client's case. (D) is incorrect because the other clients are protected by the blacking out of confidential information. Moreover, because the time records are not communications to or from the client and the identity of clients is often not considered to be within the privilege, the billing records of other clients may not be privileged and their consent may not have been necessary even without the deletions.

served several alcoholic drinks by a bartender at a local bar. The pedestrian sued the bartender in a jurisdiction that does not have a dramshop act. Is the bartender vicariously liable for the pedestrian's injuries? A No, because the driver acted recklessly by driving while intoxicated. B No, because there is no dramshop act in the jurisdiction to impose liability. C Yes, because there is no dramshop act in the jurisdiction to limit liability. D Yes, because the intoxicated driver caused the pedestrian to suffer personal injuries

(B) Because the jurisdiction does not have a dramshop statute, the bartender will not be liable for the injuries caused to the pedestrian by the intoxicated driver. At common law, no liability was imposed on vendors of intoxicating beverages for injuries resulting from the vendee's intoxication, whether the injuries were sustained by the vendee or by a third person as a result of the vendee's conduct. Many states, in order to avoid this common law rule, have enacted "dramshop acts." Such acts create a cause of action in favor of any third person injured by the intoxicated vendee. Without a dramshop act, the bartender will not be vicariously liable.

A plaintiff filed a civil action based on negligence against a defendant in federal district court, alleging that the defendant negligently ran a red light at an intersection and collided with the plaintiff's vehicle, causing the plaintiff's injuries. A week after the close of discovery, the plaintiff filed a motion for summary judgment on the issue of whether the defendant was negligent. With the motion, the plaintiff filed (i) his own sworn affidavit, which stated that the traffic signal was green as he entered the intersection, (ii) an affidavit of a witness who was driving the car behind him, which stated that the witness saw the entire incident and that the plaintiff's traffic signal was green as he approached and entered the intersection; and (iii) an affidavit of another witness, which stated that she saw the entire incident and that the defendant's signal had been red for several seconds before the defendant entered the intersection and was still red when the defendant entered the intersection. In response to the motion, the defendant filed her own affidavit, which stated that her traffic signal was green when she entered the intersection. She also filed the sworn deposition transcript of a pedestrian on the scene, in which the pedestrian said that, although he was intoxicated, he saw the entire accident and that the defendant's traffic signal was green as the defendant approached and entered the intersection. How should the court rule on the plaintiff's motion for summary judgment? A Deny the motion, because the plaintiff cannot obtain summary judgment on issues on which he has the burden of proof. B Deny the motion, because both parties have come forward with evidence from which a jury could find for either party if the jury believes that party's evidence. C Grant the motion, because valid reasons exist to question the credibility of the evidence that the defendant presented. D Grant the motion, because the evidence presented by the plaintiff is substantially more persuasive than the evidence presented by the defendant.

(B) The court should deny the plaintiff's motion. Summary judgment may be granted if, from the pleadings, affidavits, and discovery materials, it appears that there is no genuine dispute of material fact and the moving party is entitled to judgment as a matter of law. The court may not decide disputed fact issues on a motion for summary judgment; if there is a genuinely disputed material fact (meaning a dispute backed by evidence on both sides of the issue), the case must go to trial. Here, although the plaintiff's case may seem stronger, the defendant has presented some evidence showing that she was not negligent. Thus, the case must proceed to trial. (C) and (D) are incorrect because, in a motion for summary judgment, the credibility of the witnesses and the strength of the evidence is not weighed.

A father executed a deed to his art gallery "to my daughter for her life, and on my daughter's death to her children; provided, however, that if my daughter stops painting, to my brother." The daughter has two children and is still painting. At the time of the grant, what is the best description of the interest of the daughter's two children? A A contingent remainder. B A vested remainder subject to open and to total divestment. C A vested remainder subject to open. D An executory interest.

(B) The daughter's two children have a vested remainder subject to open and subject to complete divestment. A remainder is a future interest created in a transferee that is capable of taking in possession on the natural termination of the preceding estate. A remainder is vested if the beneficiaries are ascertainable and their taking in possession is not subject to a condition precedent. A vested remainder created in a class of persons that is certain to take but is subject to diminution by reason of others becoming entitled to take is a vested remainder subject to open. Vested remainders may be subject to total divestment if possession is subject to being defeated by the happening of a condition subsequent. Here, the daughter's two children have a remainder because, on the expiration of the daughter's life estate, they will be entitled to possession of the property. The remainder is not subject to a condition precedent and the beneficiaries are in existence and ascertained, so the remainder is vested, not contingent. The remainder is subject to open because the daughter may have more children. Finally, the remainder is subject to total divestment because the daughter's children's right to possession is subject to being defeated by the daughter's ceasing to paint. (A) is wrong because the remainder is vested, not contingent; i.e., it is not subject to a condition precedent, and the beneficiaries are ascertainable.

A gang member threatened to kill the defendant unless he robbed a convenience store and gave the proceeds to the gang member. The gang member also demanded at gunpoint that the defendant kill the clerk to prevent identification. In abject fear of his life, the defendant did everything that the gang member requested. If the defendant is arrested and charged with murder and robbery in a common law jurisdiction, what result? A The defendant should be convicted of murder and robbery. B The defendant should be acquitted of the robbery and convicted of murder. C The defendant should be convicted of robbery, and the killing will be reduced to voluntary manslaughter. D The defendant should be acquitted of the robbery, and the killing should be reduced to voluntary manslaughter.

(B) The defendant should be convicted of common law murder, but acquitted of the robbery. At common law, murder is the unlawful killing of a human being with malice aforethought. "Malice aforethought" exists if the defendant has any of the following states of mind: (i) the intent to kill (express malice); (ii) the intent to inflict great bodily injury; (iii) a reckless indifference to an unjustifiably high risk to human life ("abandoned and malignant heart"); or (iv) the intent to commit a felony. In the instant case, malice could be found either by the intent to kill (because the clerk was intentionally killed to prevent identification) or by the intent to commit a felony (the killing was committed during the course of a robbery). Robbery is an aggravated form of larceny and consists of the following elements: (i) a taking; (ii) of the personal property of another; (iii) from the other's person or presence; (iv) by force or intimidation; (v) with the intent to permanently deprive him of it. Clearly, the elements for robbery are met here. Thus, at first glance, the defendant has committed both murder and robbery. However, the fact pattern also raises the defense of duress. A person is not guilty of an offense, other than intentional homicide, if he performs an otherwise criminal act under the reasonable belief that another will imminently inflict death or great bodily harm on him or an immediate family member if he does not commit the criminal act. In the instant case, the defendant committed the robbery under duress and thus should be acquitted of that charge, making (A) incorrect. However, duress would not be effective against a murder charge based on an intent-to-kill theory, and here the defendant intentionally killed the store clerk under instructions from the gang member to prevent identification. Thus, because the defendant could be convicted of an intent-to-kill murder, but acquitted of the robbery charge based on duress, (B) is the correct answer.

A defendant was convicted in federal court of possession of one kilogram of heroin with intent to distribute. She was sentenced to a prison term. Subsequently, the defendant was indicted by a federal grand jury for conspiracy to distribute the same kilogram of heroin. She moved to dismiss the indictment. Should her motion be granted? A No, because the Double Jeopardy Clause does not apply when the second prosecution is for violation of a separate statute. B No, because each prosecution requires proof of an element that the other does not. C Yes, because the Double Jeopardy Clause protects her against a second prosecution for the same criminal conduct. D Yes, because the Due Process Clause protects her against double punishment for the same criminal conduct.

(B) The defendant's motion should be denied because a prosecution for conspiracy is distinct from a prosecution for any substantive offense involving the same conduct as the conspiracy. The Fifth Amendment provides that no person shall be twice put in jeopardy for the same offense. The general rule is that two crimes do not constitute the same offense if each crime requires proof of an additional element that the other crime does not require, even though some of the same facts may be necessary to prove both crimes. Furthermore, a prosecution for conspiracy is not barred merely because some of the alleged overt acts of that conspiracy have already been prosecuted. [United States v. Felix (1992)] Here, both the conspiracy charge and the possession charge require proof of an element that the other charge does not; hence, there is no double jeopardy problem with the indictment. (A) is incorrect because it is too broad a statement. The fact that separate statutes are involved does not establish that these are not the "same offense" for purposes of double jeopardy. (C) is incorrect because the "same conduct" test is not currently used by the Supreme Court to evaluate a double jeopardy claim. (D) is incorrect because the question involves the defendant's motion to quash an indictment and not her ultimate punishment.

The state has the following homicide statutes: Murder is the unlawful killing of a human being with malice aforethought. Such malice may be express or implied. It is express when there is manifested a deliberate intention to unlawfully take away the life of a fellow creature. It is implied when no considerable provocation appears or when the circumstances attending the killing show an abandoned and malignant heart. All murder that is perpetrated by willful, deliberate, or premeditated killing or committed in the perpetration of or attempt to perpetrate arson, rape, robbery, or burglary is murder of the first degree. All other kinds of murders are of the second degree. The defendant and her associate entered a jewelry store to shoplift a diamond bracelet. Just as the defendant put the bracelet into her pocket, a sales clerk saw her and grabbed her by the wrist. The associate grabbed a knife from one of the silver displays and lunged at the sales clerk, but then a store guard shot and killed her. The defendant is charged with the first degree murder of her associate. Which of the following is the defendant's strongest argument? A The defendant cannot be convicted of murder because when they went into the store they were not carrying any weapons; therefore, there was no felony on which the felony murder rule may arise. B The defendant cannot be convicted of murder because the associate's death was not murder but justifiable homicide. C The defendant cannot be convicted of murder because she and her associate had an agreement never to use violence when they stole anything. D The associate did not intend to hurt the sales clerk, but just wanted to scare him so that the defendant could run.

(B) The defendant's strongest argument is that her associate's death was justifiable homicide. Most courts today would not allow the defendant to be convicted on a felony murder theory when a co-felon is killed by a third party during the crime. Some courts base this result on the fact that the person who did the killing was justified in doing so. (A) is wrong. Aside from the fact that the attempt to steal from the jewelry store is probably statutory burglary, the fact that the associate attempted to aid the defendant in stealing the bracelet by attacking the clerk with a knife is probably robbery. When or how the associate came by the dangerous weapon is immaterial. (C) is wrong because the circumstances of one co-felon breaking an agreement not to commit violence would not prevent the application of the felony murder rule if it were otherwise applicable. (D) is wrong because if the felony murder rule is otherwise applicable, the fact that the person who killed the co-felon may have mistaken the co-felon's intentions does not prevent the operation of this rule.

A defendant is on trial for manslaughter after he hit a victim in a bar, causing the victim to fall and hit his head on the marble bar top. The defendant claims that he hit the victim in self-defense after the victim lunged at him with a knife. During the prosecution's case, a witness testifies that she heard the victim's friend shout at the defendant, "You just killed a helpless man!" A defense witness is called to testify that he was there and does not remember hearing the victim's friend say anything. Should the defense witness's testimony be admitted? A No, it is irrelevant to any issue in the case. B Yes, it is proper impeachment of the prosecution's witness. C No, it is improper impeachment of the prosecution's witness because it relates to a collateral matter. D No, it is improper impeachment because it does not positively controvert the prosecution witness's testimony, as the defense witness merely says he does not remember.

(B) The defense witness's testimony should be admitted as proper impeachment of the prosecution's witness. Impeachment is the casting of an adverse reflection on the veracity of a witness. A witness may be impeached by either cross-examination or extrinsic evidence, such as by putting other witnesses on the stand who contradict the witness's testimony. Here, the defense is using the testimony of its witness to impeach the prosecution witness's testimony as to what the victim's friend said. This is proper.

A driver and his passenger were involved in an automobile accident when the driver ran a red light and crashed into another car. Due to a manufacturing defect in the automobile's airbag system, the passenger side airbag did not deploy. The passenger was killed on impact. The passenger's estate brought suit against the driver and the airbag's manufacturer. At trial it is established that the driver was negligent in running the red light. What effect would such proof have on the claim of the passenger's estate against the airbag manufacturer? A It would reduce recovery by the estate if the action against the manufacturer is based on negligence. B It would bar recovery by the estate if the trier of fact finds that the driver was the sole legal cause of the passenger's death. C It would bar recovery by the estate if it is shown that the driver is the sole legal heir of the passenger's estate. D It would have no effect on recovery by the estate as long as the action against the manufacturer is based on strict liability.

(B) The driver's negligence would bar recovery if it was the sole legal cause of the passenger's death. Regardless of the theory that the plaintiff is using in a products liability action, actual and proximate cause must be established. If the driver's negligence is the sole legal or proximate cause of the passenger's death, it would preclude the estate's suit against the airbag manufacturer because the defect was not a legal cause of the passenger's death. (A) is incorrect because the driver's contributory negligence will not be imputed to the passenger; hence, it will not reduce the estate's recovery under comparative negligence rules. (C) is incorrect. A potential beneficiary who was negligent will be subject to the jurisdiction's fault rules. Hence, his negligence will reduce his recovery under pure comparative negligence rules but will not bar it, even if he is the sole heir. (D) is incorrect. In most pure comparative negligence jurisdictions, the same comparative fault rules will apply whether the action against the manufacturer is based on negligence or strict liability. As discussed above, whether the estate can recover, and the extent of its recovery, depends on causation issues and the driver's status as a beneficiary of the estate.

A park board in a large suburb announced that it was accepting bids for renovation work on its recreation center. A builder advertised for sub-bids for the electrical work, and a local electrician submitted to the builder by electronic bidding service a sub-bid of $130,000. However, due to the bidding service's negligence, the sub-bid that the builder received from the electrician read $30,000 instead of $130,000. Because this was the lowest sub-bid that the builder received for the electrical work, and $60,000 less than the next lowest sub-bid, the builder awarded the subcontract to the electrician. Based in part on the electrician's sub-bid, the builder came up with a bid for the job that beat out all of the competition and won the job. What is the electrician's best argument to successfully refuse to perform the resulting contract? A The contract would be unconscionable. B The builder should have been alerted to the existence of a mistake in the sub-bid. C The bidding service, not the electrician, was responsible for the faulty bid. D The builder's failure to check out all sub-bids precludes enforcement of the contract.

(B) The electrician's best argument would be that the great difference between the electrician's sub-bid, as transmitted, and the next lowest sub-bid should have alerted the builder to the obvious mistake in the electrician's sub-bid. Typically, if only one of the parties entering into a contract is mistaken about facts relating to the agreement, the unilateral mistake will not prevent formation of a contract. However, if the nonmistaken party is or had reason to be aware of the mistake made by the other party, the contract is voidable by the party who made the mistake. Thus, the electrician's best argument is that the $60,000 difference between the electrician's bid of $30,000 and the next lowest bid should have alerted the builder to the existence of a mistake, so the electrician should be able to refuse to perform the contract. (C) is incorrect because, under the prevailing view, where there is a mistake in the transmission of an offer by an intermediary, the offer as transmitted is operative unless the other party knew or should have known of the mistake. Thus, the electrician would not be excused from performance based on the faulty transmission alone.

The plaintiff was exiting from a parking garage owned and operated by the city when he discovered that the exit ramp was blocked by construction barricades and a pile of broken-up concrete. No workers or detour signs were around and the plaintiff was in a hurry, so he backed up and drove down an entrance ramp that was clearly marked as such. As he came around a corner, his car was broadsided by a pickup truck. The plaintiff was seriously injured in the collision. A statute in the jurisdiction requires drivers to obey all traffic directional markings in both public and private parking lots and garages. The jurisdiction retains governmental immunity for municipalities. If the plaintiff brings a lawsuit against the city to recover for his injuries, which of the following facts will be LEAST helpful in the city's defense? A The plaintiff was aware that another exit on the other side of the garage was open. B The construction workers responsible for blocking off the exit ramp were employees of an independent contractor rather than the city. C The city does not collect fees or make a profit in the operation of the garage. D The pickup truck driver could have avoided the plaintiff but recognized him as an old enemy and deliberately ran into him.

(B) The fact least helpful to the city's defense of the plaintiff's lawsuit is the identity of the workers who blocked the exit ramp. Under vicarious liability rules, a principal will be liable for the tortious acts of an independent contractor if the duty is nondelegable on public policy grounds; included is the duty of a possessor of land to keep its premises safe for its invitees. If the workers were negligent in leaving the ramp blocked without providing another means of exiting, the fact that they were not city employees would not absolve the city of liability; hence, their identity would be of no help to the city's defense.(D) is incorrect because the pickup truck driver's conduct under these circumstances would be deemed a superseding force that breaks the causal connection between any negligence on the part of the city and the plaintiff's injury. Assuming that the city workers were negligent, the fact that an independent intervening force caused the injury generally would not cut off the city's liability, because its negligence created a foreseeable risk of that harm occurring. However, where this foreseeable harm is caused by an unforeseeable crime or intentional tort of a third party, most courts would not hold the city liable, treating the crime or tort as a superseding force. Here, while blocking the exit ramp created a foreseeable risk that someone might collide with the plaintiff, it was not foreseeable that his enemy would take that opportunity to commit an intentional tort against him. Because the pickup truck driver's conduct was unforeseeable under the circumstances in choice (D), the city would be relieved of liability for any negligence in blocking the ramp.

Two friends entered a bar looking to get money to pay off a loan shark, but with no plan how to do so. They struck up conversations with two women. The first friend left the bar, having induced one of the women to return home with him. Once in his house, the first friend told the woman that she would not be allowed to leave unless she gave him all of her money. Fearing for her safety, the woman gave him all of the cash she had in her possession. Meanwhile, the second friend remaining at the bar noticed that the other woman left her credit card on the counter. When the woman looked away, the friend picked up the credit card and put it into his pocket. Shortly thereafter, the woman realized her card was gone and accused the man of taking it. The man pretended to be insulted, slapped the victim, and went off with the credit card in his pocket. Which of the two friends can be convicted for common law robbery? A Both can be convicted. B The first friend can be convicted, but the second cannot be convicted. C The second friend can be convicted, but the first cannot be convicted. D Neither of the two friends can be convicted of robbery.

(B) The first friend can be convicted of common law robbery, but the second cannot be convicted. Robbery is the taking and carrying away of the personal property of another from the other's person or presence by force or intimidation. In the instant case, the first friend committed a robbery when he threatened the woman and told her that she could not leave without giving him all of her money. The only issue would be whether the asportation element is satisfied. However, the asportation element is satisfied by any slight moving, and it is likely that the first friend moved the money at some point during the robbery. Thus, a jury could find the first friend guilty of robbery. In the case of the second friend, however, a conviction for robbery is unlikely. Although a close call, the taking or retention of the property was not by force or intimidation in the second case. The crime against the property was already completed when the man slapped the victim. Furthermore, the slap was not to prevent the woman from physically taking the credit card back; rather, it was a ruse used to deflect the accusation that the second friend took the credit card. Thus, (A) is wrong, and this provides a second reason why (C) is incorrect.

A statute in the jurisdiction, which was enacted with the express purpose of preventing public employees from taking advantage of the status of illegal aliens, made it a felony to accept money or other benefits in exchange for issuing a state identification card. During an undercover investigation, an illegal alien was recorded offering $500 to a clerk in exchange for issuance of a card. The clerk agreed to the deal and later that day exchanged the card for the money, after which both parties were arrested. In a jurisdiction following the common law approach to conspiracy, which of the following statements is correct? A The clerk can be convicted of violating the statute and conspiracy to violate the statute, and the illegal alien can be convicted of no crime. B The clerk can be convicted only of violating the statute, and the illegal alien can be convicted of no crime. C The clerk can be convicted only of violating the statute, and the illegal alien can be convicted as an accomplice to violation of the statute. D The clerk can be convicted of violating the statute and conspiracy to violate the statute, and the illegal alien can be convicted of conspiracy to violate the statute.

(B) The illegal alien cannot be convicted of a crime under the statute because it was enacted for his protection, and the clerk cannot be convicted of conspiracy under the statute because the illegal alien, who would otherwise be liable as an accomplice, is not subject to conviction because of a legislative intent to exempt him. If a statute is intended to protect members of a limited class from exploitation or overbearing, members of that class are presumed to have been intended to be immune from liability, even if they participate in the crime in a manner that would otherwise make them liable. Thus, the illegal alien would not be liable as an accomplice under the statute, making (C) incorrect. The clerk cannot be convicted of conspiracy. One of the implications of the common law requirement that there be at least two guilty parties in a conspiracy arises when the crime involves members of a class protected by the statute. If members of a conspiracy agree to commit an act that violates a statute designed to protect persons within a given class, a person within that class not only cannot be guilty of the crime itself, as discussed above, but also cannot be guilty of a conspiracy to commit the crime. (D) is therefore incorrect. Because the member of the protected class cannot be guilty of conspiracy, if no other guilty party exists, the other member of the agreement cannot be guilty of criminal conspiracy because there were not two guilty parties to the agreement. Thus, because the illegal alien cannot be convicted of conspiracy under the statute, neither can the clerk. (A) is therefore incorrect and (B) is correct.

Pursuant to a contract, a landscaper performed $30,000 of landscape work for a homeowner. By coincidence, the homeowner and the landscaper were involved in an automobile accident that was unrelated to the landscape work. The homeowner was injured in the accident and sued the landscaper in federal district court for negligence, seeking $100,000 in damages. The homeowner and the landscaper are citizens of different states. May the landscaper assert and maintain a counterclaim against the homeowner for breach of contract, seeking the $30,000 due under the landscape contract? A No, because the Federal Rules of Civil Procedure permit a counterclaim only if the counterclaim arises from the same transaction or occurrence as the plaintiff's initial claim. B No, because, while the court has diversity of citizenship jurisdiction over the homeowner's negligence claim and the Federal Rules of Civil Procedure permit the counterclaim, the federal court does not have subject matter jurisdiction over the landscaper's contract claim. C Yes, because the Federal Rules of Civil Procedure permit the counterclaim, and the court has diversity of citizenship jurisdiction over both claims. D Yes, because the court has diversity of citizenship jurisdiction over the homeowner's negligence claim, and it has supplemental jurisdiction over the landscaper's contract claim because it is a compulsory counterclaim.

(B) The landscaper cannot assert and maintain a counterclaim against the homeowner for breach of contract because the federal court does not have subject matter jurisdiction over the contract claim. The court has diversity of citizenship jurisdiction over the negligence claim because there is complete diversity and the amount in controversy exceeds $75,000. A compulsory counterclaim (a claim that arises out of the same transaction or occurrence as the plaintiff's claim) does not need to meet the jurisdictional amount requirement for diversity jurisdiction. However, a permissive counterclaim (a claim that arises out of an unrelated transaction) must meet the jurisdictional amount requirement. Here, the landscaper's contract claim would be a permissive counterclaim because it is unrelated to the negligence claim arising from the accident. Therefore, the claim's amount in controversy would have to exceed $75,000. The claim is only for $30,000; therefore, the court does not have subject matter jurisdiction over the contract claim. (A) is incorrect because a defendant may assert claims against a plaintiff that are unrelated to the plaintiff's claims, provided there is subject matter jurisdiction. (C) is incorrect because the court does not have diversity jurisdiction over the contract claim. (D) is incorrect because the contract claim is not a compulsory counterclaim.

A landlord owned a prestigious downtown office building. A law firm leased the entire building from the landlord for a term of 20 years. The lease included a provision that taxes on the building would be paid by "the lessee, his successors, and assigns." The law firm occupied the building and paid the rent and taxes for eight years. At the end of the eight-year period, the law firm assigned the balance of the lease to an accounting firm and vacated the premises. The assignment was written, but there was no provision concerning the accounting firm's assumption of the duties under the lease. The accounting firm occupied the building and paid the rent and taxes for five years. At the end of the five-year period, the accounting firm subleased the building for five years to an investment company and vacated the premises. The sublease was written, but there was no provision concerning the investment company's assumption of the duties under the lease. The investment company now occupies the building and has paid the rent but not the taxes. The landlord has sued all three (i.e., the law firm, the accounting firm, and the investment company) for failure to pay the taxes. The landlord should prevail against whom? A The law firm only. B The law firm and the accounting firm, but not the investment company. C The accounting firm and the investment company, but not the law firm. D The law firm, the accounting firm, and the investment company.

(B) The law firm and the accounting firm are liable. After an assignment, the original tenant is no longer in privity of estate with the landlord. However, a tenant may still be held liable on its original contractual obligations to the landlord on privity of contract grounds. Here, the law firm is liable because it made the original deal with the landlord, which included the obligation to pay taxes on the building. The law firm remains in privity of contract with the landlord throughout the term of the lease unless it is otherwise discharged. In an assignment, the assignee stands in the shoes of the original tenant in a direct relationship with the landlord. Each is liable to the other on all covenants in the lease that run with the land, which would include the obligation of the lessee to pay taxes on the property. Here, the accounting firm is liable because as an assignee it is in privity of estate with the landlord. The accounting firm remains in privity of estate until it assigns to someone else. The sublease to the investment company is not an assignment. A sublessee is not personally liable to the landlord for rent or for the performance of any other covenants made by the original lessee in the main lease (unless the covenants are expressly assumed) because the sublessee does not hold the tenant's full estate in the land (so no privity of estate). Here, the investment company is not liable because, as a nonassuming sublessee, it is not in privity of contract or estate with the landlord. Therefore, (B) is the correct choice, and (A), (C), and (D) are wrong.

An uncle's will devised his lakefront estate "to my butler for life, remainder to my niece." The 40-acre estate includes a mansion, a 20-acre orchard, a beach, and gardens. At the time of the uncle's death, the butler was 40 years old and of modest means. The niece was 18 years old and quite wealthy. The estate was encumbered by a mortgage that was not entitled to exoneration. After the first year, the butler could no longer make the mortgage payments, so the niece paid them. Ten years after the uncle's death, the town in which the estate was located became a hot resort area. A major resort chain approached the butler with a multimillion-dollar offer for the easternmost 20 acres of the estate, which included the residence and beach. The resort chain planned to raze the mansion to erect a high-rise hotel. The butler approached the niece about the offer. He proposed to give her most of the money from the sale and offered to build any house she desired on the remaining land. The niece refused to go along with the plan. The butler decided to proceed with the sale, and the niece brought a suit to enjoin the butler's proposed actions. Which of the following is the niece's best argument? A The eventual use of the property by the remainderman will be as a residence. B Destruction of the mansion constitutes waste. C Because the niece paid the mortgage payments, the butler is subrogated to her rights. D The butler has no right to transfer his life estate.

(B) The niece's best argument is that destruction of the residence constitutes waste. The other choices do not present arguments giving her a chance of success. A life tenant is entitled to all ordinary uses and profits of the land, but he cannot lawfully do any act that would injure the interests of the remainderman. A grantor intends that the life tenant have the general use of the land in a reasonable manner, but that the land pass to the owner of the remainder, as nearly as practicable, unimpaired in its nature, character, and improvements. Even ameliorative waste, which actually increases the value of the land, is actionable if there is no reasonable justification for the change. A life tenant can substantially alter or even demolish existing buildings if (i) the market value of the future interests is not diminished and either (ii) the remainderman does not object, or (iii) a substantial and permanent change in the neighborhood conditions has deprived the property in its current form of reasonable productivity or usefulness. Here, the market value of the property would not be diminished. The remainderman (the niece), however, is objecting, making option (ii) unavailable. Furthermore, although the neighboring properties have been sold for hotels and resorts, it does not necessarily follow that the conditions have changed to such a degree that the estate should be similarly converted. The property is large enough to be somewhat isolated from the changes in the surrounding areas; thus, despite the surrounding hotels, an owner could still enjoy the land as a private residence, orchard, and beach. Therefore, the property is still useful and option (iii) is also unavailable. In this case, the life tenant's desire to raze the mansion is not because the changes in the neighborhood have made the mansion uneconomical or impractical. The life tenant can make more money by tearing the mansion down, but its usefulness and value are apparently unaffected by the changes in the neighborhood. Thus, the niece will be able to enjoin the butler from allowing the resort chain to raze the mansion and build a hotel. (A) is wrong because the fact that the niece intended to use the property as a residence is irrelevant. Even if the niece intended to change the use of the property, she is still entitled to receive the land in the condition in which it passed to the butler.

A man was driving very erratically when he was stopped by state troopers and arrested for drunk driving. He was advised of his constitutional rights and elected to remain silent. At trial for his drunk driving charge, the man testified in his own defense, stating that he had just left his doctor's office and had been administered medication without being told that it would seriously and immediately hamper his coordination. On cross-examination, the prosecutor asked whether the defendant just made up this medication story after the fact to evade legitimate liability for driving while intoxicated and the man said he had not. The prosecutor then asked why the defendant had not told the arresting officer about the medication, and defense counsel objects. The trial court should rule that the question is: A Improper, because to require the defense to inform the prosecution of defendant's testimony prior to trial would be unconstitutional pretrial discovery. B Improper, because use of defendant's post-arrest silence violates his right to due process of law. C Proper, because defendant's silence was not used as direct evidence but only for impeachment on cross-examination. D Proper, because defendant's post-arrest silence is a prior inconsistent statement which is admissible to show recent fabrication.

(B) The question was improper because it effectively comments on the defendant's post-arrest silence and thus violates his right to due process of law. A prosecutor may not comment on the defendant's silence after being arrested and receiving Miranda warnings. The warnings carry an implicit assurance that silence will carry no penalty. Thus, the defendant's invocation of his right to remain silent cannot be used as evidence against him at trial, nor can he be questioned or cross-examined about his decision to remain silent, even if he testifies at trial. Thus, (B) is the correct answer, and (C) is wrong. (Distinguish: If the defendant fails to disclose potential exculpatory evidence to the police after waiving his right to remain silent, he may be cross-examined at trial on that failure in an effort to show that it is a recent fabrication. In this case, however, the defendant invoked his right to remain silent.) (A) is wrong; the question is improper, but not because it would be unconstitutional pretrial discovery. (D) is also wrong. Although statements made without proper Miranda warnings can sometimes be used to impeach, the defendant's silence would not be classified as a prior statement.

A defendant was charged with arson (a felony) of an antique shop. Only one corner of the shop was damaged before the fire was extinguished. Under a plea agreement, the defendant pled guilty and received a suspended sentence. Because the owner of the shop had not yet insured a recently acquired 400-year-old refectory table that was destroyed by the fire, he sued the defendant for damages. At trial, the owner offers the properly authenticated record of the defendant's conviction for arson. Should the record be admitted into evidence? A Yes, as proof of the defendant's character in order to infer liability. B Yes, as proof that the defendant set the fire. C No, because the conviction was not the result of a trial. D No, because it is hearsay not within any exception.

(B) The record of the defendant's conviction should be admitted to prove that the defendant set the fire. The record of the conviction is hearsay; Under the Federal Rules, however, such judgments fall within the hearsay exception for records of felony convictions. Under the Federal Rules, judgments of felony convictions are admissible in both criminal and civil actions to prove any fact essential to the judgment, whether the judgment arose after trial or upon a plea of guilty. [Fed. R. Evid. 803(22)] For purposes of this Rule, a felony is any crime punishable by death or imprisonment in excess of one year. Arson is a felony. Consequently, a properly authenticated copy of the defendant's conviction of this crime is admissible to prove the fact that the fire that destroyed the table was set by the defendant, a fact essential to the judgment of conviction. Note that the actual plea of guilty is also admissible as a statement of a party-opponent This type of judicial admission is not conclusive, and the defendant may explain the circumstances of the plea. The plea, being an admission, is nonhearsay under the Federal Rules.

While fleeing from an armed robbery he had just committed, a man struck a pedestrian with his car, seriously injuring the pedestrian. The robber was soon apprehended and charged with armed robbery and reckless driving, both felonies. Just prior to trial, the pedestrian died from his injuries. The trial on the robbery and driving charges proceeded, and the robber was convicted of the armed robbery charge and acquitted of the reckless driving charge. The robber was then indicted under the jurisdiction's felony murder statute for causing the death of the pedestrian during the course of committing an armed robbery. The robber moved to dismiss the indictment on the ground that a second trial would violate double jeopardy. Is the robber's claim correct? A Yes, because he was acquitted of the reckless driving charge. B Yes, because the pedestrian died before the robber's first trial had begun. C No, because he was convicted of the armed robbery charge. D No, because felony murder requires proof of an additional element not required by the felony itself.

(B) The robber's claim is correct because the victim died before jeopardy attached for trial on the lesser included offense. The Fifth Amendment right to be free of double jeopardy provides that once jeopardy attaches for an offense, the defendant may not be retried for the same offense. Under the Blockburger test, two crimes do not constitute the same offense if each crime requires proof of an additional element that the other crime does not require. Under this test, a lesser included offense and the greater offense would be considered the "same offense," because the lesser included offense consists entirely of some, but not all, elements of the greater crime. Hence, under double jeopardy rules, attachment of jeopardy for the greater offense bars retrial for lesser included offenses, and attachment of jeopardy for a lesser included offense generally bars retrial for the greater offense. An exception to this latter rule exists if all of the elements for the greater offense had not occurred at the time of prosecution for the lesser offense, but in this case the final element for the felony murder charge—the death of the victim—occurred before jeopardy had attached in the first trial, so the prosecution could have added a charge of felony murder prior to proceeding with the first trial. Thus, the underlying felony of armed robbery was a lesser included offense of the felony murder and the robber's being placed in jeopardy for it bars the subsequent trial for the felony murder. (D) is incorrect because, as discussed above, two crimes are not the "same offense" for double jeopardy purposes only if each crime requires proof of an additional element that the other crime does not require. Because the underlying felony is a lesser included offense of the felony murder charge, i.e., it has no other elements not required by the felony murder charge, it constitutes the "same offense" for purposes of double jeopardy.

In a drug raid, police in a city searched 25 apartments selected at random in a 300-unit housing project. In a class action approved by the federal court, the 25 tenants sued the city for violation of their constitutional rights. The tenant named as class representative gave notice to all unnamed class members, including another tenant who decided not to opt out. The class action was then certified as a "common question" type. After negotiating with the class representative, the city police agreed to pay each tenant $500 and to conduct no further raids without proper warrants. The representative and the city signed a settlement agreement and a stipulation of dismissal of the class action. The other tenant objects to the amount of damages he is to receive and would rather opt out now and proceed on his own. May the tenant opt out now? A Yes, because a class member of a "common question" class action always has the right to opt out. B Yes, if the court allows the tenant a second opportunity to opt out. C No, because the tenant did not opt out after receiving notice of the class action. D No, if class certification was proper.

(B) The tenant may opt out if the court allows class members a second opportunity to opt out. In a "common question" class action, a judge may refuse to approve a settlement of a class action unless the class members are given a second opportunity to opt out. [Fed. R. Civ. P. 23(e)] (A) is incorrect because there are only two opt-out periods, with the initial notice and with the settlement agreement, and the latter only applies if the judge requires it. (C) is incorrect and reflects a prior version of Rule 23. (D) is incorrect for the reasons stated above.

An environmentalist divided her 25-acre property into 100 quarter-acre residential lots. At the time the environmentalist sold her lots, there was a recycling center about one mile from the western boundary of the development. She included in the deed of all 100 grantees the following provision: "Grantee covenants for herself and her heirs and assigns that all aluminum cans, glass bottles, and grass clippings of Grantee and her heirs and assigns shall be recycled. This covenant runs with the land and shall remain in effect as long as there is a recycling center within five statute miles of the development." A buyer purchased a lot in the development. Her deed, which contained the recycling clause, was duly recorded. Two years later, the buyer decided to give the property to her niece as a gift. The niece's deed to the property contained the recycling covenant, and she too recorded her deed. Shortly after the niece took possession of the house, the recycling center moved its location to a new site about four and a half miles from the development. When the niece put the house up for sale, she said nothing to prospective buyers about recycling. The house was purchased by a veteran who had lost the use of his legs. The veteran's deed did not contain the recycling clause, and he hired a local disposal service to carry away his garbage and a landscaper to maintain the yard. The landscaper bagged the grass clippings and they were removed by the disposal service, which put all the trash and clippings in a landfill. When the veteran's neighbors informed him of his duty to recycle, he told them that he knew nothing of the covenant and that it would be difficult for a person in his physical condition to haul cans, bottles, and clippings to the recycling center. Unfazed, the neighbors filed suit to require the veteran to comply with the covenant or pay damages. The veteran's best defense is which of the following? A The veteran's deed did not contain the covenant. B The covenant does not touch and concern the land. C An intelligent inspection of the neighborhood would raise no inference that the covenant existed. D The veteran's physical condition requires a balancing of hardships by the court.

(B) The veteran's best defense is that the covenant does not clearly "touch and concern" the land. While recycling may benefit the community at large, "touch and concern" involves the relationship between landowners at law. Recycling by the veteran does not directly benefit the other landowners in the use and enjoyment of their land. Thus, (B) is correct. (A) is wrong because even though the veteran's deed does not contain the covenant, he has record notice because the restriction is in his chain of title. (C) is wrong because servitudes implied from a common scheme apply only to negative covenants, and the recycling requirement is an affirmative covenant. Thus, this defense does not go to the point. (D) is wrong because it goes only to issues in equity. The suit includes a claim for damages at law. In any case, balancing of hardships is not generally applied in such cases (although some courts might elect to do so).

While at a party, the defendant ran into an acquaintance. The acquaintance proceeded to ridicule the defendant about his looks. After an hour of verbal abuse by the acquaintance, the defendant suddenly took a champagne bottle that was on a nearby table and struck the acquaintance over the head, killing him instantly. At his arrest, the defendant told the police that voices inside his head told him to shut the acquaintance up, permanently. The defendant was tried in a jurisdiction that follows the Model Penal Code test for insanity. At trial, the defendant's lawyer introduced psychiatric testimony indicating that the defendant suffered from a mental illness. Which of the following, if proved by the defense, would most likely relieve the defendant of criminal responsibility? A The defendant's actions were a product of his mental illness. B The defendant could not appreciate the criminality of killing the acquaintance, or he could not conform his conduct to the requirements of the law. C The defendant did not know that killing the acquaintance was wrong, or he could not understand the nature and quality of his actions. D The defendant was unable to control himself or conform his conduct to the law.

(B) This choice states the Model Penal Code test. Pursuant to the Model Penal Code, a defendant is entitled to acquittal if he suffered from a mental disease or defect and as a result lacked substantial capacity to either: (i) appreciate the criminality of his conduct; or (ii) conform his conduct to the requirements of law. (A) is wrong because it would be helpful only if the jurisdiction followed the Durham insanity test, pursuant to which a defendant is entitled to acquittal if his crime was the product of mental disease or defect. (C) is wrong because it presents a valid defense under the M'Naghten rule, which provides for acquittal if a disease of the mind caused a defect of reason, such that the defendant lacked the ability at the time of his actions to either: (i) know the wrongfulness of his actions; or (ii) understand the nature and quality of his actions. (D) is wrong because it presents the irresistible impulse test, which provides for acquittal if, because of mental illness, the defendant was unable to control his actions or to conform his conduct to the law. Note that the Model Penal Code test combines the M'Naghten and irresistible impulse tests. Thus, choices (C) and (D) contain elements of the Model Penal Code test, but are not as good as (B) because the question asks for the set of facts that gives the defendant the greatest likelihood of being relieved of criminal liability. Therefore, (B), which sets forth the complete test used in the jurisdiction, is the best choice.

Which of the following statements regarding speedy trials is correct? A Delays caused by counsel assigned by the court to the defendant should ordinarily be attributed to the state. B One factor in determining whether a defendant's right to a speedy trial was violated is whether the defendant asserted his right. C The remedy for a violation of the constitutional right to a speedy trial is dismissal without prejudice. D A defendant is entitled to speedy trial relief for the period between the dismissal of charges and later refiling.

(B) Whether a defendant asserted his right is one factor in determining whether the defendant's right to a speedy trial was violated. The determination is made by an evaluation of the totality of the circumstances, and the following factors should be considered: (i) length of the delay, (ii) reason for the delay, (iii) whether the defendant asserted his right, and (iv) prejudice to the defendant. The remedy for a violation of the constitutional right to a speedy trial is dismissal with prejudice. Delays caused by counsel assigned by the court to the defendant should ordinarily be attributed to the defendant and NOT to the state. A defendant is NOT entitled to speedy trial relief for the period between the dismissal of charges and later refiling

A credit card company filed a civil action against a consumer in federal district court seeking to recover the unpaid balance on the consumer's account. The credit card company properly served process on the consumer, but the consumer failed to file or serve a timely answer to the complaint. The clerk of court on motion of the credit card company made an entry of default, but default judgment has not been entered. On which grounds may the court set aside the entry of default? A For the same limited grounds for which any court judgment may be set aside. B Only if the court finds that the consumer could not have filed and served a timely answer despite using reasonable diligence. C If the consumer demonstrates that there was good cause for his failure to file and serve a timely answer and that he has a viable defense. D On any grounds that the court, in its discretion, finds just.

(C) An entry of default may be set aside for "good cause shown." Although not specifically required by the Federal Rules, a majority of courts also will require some showing of a meritorious defense. (A) is incorrect because the grounds are not limited to the grounds required for setting aside regular court judgments. (B) is incorrect because that answer is also too limiting on the concept of "good cause shown." For example, a default might result from an honest mistake of the attorney, but the entry of default may nonetheless be set aside if the attorney acts promptly to correct the mistake. (D) is incorrect because a majority of courts will also require a showing of a meritorious defense.

The owner of a valuable painting hired professional movers to transport it to an auction house when she decided to sell it. As the movers were carrying it to their van, a window air conditioner that a tenant had been trying to install fell out of his second floor window and crashed through the painting and onto the ground. The owner had been watching from her apartment across the street and saw her painting destroyed. She became extremely upset and needed medical treatment for shock. If the owner brings a claim for negligent infliction of emotional distress against the tenant, is she likely to recover? A Yes, because she suffered physical symptoms from her distress. B Yes, because she was a foreseeable plaintiff. C No, because she was not within the zone of danger. D No, because she suffered no physical impact.

(C) Because the owner was across the street at the time of impact, she was not within the zone of danger, thus precluding her recovery for negligent infliction of emotional distress. A defendant breaches a duty to avoid negligent infliction of emotional distress when he creates a foreseeable risk of physical injury to the plaintiff through causing a threat of physical impact that leads to emotional distress. Damages generally are recoverable only if the defendant's conduct causes some physical injury, rather than purely emotional distress (although a severe shock to the nervous system that causes physical symptoms is sufficient). If plaintiff's distress is caused by threat of physical impact to her, she must have been within the zone of danger. Here, the owner witnessed the air conditioner striking her painting from across the street. This vantage point placed her outside the zone of danger from the falling air conditioner. Thus, the owner cannot recover for negligent infliction of emotional distress.

After the release of various news stories about the President's possible violation of political campaign funding laws, a federal grand jury investigation and an investigation by a special Senate subcommittee were initiated. The Senate subcommittee subpoenaed documents and records from several top officers of the executive branch. Learning of the subpoenas, the President ordered all executive officials to refuse to turn over materials, claiming "executive privilege." Which of the following statements is most accurate? A The subpoena violates the constitutional principle of separation of powers. B The President's executive privilege is absolute, except in cases of impeachment. C The presidential papers are presumptively privileged, but the privilege must yield to a demonstrated specific need for evidence in a pending legislative proceeding. D The President's executive privilege applies to proceedings by Congress, but not to proceedings by the courts.

(C) Executive privilege is an inherent privilege necessary to protect the confidentiality of presidential communications. Under this privilege, presidential documents and conversations are presumptively privileged, but this privilege must yield to a demonstrated need for such materials as evidence in a criminal case in which they are relevant and otherwise admissible. [United States v. Nixon (1974)] Although the Supreme Court has not expressly decided that the privilege must also yield to a demonstrated need for evidence in a pending legislative proceeding, such an extension of Nixon is likely, and none of the other alternatives is at all accurate.

A corporation was in the business of purchasing real property at below-market prices and reselling the properties to investors. The bylaws of the corporation authorized the chief executive officer ("CEO") and the director of the marketing division to enter into contracts on behalf of the corporation for the purchase or sale of properties. The corporation had recently purchased a large parcel of beachfront property for resale. The CEO secretly opened negotiations with an amusement park to sell the property. However, unknown to the CEO or anyone else in the corporation, the marketing director had already reached an agreement with a hotel for the sale of the property. On April 23, the marketing director and the hotel signed a written contract providing for sale of the property by the corporation to the hotel for $35 million. On April 25, the board of directors amended its bylaws, effectively depriving the marketing director of authorization to bind the corporation in purchase or sale transactions. This action was immediately publicized and became known to both the marketing director and the hotel. On April 26, the hotel duly recorded its contract. On May 1, the CEO, still unaware of the marketing director-hotel agreement, approved sale of the property to the amusement park for $39 million. The necessary documents of title were prepared and properly recorded by the amusement park on May 5. Two days later, the amusement park learned of the marketing director-hotel agreement. On May 10, the date scheduled for closing of the hotel's sale agreement, the CEO refused to accept the hotel's tender of $35 million and refused its demand for a deed to the property. The hotel subsequently brings action against the corporation and the amusement park for specific performance and to quiet title to the property. For whom will the court likely rule? A The defendants, because the board of directors had deprived the marketing director of authority to bind the corporation in the sale of real property. B The defendants, because the amusement park is the only purchaser who properly recorded a deed to the property. C The hotel, because the amusement park had constructive notice of the hotel's interests in the property when the agreement with the CEO was made. D The hotel, because the attempt to divest the marketing director of authority to approve sales of the corporation's property was invalid.

(C) Judgment should be for the hotel regardless of whether the jurisdiction has a notice statute or a race-notice statute. Under either type of recording statute, the only persons protected by the statute are bona fide purchasers. To attain this status, the person must take without notice—either actual, constructive, or inquiry—of the prior instrument. Because the marketing director-hotel contract was properly recorded, the amusement park had constructive notice of the hotel's interest in the property. Thus, the park could not become a bona fide purchaser when it entered into its contract. (A) is wrong because the marketing director had not been deprived of authority to bind the corporation at the time she signed the agreement with the hotel, and any subsequent change in her powers did not affect the validity of that agreement, nor the hotel's power to subsequently record the agreement. (B) is wrong because the hotel recorded its contract of sale. Any instrument creating or affecting an interest in land (e.g., deed, mortgage, contract to convey) can be recorded, providing constructive notice to subsequent purchasers. Thus, the hotel's failure to record a deed does not deprive it of protection of the recording statute. (D) is wrong because regardless of the validity of the board's attempt to divest the marketing director of authority, it is immaterial to the hotel's rights; it came after the valid marketing director-hotel contract was properly signed by the marketing director.

The United States was involved in a dispute with a small island nation over the ownership of an archipelago. On discovering that the archipelago was rich in oil, the President announced that he would appoint an ambassador to negotiate a treaty with the island nation to jointly exploit the oil reserve. A majority of Senators believed that the island clearly belonged to the United States and did not want to negotiate with the island nation. They passed a resolution requiring the President to include a Senator in his diplomatic mission to ensure that the Senate's view was presented in any negotiation with the island nation. What is the strongest constitutional ground for the President's refusal to do so? A As commander in chief, the President has the exclusive power to determine how to protect our national interest abroad. B The resolution is unreasonable because it includes a Senator and not any Representatives. C The President has the exclusive power to select diplomatic representatives of the United States. D The Senate, if it does not like the President's actions, can refuse to appropriate the necessary monies for the President to implement his policies.

(C) The President's strongest argument is that the power to select ambassadors is vested by the Constitution in the President, and the Senate's only power in this respect is to advise and give (or withhold) its consent. The Senate is not given the power to force ambassadors on the President.

A defendant was convicted after a jury trial of violation of federal statutes prohibiting the sale of automatic weapons to foreign nationals. It was established at trial that the defendant had purchased a number of stolen United States Army heavy machine guns and attempted to ship them abroad. The trial court expressly based its imposition of the maximum possible sentence for the conviction on the defendant's refusal to reveal the names of the persons from whom he purchased the stolen weapons. His counsel argues that this consideration is reversible error. If the defendant appeals the sentence imposed, what should the appeals court do? A Reverse the trial court, because the consideration of the defendant's silence violates his Fifth Amendment privilege against self-incrimination. B Reverse the trial court, because the consideration of collateral circumstances in sentencing violates his due process rights. C Affirm the trial court, because the right to remain silent granted by the Fifth Amendment does not include the right to protect others from incrimination. D Affirm the trial court, because citizens must report violations of the criminal statutes.

(C) The appeals court should affirm the trial court because the right to remain silent does not include the right to protect others from incrimination. The defendant was not privileged to refuse revealing the names of the stolen weapon sellers. The United States Supreme Court held, in Roberts v. United States (1980), that a defendant's refusal to cooperate with an investigation of the criminal conspiracy of which he was a member may properly be considered in imposing sentence. This is because the Fifth Amendment right to remain silent does not afford a privilege to refuse to incriminate others. (C) is therefore correct and (A) is incorrect. (B) is incorrect because the court's consideration of the defendant's refusal to cooperate does not violate due process. (D) is not an accurate statement of the law.

A landowner owns 15 acres of undeveloped property. He plans to build a stadium complex on the property to house a football team two years from now, but would like to open the 15 acres to public use for picnicking and similar activities until then. Which of the following would best accomplish the landowner's goal? A Dedicate the 15 acres for use as a public park. B Lease the 15 acres to the city for two years. C Grant the city an easement for public recreational uses for two years. D Covenant that the city may use the 15 acres for recreation for two years.

(C) The best way for the landowner to accomplish his goals is to grant the city an easement for recreational use for two years. An easement would allow the city to use the land only for the purposes provided for in the easement, and the landowner could limit the purposes to recreational uses. Thus, (C) is the best answer. (A) would not be a good choice because if the landowner dedicated the land to public use, he would be giving title to the land to the government, so he would not be able to reclaim the land and build his stadium in the future. (B) would not be a good choice because a lease would give the city more control over the land than an easement, and would be more complicated to create. A lease grants the lessee the exclusive right to possess the premises, and broad rights to use them in any manner, unless specifically restricted. Thus, if the landowner leases the land to the city, he would not have access to the land, and if he wanted it used only for recreational purposes, he would have to specifically restrict any undesired uses. Any restriction not included in the lease will be unenforceable. An easement, on the other hand, grants only a limited interest in the land—to use it for only those purposes stated in the easement; thus, it would be better than a lease. (D) is not a good choice because covenants usually are made in conjunction with a lease, deed, or other instrument; they promise some act or forbearance with respect to property and are generally not used to grant rights for access to property.

A grandfather told his granddaughter that she could have his house because he was moving to a retirement home, and entered into a valid contract to convey it to her. He promised her that he would have another wing added to the house in the back before turning it over to her, and entered into a written contract with a builder to construct the addition for his granddaughter. Before the grandfather had entered into the contract with the builder, the granddaughter had paid $5,000 for a 60-day option to purchase another house because she was not sure she would like the addition. However, when her grandfather showed her the plans for his house prepared by the builder, she liked it very much and decided to let her option to purchase the other house lapse. Shortly thereafter, the local zoning authority increased the minimum lot line setbacks, making it impracticable to put the addition on the back of the house. The builder offered to put an addition above the existing floor rather than in the back, and the grandfather agreed. After the granddaughter's option had lapsed, she discovered that the addition was now going up rather than in the back. She angrily demanded that the builder either build the addition according to the original specifications that she approved or pay her damages. The builder refused and the granddaughter filed suit. Who is more likely to prevail? A The granddaughter, because she was an intended beneficiary of the contract whose rights had vested. B The granddaughter, because the subsequent agreement between her grandfather and the builder to modify the construction was unsupported by consideration. C The builder, because he may raise all defenses that he had against the grandfather against the granddaughter. D The builder, because the granddaughter is merely an incidental beneficiary of the contract between the grandfather and the builder and, as such, has no power to enforce the contract against the builder.

(C) The builder will prevail because he may raise all defenses that he had against the grandfather against the granddaughter. The granddaughter is an intended third-party beneficiary of the contract between the grandfather and the builder. Generally, a third-party beneficiary has rights under the contract as soon as she does something to vest her rights (manifests assent to the promise, brings suit to enforce the promise, or materially changes position by justifiably relying on the promise). Here, the granddaughter materially changed her position by justifiably allowing her option on the other house to lapse. Generally, once the third-party beneficiary's rights have vested, the original contracting parties may not modify the contract without the assent of the third-party beneficiary. However, the third-party beneficiary is subject to any defenses that the promisor could have used against the original promisee, and here the builder could have used the defense of impracticability against the promisee. Therefore, he could use that defense against the granddaughter to avoid having to pay damages for not building the house as he originally agreed. (A) is incorrect because although it is true that the granddaughter's rights had vested, the answer fails to take into account the defenses available to the builder.

A buyer and seller entered into a written contract on March 31 for the sale of a beach house. Under the terms of the agreement, the buyer would purchase the house for $275,000, with 10% due at closing on May 1 and a 15-year mortgage. At the time the contract was entered into, the parties agreed orally that the written agreement would not become binding unless the buyer notified the homeowner, in writing, by the end of the day on April 15, that she had secured the proper financing. With the summer season approaching, the seller did not wish to risk any delay in selling the house if the buyer was not in a position to buy it. On the morning of April 15, the buyer's financing was approved. On April 16, the buyer telephoned the seller and told him that her financing had been approved. The buyer also told the seller that she was not able to get written confirmation to him by April 15 because of the postal workers' slowdown and because her fax machine just broke down. The seller assured the buyer that this was not a problem. However, before closing, the seller had a change of heart and decided not to sell the beach house after all. The buyer files an action for breach. Which of the following would be the basis for the buyer's best argument? A Statute of Frauds. B Parol evidence rule. C Waiver of condition. D Excuse of condition by hindrance.

(C) The buyer's best argument is that the seller's assurances that there was no problem with the buyer's failure to provide written notification by April 15 amounts to a waiver of the condition. The buyer's written notification by April 15 that he had obtained the proper financing was a condition precedent to the seller's absolute duty to perform under the contract. It is clear that the buyer did not provide the required notification by April 15; thus, the condition was not fulfilled. However, one having the benefit of a condition may indicate by words or conduct that he will not insist upon it. When a condition is broken, the beneficiary of the condition has an election: (i) he may terminate his liability; or (ii) he may continue under the contract. If a choice is made to continue under the contract, the person is deemed to have waived the condition. The seller was fully aware that the buyer had not satisfied the condition, yet, when speaking with the buyer on April 16, he stated unequivocally that it was not a problem. This is a definite indication that the seller elected to continue under the contract. Having so elected, the seller is deemed to have waived the condition. Therefore, the seller's duty of performance under the contract became absolute. Regarding (A), the only way the Statute of Frauds could bolster the buyer's position would be if the original oral agreement setting forth the condition were required by the Statute to be in writing. If that were the case, the buyer could argue that the condition is unenforceable because it is not in writing. However, the oral agreement is not of a type that falls within the purview of the Statute of Frauds. Therefore, the Statute of Frauds will provide no help to the buyer. Similarly, regarding (B), the parol evidence rule could help the buyer only if it could be used to preclude admissibility of the original oral agreement. Under the parol evidence rule, where the parties to a contract express their agreement in a writing with the intent that it embody the final expression of their bargain, any expression made prior to the writing and any oral expression contemporaneous with the writing is inadmissible to vary the terms of the writing. However, where it is asserted that there was an oral agreement that the written contract would not become effective until the occurrence of a condition, evidence of the oral agreement may be offered and received. Because the original oral agreement between the seller and buyer established a condition precedent to the effectiveness of the written agreement, the buyer will be unable to raise the parol evidence rule as a bar to the admissibility of evidence relating to the oral agreement. (D) is unsupported by the facts. If a party with a duty of performance that is subject to a condition prevents the condition from occurring, he no longer has the benefit of the condition. This is referred to as excuse of condition by hindrance. The seller did nothing to prevent the occurrence of the condition regarding written notification by April 15. Consequently, it cannot be said that the condition is excused by hindrance.

A decedent died without having executed a will, leaving a substantial estate to be distributed by the probate court. The jurisdiction's applicable statute provides that where a decedent leaves neither issue nor spouse, nor parents, his estate goes to his brothers and sisters and their descendants. The decedent was never married, had no children, and both of his parents are dead. A woman whose birth certificate was destroyed by fire seeks to establish that she is the daughter of the decedent's only sibling, who is now also deceased. The woman offers into evidence a statement in a properly recorded trust instrument. The instrument was executed by the decedent's father and recited that certain specified real property conveyed by the decedent's father into the trust should be held for her benefit, as "my loving granddaughter." The document actually offered is an enlarged print photocopy of microfilm records, authenticated by an employee of the county. What should the trial court do? A Exclude the evidence, because it is not the best evidence. B Exclude the evidence, because it is inadmissible hearsay not within any recognized exception. C Admit the evidence, because it is a record of a document affecting an interest in property. D Admit the evidence, because it constitutes a past recollection recorded.

(C) The court should admit the evidence. Statements in a document affecting an interest in property are admissible, pursuant to Federal Rule 803(15), if they are relevant to the purpose of the document. Thus, (B) is incorrect. (A) is incorrect because properly authenticated copies of recorded writings may be used in lieu of originals. [Fed. R. Evid. 902(4)] (D) is incorrect because the trust instrument cannot qualify as a recorded recollection; there is no witness testifying that he made or adopted the writing while the events were fresh in his mind and he has no present recollection.

A foreign student who had entered the United States on a student visa four years ago was notified by federal immigration authorities that he was subject to being deported because his visa had expired. Federal law provided that an alien who is subject to being deported has the right to appear before an administrative officer appointed by the Attorney General's office for a hearing on whether he should be deported. This officer, appointed by the executive branch of the government, has the right under law to make a final order concerning whether the alien should be deported. After a hearing, the administrative officer entered an order allowing the student to remain in the United States as a permanent resident. However, a congressional rule permitted the House of Representatives, by resolution, to deport "undesirable aliens." After the administrative judge entered his order, the House passed a resolution that the student should be deported. The student petitioned the federal court to declare the legislative resolution invalid. Should the court find the resolution to be valid? A Yes, because Congress has plenary powers with regard to aliens and naturalization. B Yes, because aliens are not "citizens" within the meaning of the Fourteenth Amendment. C No, because the federal law removed congressional power with regard to aliens in this circumstance, and the resolution of the House violates the separation of powers doctrine. D No, because the student was denied due process when he was not given a hearing before the House of Representatives.

(C) The court should find the resolution invalid. While Congress has broad power to delegate, the separation of powers doctrine forbids Congress from trying to control the exercise of the power delegated in various ways, such as by overturning an executive agency action without bicameralism (i.e., passage by both houses of Congress). By enacting the federal law allowing the administrative law judge to enter a final order with regard to aliens, Congress has given up any control it may have had previously in these situations. The resolution by the House here is an unconstitutional legislative veto that violates the separation of powers doctrine. (A) is incorrect because, while Congress does have plenary power over aliens with regard to immigration and naturalization, here it has given up control over this area by enacting a law allowing an administrative officer appointed by the executive branch to make a final order concerning whether an alien should be deported. (B) is incorrect because the fact that aliens are not citizens has no bearing on whether the House resolution violated the Constitution. (D) is incorrect because, while resident aliens are entitled to notice and hearing before they can be deported, the student did receive a hearing before the administrative officer. There is no requirement that persons affected by legislative action have the right to be heard by the legislative body taking the action. Thus, the better argument as to why the resolution was invalid is based on separation of powers.

An elderly woman filed a complaint in federal district court, alleging that a salesperson "made fraudulent statements that induced the plaintiff to sign the contract to the plaintiff's detriment." The salesperson then filed a motion to dismiss the complaint for failure to state a claim upon which relief can be granted. How should the court rule? A Deny the motion, because the complaint stated a possible claim for relief. B Deny the motion, because the complaint stated a plausible claim for relief. C Grant the motion, because the complaint did not state the claim for relief with particularity. D Grant the motion, because the complaint did not state the claim for relief with probability.

(C) The court should grant the motion to dismiss for failure to state a claim upon which relief can be granted. The federal pleading rules generally require the pleader to assert short and plain statements in the complaint to put the other side on notice of the claim being asserted; detailed assertions of facts underlying the claim are generally not required. However, there are certain special pleading rules that require a party to state more detail under special circumstances, including claims that assert fraud or mistake. Under such special circumstances the federal rules specifically require that a plaintiff assert the claim for relief with particularity. Here, choice (C) is the best answer because it is the only choice to correctly state the rule. The salesperson's motion to dismiss should be granted in this case because the plaintiff's complaint fails to state the claim with particularity.

A developer owned a 240-acre parcel of land zoned for commercial and residential use. He prepared and recorded, after obtaining approval from all appropriate agencies, a subdivision plan that included a commercial center and a number of lots for single- and multi-family residences. The list of covenants, conditions, and restrictions recorded with the plan included provisions that required every building constructed in the subdivision to be of "simulated adobe style" architecture approved in advance by an association. A year later, the developer sold many of the lots in the commercial center, including several to a real estate firm. Each deed prepared by the developer contained a reference to the design restriction in the recorded plan. The developer also sold almost all of the residential lots, the deeds of which contained the same reference to the restriction. The following year, the real estate firm sold one of its lots to a burger franchise. The deed contained no reference to the design restriction. The franchise's prefabricated restaurant, complete with a giant burger logo mounted on the roof, was constructed over the weekend. A merchant, an original purchaser of one of the commercial lots, owned the lot next to the burger franchise. She did not learn of construction of the restaurant until she came in to work on Monday, and saw the giant burger logo. The merchant brings an action seeking a mandatory injunction compelling the burger franchise to demolish the restaurant. At trial, the merchant proves that the burger franchise did not seek or obtain approval of the association for its building. Should the court issue the injunction? A No, because destruction of the restaurant would be a tremendous waste of resources. B No, because the burger franchise's deed contained no restriction on the type of building that could be constructed on the lot. C Yes, because the restrictive covenant runs with the land. D Yes, unless the burger franchise can establish to the court's satisfaction that its restaurant design has at least as much aesthetic merit as any "simulated adobe style" design.

(C) The court should issue the injunction because the covenant runs with the land. A covenant will be enforceable as an equitable servitude—allowing a covenantee, covenantor, or successor to enforce the covenant in equity by way of injunction—when there is (i) a covenant in a writing satisfying the Statute of Frauds, that (ii) touches and concerns the land (i.e., the effect of the covenant makes the land more useful or valuable to the benefited party) and that (iii) indicates an intention that the servitude exists, and (iv) notice is given to future owners of the burdened land. Here, the covenant was in writing in the subdivision plan and presumably it satisfied the Statute of Frauds. It touches and concerns the land—benefiting all of the lots and burdening all of the lots. The intention to create the servitude is established by the writing and can also be implied from the common scheme for development. There was sufficient record notice of the covenant because the plan was recorded and was noted in all of the original deeds prepared by the developer, including the one in the burger franchise's chain of title. Thus, the covenant is enforceable and (C) is the best answer.

A company that was the leading supplier of home water filtration systems had a network of sales promoters who were under contract for two- or three-year terms and were compensated solely by commissions earned from sales and by occasional bonuses. Veteran promoters also earned commissions by recruiting other promoters for the company. One of the company's veteran promoters was contacted by a former top sales representative for another manufacturer who was looking for similar sales opportunities in the region. The sales rep knew that the promoter might be able to get her a position with his company, which was looking for additional promoters. At the time he met with the sales rep, the promoter's contract with the company had one more month to run. When the promoter's contract with the company expired, he announced that he was forming his own business to market a different line of water filtration systems manufactured by a competitor of the company, and that the sales rep would be in charge of his promotional network. The company brought an action against the promoter for interference with business relations for hiring the sales rep. At a preliminary hearing, the parties stipulated to the above facts and that the promoter was an independent contractor rather than an employee of the company. The promoter then filed a motion for a summary judgment in his favor. Should the court grant the promoter's motion? A Yes, because the sales rep had no business relationship with the company at the time the promoter's alleged interference occurred. B Yes, because the promoter was an independent contractor rather than an employee of the company. C No, because the jury could find that the means the promoter used to obtain the sales rep were not privileged. D No, because the jury could find that the promoter breached his contract with the company by meeting with the sales rep.

(C) The court should not grant the promoter's motion because the jury could find that the promoter used improper means, while working for the company, to divert the sales rep for his own purposes. To establish a prima facie case for interference with business relations, the following elements must be proved: (i) existence of a valid contractual relationship between plaintiff and a third party or a valid business expectancy of plaintiff; (ii) defendant's knowledge of the relationship or expectancy; (iii) intentional interference by defendant that induces a breach or termination of the relationship or expectancy; and (iv) damage to plaintiff. Thus, a plaintiff has a cause of action for interference with probable future business relationships for which the plaintiff has a reasonable expectation of financial benefit. On the other hand, an interferer's conduct may be privileged where it is a proper attempt to obtain business for the interferer, particularly if the interference is only with a prospective business relationship rather than with an existing contract. What is proper depends on various factors, including the means of persuasion used. Here, the promoter's conduct would not be privileged if the jury were to find that he improperly used his position with the company to develop a relationship with the sales rep.

A defendant visited her doctor to seek treatment for a bullet wound. While he was treating the wound, the doctor asked the defendant how she was shot. The defendant replied that she was struck by a police officer's bullet while running away from a jewelry store she had robbed, but she implored the doctor not to tell this to anyone. The doctor promised that he would not. Although the defendant was never charged by the police, the owner of the jewelry store brought suit against her seeking the value of the stolen goods. The defendant denied robbing the store. At the trial, the owner calls the doctor to testify to the statement made to him by the defendant. The defense attorney objects on the ground that such testimony is barred by the jurisdiction's physician-patient privilege. Should the objection be sustained? A Yes, because the doctor acquired this information while attending the defendant in the course of treatment. B Yes, because the doctor agreed to the defendant's specific request that this information be kept confidential. C No, because the physician-patient privilege is inapplicable to the defendant's statement. D No, because the doctor is the one who is entitled to either claim this privilege or waive it.

(C) The court should overrule the objection because the physician-patient privilege cannot be invoked for information dealing with a nonmedical matter. Under the physician-patient privilege, a physician is foreclosed from divulging in judicial proceedings information that he acquired while attending a patient in a professional capacity, which information was necessary to enable the physician to act in his professional capacity. Information given by a patient that deals with a nonmedical matter is not protected by the privilege. Hence, the defendant's admission that she was shot while running from a jewelry store that she robbed is not barred by the privilege. (A) is incorrect because, although it is true that the doctor acquired the information while attending the defendant in the course of treatment, the privilege is inapplicable because, as discussed above, the statement deals with a nonmedical matter.

The defendant planned to break into a home, steal any valuables that he could easily pawn, and then burn down the home using gasoline from his lawnmower. When the defendant got to the home that night, he realized that he had forgotten the gas at home. Nonetheless, the defendant broke into the home through a basement window. Unbeknownst to him, the police were alerted by a silent alarm and arrested the defendant just as he was leaving the home with a sack filled with valuables. At common law, what crimes has the defendant committed? A Burglary and attempted larceny. B Burglary, attempted larceny, and attempted arson. C Burglary and larceny. D Burglary, larceny, and attempted arson.

(C) The defendant has committed burglary and larceny. Burglary consists of a breaking and entry of the dwelling of another at nighttime, with the intent of committing a felony therein. The felony need not be carried out—all that is required is that the person committing the crime have the intent to commit a felony at the time of entry. At night, the defendant broke into and entered the house with the intent to commit the felony of larceny. Regardless of whether the defendant took any property or committed a burning, the burglary was complete on his breaking and entering the dwelling at nighttime with the requisite intent. The defendant has also committed common law larceny. Larceny is the taking and carrying away of the personal property of another, by trespass, with the intent to permanently deprive the owner of his interest in the property. The element of carrying away, or asportation, is satisfied as long as there is some movement of the property as a step in carrying it away. The movement need only be slight as long as it was part of the carrying away process. Here, the defendant placed valuables in a sack and started to leave the home. This movement was sufficient to constitute a carrying away. Having acted with the requisite intent to permanently deprive the true owner of his property, the defendant has committed larceny.

A plaintiff sued a defendant for negligence, seeking $100,000 in the federal court of State A, after the two were involved in a car accident while driving through State B. The defendant's defense was that the plaintiff was contributorily negligent; contributory negligence is a complete defense under the applicable state law. The plaintiff won after a full trial. May the defendant now bring a negligence claim against the plaintiff in a new suit for injuries from the accident that was previously litigated? A Yes, because the defendant's claim is not the same as that of the plaintiff. B Yes, because claim preclusion (res judicata) applies only if the claims are brought by the same parties. C No, because the counterclaim is barred as an unasserted compulsory counterclaim. D No, because the defendant's claim merged with the prior judgment and is now barred.

(C) The defendant may not bring a negligence claim against the plaintiff. This question asks not only about preclusion, but about the related concept of compulsory counterclaims. If a counterclaim arises out of the same transaction or occurrence as one of the plaintiff's claims, it is a compulsory counterclaim and must be pleaded or it will be barred. Any other counterclaim is permissive and may be asserted (assuming there is subject matter jurisdiction) even though there is no connection at all between it and the plaintiff's claim. Although the defendant's compulsory counterclaim is not claim precluded under preclusion principles (as discussed below), it is barred as an unasserted compulsory counterclaim. (A) and (B) are incorrect because, while they correctly note that there is no claim preclusion for claims against the defendant, that is not the only issue, as discussed above. (D) is incorrect because claims by the opposing party cannot be merged. Once a final judgment on the merits is reached on a claim, the claimant is barred by claim preclusion (also known as res judicata) from asserting the same claim in a subsequent suit against the same defendant. In this case, there is a final judgment on the plaintiff's negligence claim, which would bar the plaintiff from asserting any other claims relating to the same facts in a subsequent suit against the same defendant. Any such claims are considered to be "merged" with the adjudicated claim and thus bar the plaintiff from properly asserting them. However, this bar is only against the plaintiff's claim. Claim preclusion does not bar a claim by an opposing party.

After a single vehicle accident, the passenger filed a negligence action in federal district court against the driver to recover for a whiplash injury allegedly suffered in the accident. On the advice of his attorney, the passenger consulted and retained five physicians in search of one who would serve as an expert witness on his behalf at trial. Four of the physicians determined that the passenger had suffered no injury. Obviously, the passenger does not intend to use those four physicians as witnesses at trial. May the driver obtain in discovery the opinions of the four physicians whom the passenger does not intend to have testify at trial? A Yes, the passenger must disclose in his required disclosures the identity of the four physicians and provide a written report concerning their qualifications and opinions. B Yes, the driver may obtain information concerning their opinions through interrogatories, but the driver may not depose the four physicians. C No, the driver will not be permitted to discover the facts known and opinions held by these four physicians unless the driver can demonstrate exceptional circumstances under which it is impracticable to obtain facts or opinions on the same subject by other means. D No, the parties may not under any circumstances obtain discovery of opinions held by experts retained by another party when the other party does not intend to have the expert testify at trial.

(C) The driver may not obtain in discovery the opinions of the four physicians unless he can demonstrate exceptional circumstances. The opinions of experts who are retained in anticipation of litigation but who are not expected to testify at trial may be discovered only upon a showing of exceptional circumstances under which it is impracticable to obtain facts or opinions by other means or when a medical report for an exam conducted under FRCP 35 is requested. (D) is therefore incorrect. (A) is incorrect because it describes the requirements for expert witnesses who are expected to be used at trial. (B) is incorrect because the opinions of experts who will not testify at trial cannot be discovered by any means unless there is a showing of exceptional circumstances.

A homeowner, a citizen of State A, hired an electrician, a citizen of State B, to fix the wiring in her basement and hired a gas worker, also a citizen of State B, to install a new gas stove in her kitchen. Unfortunately, the home caught fire and burned down while they were both working on their separate jobs. The homeowner sued the gas worker for negligence in federal court in State A, seeking $100,000. The homeowner promptly served the gas worker, and the gas worker timely filed an answer with the court. One month after filing the answer, the gas worker moved to file and serve a third-party complaint against the electrician, alleging that the electrician was the sole cause of the accident. Which of the following arguments is most likely to achieve the electrician's goal of dismissal of the third-party complaint? A The gas worker's motion for leave to file a third-party complaint is untimely and thus should be denied as a matter of law. B The court does not have subject matter jurisdiction over the third-party complaint because the electrician's claim and the gas worker's claim do not arise from a common nucleus of operative fact. C The gas worker's claim against the electrician is not a proper third-party claim. D Dismissing the gas worker's claim will not impede his ability to protect his rights in a separate action.

(C) The electrician's best argument is that the gas worker's claim against the electrician is not a proper third-party claim. Under Rule 14, a defendant may assert a third-party claim against "a nonparty who is or may be liable to it for all or part of the claim against it." In other words, a third-party claim must be a derivative claim; the third-party plaintiff must be seeking indemnification or contribution from the third-party defendant. Here, the gas worker's claim is not that the electrician must indemnify him or that the electrician is a joint tortfeasor who may be jointly liable under principles of contribution. Rather, the gas worker is alleging that he (the gas worker) is not liable and that the electrician is. Because the claim is not derivative, it is not properly asserted as a third-party claim under Rule 14. (A) is incorrect. A defendant may serve a third-party complaint as of right within 14 days of serving his original answer. Thereafter, he must make a motion to serve the complaint, and it is within the trial court's discretion whether to grant or deny the motion. Here, it is unlikely that a court would deny a defendant's motion to serve a third-party complaint at such an early stage of the proceeding. (B) is incorrect because both claims arose from a single occurrence: the fire that burned down the homeowner's house. Thus, there would be supplemental jurisdiction for the gas worker's claim because it arises from the same set of facts as the homeowner's claim against the gas worker, which is based on diversity of citizenship. (D) the gas worker's ability to bring a separate action against the electrician is not a compelling reason for dismissing a properly asserted third-party claim

A hockey player who was playing in the final game of the season before a hostile crowd in the opponent's packed stadium had an opportunity to get his team into the playoffs, but he missed a shot into an open net as the horn sounded, ending the game. As the crowd cheered and jeered, the puck bounced back to him and he shot it in anger toward the stands. A fan who had been looking the other way turned back toward the rink just in time to be struck in the face by the puck. He suffered a broken nose and a severe gash under his eye. After the game, the league commissioner fined the player for violating league rules by intentionally directing the puck out of the playing area. If the fan sues the player for battery, will the fan likely prevail? (A) No, because by attending a hockey game, the fan assumed the risk of pucks being shot into the stands. (B) No, because the player did not have the intent to strike the fan with the puck. (C) Yes, because the player knew that it was substantially certain that a fan would be hit by the puck. (D) Yes, because the player violated league rules by intentionally shooting the puck out of the playing area.

(C) The fan will prevail in his battery action because the player had the requisite intent for battery. A prima facie case for battery requires plaintiff to prove (i) an act by defendant that brings about a harmful or offensive contact to the plaintiff's person, (ii) intent on defendant's part to bring about harmful or offensive contact, and (iii) causation. The intent element is satisfied as long as the defendant knew with substantial certainty that the harmful or offensive contact would result. Here, the player's conduct caused a harmful contact to the fan, because the player set into motion the force that caused injury to the fan. His intentionally shooting the puck into the crowded stands is enough to establish that he knew with substantial certainty that the puck would strike a spectator.

A vintner divided his vineyard into two parcels, drawing the boundaries so that the single well that had irrigated the entire vineyard fell on the border of the two properties. The vintner then conveyed the eastern parcel to his friend by a deed that contained the following covenant: If the well located on the boundary of the eastern and western parcels continues to be used for irrigation purposes and becomes in need of repair or replacement, the grantee, his heirs, and assigns and the grantor, his heirs, and assigns each promise to pay one-half of the cost of such repair or replacement. This covenant shall run with the land. The deed from the vintner to the friend was not recorded, and the vintner did not record a copy of the deed with the records for the western parcel. The friend later sold the eastern parcel to a farmer. The farmer's deed did not contain the covenant about the well. After 15 years of use by the owners of both the eastern and western parcels, the well began to fail. The farmer took it upon himself to have the well repaired at a cost of $30,000. About two weeks later, the farmer discovered the deed from the vintner to the friend in some old files. By this time, the western parcel had passed to the vintner's son by inheritance and again to the son's daughter by inheritance from the now-deceased son. The daughter knew nothing of the covenant concerning the well. The farmer presented the daughter with the bill for the well repair with a copy of the vintner/friend deed and a note that said he expected to be reimbursed for $15,000. The daughter refuses to pay, and the farmer sues. The jurisdiction has a 10-year statute of limitations for acquiring property by adverse possession, and the following recording statute: "Any conveyance of an interest in land shall not be valid against any subsequent purchaser for value, without notice thereof, unless the conveyance is recorded." For whom is the court most likely to rule? A The daughter, because the deed from the vintner to the friend was never recorded. B The daughter, because the farmer has acquired the well by adverse possession. C The farmer, because the covenant runs with the land. D The farmer, because he is a bona fide purchaser.

(C) The farmer will most likely prevail in his suit for one-half the cost of the well repairs because the covenant runs with the land. When a covenant runs with the land, subsequent owners of the land may enforce or be burdened by the covenant. If all of the requirements for the burden to run are met, the successor in interest to the burdened estate will be bound by the arrangement as effectively as if he had himself expressly agreed to be bound. To be bound: (i) the parties must have intended that the covenant run with the land; (ii) the original parties must have been in horizontal privity; (iii) the succeeding party must be in vertical privity with the original promisor; (iv) the covenant must touch and concern the land; and (v) generally, the burdened party must have actual or constructive notice of the covenant. Here, the intent is shown by the express language of the covenant, which says that it is intended to run with the land. Even without that language, the use of the words "heirs" and "assigns" would show the intent for the covenant to run. The original parties were in horizontal privity because at the time the vintner entered into the covenant, he and the friend shared an interest in the land independent of the covenant—as grantor and grantee. The daughter is in vertical privity with the vintner because she holds the entire interest in the western parcel held by the vintner. The covenant touches and concerns the land because promises to pay money to be used in a way connected with the land are held to touch and concern the property. Because the daughter was unaware of the covenant, the required notice seems to be missing. While it is generally true that the owner of the burdened land must have notice, it should be remembered that the requirement is a function of the recording statute. (At common law, the covenant was enforceable in an action for damages regardless of notice; this was changed by the recording statutes.) However, because the daughter is a donee (an heir) and not a bona fide purchaser, she is not protected by the recording statute and thus is subject to the covenant even without notice. For that reason, (A) is wrong. (D) is wrong because the farmer's status as a bona fide purchaser has no effect on his ability to enforce the covenant. A successor in interest to the original promisee may enforce the covenant (enjoy the benefit) if there was intent and vertical privity, and the covenant touches and concerns the land. Notice is not required for the benefit to run. Thus, because the above requirements are met here, the farmer may enforce the covenant regardless of his status as a bona fide purchaser. Had the farmer taken the property as a donee, the above analysis would be the same.

A landowner's will left his ranch to a rancher, his heirs, and assigns, so long as the property was used exclusively for ranch purposes, then to the landowner's grandson. The remainder of the landowner's property passed through the residuary clause of his will to the grandson. Seven years after the landowner's death, the rancher began strip mining operations on the ranch. The grandson brought an action to quiet title to the ranch against the rancher, and the rancher counterclaimed on the same theory. Who should prevail? A The rancher, because the condition imposed on his interest under the will is void as violating the Rule Against Perpetuities. B The rancher, because the condition imposed is a restraint against alienation. C The grandson, pursuant to the residuary clause. D The grandson, because the condition imposed is valid and he takes according to the subsequent provision.

(C) The grandson prevails because the ranch passed through the residuary clause. Under the Rule Against Perpetuities, the attempt to give the grandson an executory interest is void, so (D) is incorrect. However, the courts would simply read the conveyance without the language of the executory gift, leaving a possibility of reverter in the grantor, the landowner. Thus, (A) is incorrect. Because the grandson succeeded to the landowner's interest as grantor via the residuary clause of the will, he will prevail. (B) is incorrect because there is no restraint on alienation contained in the will.

A beneficiary has filed a petition in the probate court to contest the validity of a testator's will. The beneficiary contends that when the testator executed the will eight years before, he had a severe mental illness and was incapable of forming a valid testamentary intent. In support of this contention, the beneficiary seeks to offer an affidavit prepared by the testator's former attorney, which states that she was asked to prepare a will for the testator just four months before this will was made. The attorney had refused to do so because it was her opinion that the testator seemed incoherent and paranoid. How should the judge rule on the admissibility of this affidavit? A Admissible. B Inadmissible, as being violative of the attorney-client privilege. C Inadmissible, because it is hearsay not within any exception. D Inadmissible, because it is improper opinion evidence.

(C) The judge should rule this affidavit to be inadmissible hearsay. This affidavit is clearly hearsay, and there is nothing in the facts that shows that it is admissible under any of the exceptions to this rule. Hence, (A) is wrong. (B) is wrong because the observations of the attorney would not be deemed a "communication received from the client." Also, while the attorney-client privilege generally survives the client's death, it does not apply to communications relevant to an issue between parties who are claiming through the same deceased client, such as in the probate proceedings here. (D) is incorrect because a lay person could probably testify to her opinion in this situation since it is rationally based on her own perception, it is helpful to a determination of a fact in issue, and it is not based on scientific, technical, or other specialized knowledge.

A locksmith knew that his friend had been having marital troubles. The friend had told the locksmith that he suspected his wife was having an affair with his rival. One afternoon, the friend, visibly upset, asked to borrow some of the locksmith's tools, telling him that he knew that his rival was going to meet up with his wife later that day. The locksmith gave his friend the tools, advising him not to do anything that he would regret later. The friend stated that it would be others who would have regrets. The friend went to his rival's apartment and picked the door lock with the locksmith's tools. He found his wife and rival in bed together. The friend stabbed his rival, seriously wounding him. A few minutes later the locksmith called the apartment to try to warn the rival that his friend might come over. After the friend was arrested, he agreed to plead guilty to aggravated battery and attempted voluntary manslaughter in exchange for testifying against the locksmith, who was charged as an accomplice to attempted murder. Can the locksmith be convicted of that charge? A Yes, because he recklessly disregarded a substantial risk to human life and was not provoked. B Yes, because his failed attempt to neutralize his assistance did not prevent the crime from occurring and therefore did not constitute an adequate withdrawal. C No, because he did not have the requisite intent to be liable as an accomplice. D No, because an accomplice cannot be found guilty of a more serious offense than that for which the principal has been convicted.

(C) The locksmith cannot be convicted as an accomplice because he did not have the requisite intent for attempted murder. To be convicted as an accomplice under the prevailing rule, a person must have given aid, counsel, or encouragement with the intent to aid or encourage the principal and the intent that the principal commit the substantive offense. Mere knowledge that a crime would result from the aid provided is generally insufficient for accomplice liability. Here, the locksmith did not provide the tools to the friend with the intent that he kill the rival. His knowledge that the friend might be intending harm to the rival is not sufficient to establish the intent to kill required for attempted murder. (A) is incorrect because even if the locksmith's conduct constituted reckless disregard of high risk to human life, that state of mind is not sufficient for attempted murder. Unlike murder, attempted murder is a specific intent crime and requires the intent to kill. (B) is incorrect. Although the locksmith's attempt to neutralize his assistance would not have been enough to raise the defense of withdrawal if he had incurred liability as an accomplice, here he did not have the requisite intent for accomplice liability. (D) is an incorrect statement of law; the degree of liability of a principal is irrelevant to the potential liability of an accomplice. If the locksmith had had the intent to aid his friend in killing the rival, the fact that the friend could show adequate provocation to reduce his offense to attempted voluntary manslaughter would have no effect on the locksmith's liability for attempted murder.

A defendant held up a gasoline station. During the robbery, he shot and killed a customer who attempted to apprehend him. The defendant was prosecuted for premeditated murder and convicted. Thereafter, he was indicted for armed robbery of the station. Before the trial, his attorney moved to dismiss the indictment on the ground that further proceedings were unconstitutional because of the defendant's prior conviction. Should the motion to dismiss be granted? A Yes, because once the defendant was convicted on any of the charges arising out of the robbery, the prosecution was constitutionally estopped from proceeding against the defendant on any charge stemming from the same transaction. B Yes, because the Double Jeopardy Clause prohibits a subsequent trial on what is essentially a lesser included offense. C No, because there is no constitutional requirement that all known charges against a defendant be brought in the same prosecution. D No, because estoppel does not apply when a defendant is charged with violating two different statutes.

(C) The motion to dismiss should be denied. For purposes of the Double Jeopardy Clause, two crimes do not constitute the "same offense" if each crime requires proof of an additional element that the other crime does not require, even though some of the same facts may be necessary to prove both crimes. Here, even though the same facts are involved for both crimes, the robbery charge requires proof of a taking by force but not a death, while the murder charge requires proof of a death but not of a taking of property. Thus, (C) is correct and (A) is incorrect. (B) is incorrect because armed robbery is not a lesser included offense of premeditated murder. (D) is incorrect because the prosecution would be estopped if violation of one statute constituted a lesser included offense of the other statute.

A student started a small fire in a trash can in the men's room at his university. His plan was to set off the school's fire alarms so that he could break into the computer lab and steal a laptop computer while the building was being evacuated. The student was stopped after he had set the fire and was attempting to smash in the glass on the computer lab door. The fire was quickly extinguished and no serious damage was done to the building by the fire beyond some charring on the walls in one stall of the men's room. A statute in the jurisdiction extends the crime of arson to buildings other than dwellings. Which of the following best describes the crimes of which the student could be properly convicted? A Larceny only. B Larceny and attempted arson. C Attempted larceny and arson. D Arson only.

(C) The student is guilty of the crimes of arson and attempted larceny. At common law, arson was defined as the malicious burning of the dwelling of another. The mens rea required for arson is malice, which is broader than the intent required for specific intent crimes. All that malice requires is that the defendant have acted with the intent or knowledge that the structure would burn, or with reckless disregard of an obvious risk that the structure would burn. Here, the student's intent was to set off the fire alarm so that the building would be evacuated. Nevertheless, he intended to start a fire with reckless disregard of a high risk that it would cause damage to the building. The risk or hazard is not that the building will burn down, merely that damage to the structure from a burning will occur. The "burning" required for arson does not require significant damage to the building; a charring of the combustible material is sufficient. Here, the wall of a stall in the men's room was charred. This satisfies the "burning" requirement. The common law requirement that the structure be a dwelling has been broadened by the statute in this question to include other buildings. Thus, the student's conduct satisfies all of the elements of the crime of arson. The student is also guilty of attempted larceny. Common law larceny requires a taking and carrying away of the personal property of another by trespass with intent to permanently (or for an unreasonable time) deprive the other of his interest in the property. Here, the student planned to take and carry away a laptop computer by trespass (i.e., without permission) and with the intent to permanently deprive the university of it. However, he cannot be convicted of a completed larceny because he never actually took the laptop.

A plaintiff was injured in an automobile accident caused by the defendant. The plaintiff sued the defendant for his injuries. In preparation for trial, the plaintiff's attorney hired a doctor to examine the plaintiff. At trial, the defense attorney attempts to call the doctor as a witness to testify about statements the plaintiff made in confidence to the doctor about his injuries, which the doctor then communicated to the plaintiff's attorney. The state recognizes only the common law privileges. Should this testimony be admitted? A Yes, because the plaintiff's statements are the statements of a party-opponent. B Yes, because the plaintiff waived the physician-patient privilege by placing his physical condition in issue. C No, because the plaintiff's statements are protected by the attorney-client privilege. D No, because the plaintiff's statements are protected by the physician-patient privilege.

(C) The testimony should be excluded because the attorney-client privilege applies to the examination done in preparation for trial. The communication between the doctor and the attorney's client is necessary to help the client convey his condition to the attorney. (A) is incorrect because admissions by party-opponents, while not hearsay under the Federal Rules, are still subject to potential privilege assertions. (B) is a true statement; the physician-patient privilege does not apply to any proceeding in which the condition of the patient has been put in issue by the patient. This is the case in the plaintiff's suit, so (D) is incorrect. However, (B) is incorrect because when a client is examined by a doctor at the attorney's request, the communications involved between the client and doctor (and the doctor and attorney) are not covered by the physician-patient privilege because no treatment is contemplated. Moreover, the physician-patient privilege is a statutory privilege, and this jurisdiction recognizes only the common law privileges (e.g., the attorney-client privilege).

A woman purchased a tract of land from a man by warranty deed. Unbeknownst to the woman, the man was not the actual owner of the tract. The woman built a home on the tract and moved into it. Two years later, the actual owner learned of the man's transaction with the woman and prevented the woman from entering the tract from that point forward. This led to a costly court battle. When the woman notified the man and told him that she thought it was his duty to straighten this out, he ignored her. The woman would succeed in a suit for damages against the man for breach of which of the following covenants of title? A The covenant of quiet enjoyment only. B The covenants of seisin, right to convey, quiet enjoyment, warranty, further assurances, and the covenant against encumbrances. C The covenants of seisin, right to convey, quiet enjoyment, warranty, and further assurances. D The covenants of seisin and right to convey only.

(C) The woman would succeed in a suit for damages against the man for breach of the covenants of seisin, right to convey, quiet enjoyment, warranty, and further assurances, but not on the covenant against encumbrances. A general warranty deed gives the grantee six covenants of title: the right to seisin, the right to convey, a covenant against encumbrances, the covenant of quiet enjoyment, the covenant of further assurances, and a general warranty. Under the covenants of quiet enjoyment, warranty, and further assurances, the man promised that (i) the woman would not be disturbed in her possession of the tract; (ii) he would defend the woman's title against lawful claims; and (iii) he would perform whatever acts are necessary to perfect the woman's title. Because the man neither owned the tract of land nor was acting as the actual owner's agent, he breached the covenants of seisin and right to convey at the time of the conveyance to the woman. When the actual owner prevented the woman from re-entering the property, this interfered with the woman's quiet enjoyment, and the man's refusal to "straighten this out" was a breach of the covenant of further assurances. Thus, (C) is the correct answer. There is nothing in the facts to suggest the property is encumbered; thus, the man did not breach the covenant against encumbrances, and (B) is therefore incorrect.

A woman was arrested outside of a house shortly after she had broken in and stolen some jewelry. She was indicted for larceny and later for burglary. She was tried on the larceny indictment and convicted. Thereafter, she was brought to trial on the burglary indictment. Relying on the Double Jeopardy Clause of the Constitution, the woman moves to dismiss the indictment. Should the motion be granted? A Yes, because the Double Jeopardy Clause requires that all offenses arising out of the same transaction be adjudicated in the same trial. B Yes, because the Double Jeopardy Clause allows the imposition of separate sentences for separate offenses occurring during the same criminal episode only if the offenses are tried together. C No, because larceny and burglary are offenses that may constitutionally be tried and punished separately, even if they arise out of the same transaction, because each requires proof of a fact that the other does not. D No, because the only protection double jeopardy affords to a defendant charged with multiple counts is under the doctrine of collateral estoppel.

(C) The woman's motion should be denied because the Double Jeopardy Clause does not prohibit the second prosecution. The Double Jeopardy Clause of the Fifth Amendment provides criminal defendants with the right to be free of double jeopardy for the same offense. However, two crimes do not constitute the same offense if each crime requires proof of an additional element that the other crime does not require, even though some of the same facts may be necessary to prove both crimes. Here, larceny requires a taking and carrying away of the property of another, which burglary does not require, and burglary requires a breaking and entry, which larceny does not require. Hence, they are distinct offenses for purposes of the Double Jeopardy Clause. (A) and (B) are incorrect because the Supreme Court does not use a "same transaction" or "same episode" test suggested by these answer choices; instead, the Blockburger test is used regardless of whether the two offenses were tried together at a single trial or at separate trials. (D) is incorrect because while double jeopardy also protects against inconsistent factual determinations at a subsequent trial, it protects against multiple prosecutions as well, as long as the crime is the "same offense."

A landowner possessed a 40-acre tract of land. He had inherited 30 acres and had possessed the other 10 acres for longer than the statutory period necessary to acquire title by adverse possession from a rancher. The landowner entered into a land sale contract promising to convey the 40 acres to a developer. The contract provided that the landowner would convey marketable title. The developer paid the landowner the purchase price and accepted a deed from him. The developer promptly recorded the deed. The rancher, having learned of the sale, brought a successful action against the developer to quiet title. The developer realized for the first time that there were no covenants for title in his deed. The developer brings an action against the landowner. What is the most likely outcome of the suit? A The developer will win, because the landowner breached the terms of the contract. B The developer will win, because the landowner misrepresented the size of the tract. C The landowner will win, because the terms of the deed control his liability. D The landowner will win, because the developer was negligent in not checking the covenants of title at the time of closing.

(C) The landowner will win because the terms of the deed, not of the contract, control his liability. There is an implied covenant in every land sale contract that at closing the seller will provide the buyer with a title that is "marketable." Marketable title is title reasonably free from doubt, i.e., title that a reasonably prudent buyer would be willing to accept. It need not be a "perfect" title, but the title must be free from questions that might present an unreasonable risk of litigation. Generally, this means an unencumbered fee simple with good record title. Generally, a title acquired by adverse possession is not considered marketable because the purchaser might be later forced to defend in court the facts that gave rise to the adverse possession against the record owner. Here, the marketability requirement did not have to be implied, it was an express term of the contract. Under the doctrine of merger, the contract merges into the deed, and the terms of the contract are meaningless. Even though the contract specified a "good and marketable title," it is the deed that controls, and the deed contained no covenants of title. A deed does not incorporate the title terms of a contract. Thus, (A) is wrong. (B) is wrong; it is not supported by the facts. (D) is wrong because the developer's negligence is irrelevant.

A retailer entered into an oral contract with an office supply wholesaler to buy 100 file boxes for an upcoming back to school sale at the retailer's store. The wholesaler agreed to deliver the file boxes in two weeks at a cost of $4 per file box. A week later, the retailer phoned the wholesaler and asked if she could increase her order to 200 file boxes. The wholesaler agreed. The wholesaler delivered the 200 file boxes as promised, but the retailer accepted only 150 upon discovering that she lacked storage space for all 200. May the wholesaler recover damages with respect to the 50 file boxes that were not accepted? (A) Yes, because the retailer accepted $600 worth of file boxes. (B) Yes, because the modification was for less than $500. (C) No, because the contract as modified was for $800. (D) No, because the wholesaler is a merchant with respect to file boxes.

(C) The wholesaler may not recover damages. Under the UCC Statute of Frauds, a contract for the sale of goods for $500 or more is unenforceable unless evidenced by a writing signed by the party sought to be held liable. The original contract was for $400 and, thus, was not within the Statute. Whether a modification must be in writing to be enforceable depends on whether the entire contract price as modified is within the Statute. Here, the retailer and the wholesaler modified their original contract to 200 file boxes, bringing the total price to $800. Thus, the modification was unenforceable under the Statute of Frauds, and the wholesaler cannot collect damages with respect to the 50 unaccepted file boxes. (B) is incorrect because, as noted above, when determining whether a contract for the sale of goods is enforceable, we look at the whole contract price as modified; the price of the modification itself does not matter. (D) is incorrect because the fact that the wholesaler is a merchant with respect to the goods being sold (file boxes) has no bearing on the enforceability of the contract here.

A man from a foreign country obtained a doctorate in political science from a state university and applied to teach there. The man was denied employment at the university under a state law requiring all teachers within the state to be United States citizens. Is the state's citizenship requirement constitutional as it applies to the man? A Yes, because states have the right to set minimal standards for state employees under the Tenth Amendment. B Yes, because a university political science teacher would exert a great deal of influence over the attitudes of students toward government, the political process, and citizenship. C No, because the citizenship requirement is not rationally related to a legitimate state interest. D No, because the citizenship requirement is not necessary to achieve a compelling state interest.

(D) A state generally may not discriminate against aliens absent a compelling state interest, and no compelling interest is served by prohibiting aliens from teaching at a state university. (A) is incorrect. The Tenth Amendment reserves to the states power not granted to the federal government. The Constitution vests the power to regulate aliens in Congress, and thus the states do not have power to control aliens under the Tenth Amendment. (B) is incorrect because it states the standard that the Supreme Court has applied to primary and secondary school teachers. The Supreme Court has upheld state statutes prohibiting aliens from teaching primary or secondary school on the rationale that teachers at the elementary and high school level have a great deal of influence over the attitudes of young students toward government, the political process, and citizenship. It is doubtful that the Court would extend this rationale to university teachers. (C) is incorrect because it states the wrong standard. If state discrimination against aliens relates to participation of aliens in the functioning of state government, the rational basis test applies. Merely teaching political science at a state university is not equivalent to participating in the political process.

An antique lover spotted a beautiful Early American bedroom ensemble at her favorite antique store. The ensemble included a bed, a mirror, and two dressers. Over a period of several weeks, the shop owner and the antique lover negotiated over a price, but they were unable to come to an agreement. On April 3, the shop owner and the antique lover signed a statement whereby the shop owner offered to sell to the antique lover an Early American bedroom ensemble, recorded as items 20465, 20466, 20467, and 20468 in the shop's registry, if the parties agree upon a price on or before April 12. On April 6, the shop owner sent a letter to the antique lover, telling her that she could have the bedroom ensemble for $22,000. Also on April 6, the antique lover sent a letter to the shop owner telling him that she was willing to pay him $22,000 for the bedroom ensemble. Both parties received their letters on April 7. Without assuming any additional facts, which of the following statements is most correct as of April 8? A The shop owner and the antique lover had a valid contract from the moment the letters of April 6 were mailed. B A contract exists between the shop owner and the antique lover, because the shop owner, a merchant, sent the antique lover an offer in writing. C A contract exists between the shop owner and the antique lover, because the crossing offers were identical and received before April 12. D No contract exists between the shop owner and the antique lover, because of a lack of mutual assent.

(D) Although the crossing offers as to price were identical, there is no requisite mutual assent absent an acceptance. If offers stating precisely the same terms cross in the mail, they do not give rise to a contract despite the apparent meeting of the minds. An offer cannot be accepted if there is no knowledge of it. Here, the shop owner and the antique lover each sent offers setting the price of the ensemble at $22,000. Despite the fact that these offers were identical, there is no mutual assent without at least one of the parties manifesting acceptance of the terms of the offer, and communicating that acceptance to the other. We are told that this has not yet happened even though the shop owner and the antique lover both have received the letters. Consequently, although there is an apparent meeting of the minds as to price, there has not been a sufficient objective manifestation of this agreement as to denote a mutual assent. (C) fails to account for the principle discussed above, that identical crossing offers do not give rise to a contract. Despite their receipt of identical offers before April 12, there is no agreement between the parties. (A) is incorrect because it misstates the mailbox rule. Acceptance by mail or similar means creates a contract at the moment of posting, properly addressed and stamped, unless the offer stipulates that acceptance is not effective until received, or unless an option contract is involved. This rule does not operate to create a contract from the moment an offer is mailed (or in this case, two identical offers are mailed). Thus, (A) is incorrect. Regarding (B), the fact that a merchant sends an offer in writing is significant because it will limit the offeror's power to revoke if it gives assurances that it will be held open for a stated time. Here, the written offer by the shop owner is irrevocable at least until April 12, but the issue in the question is whether it has been accepted rather than whether it has been revoked.

At a waterfront bar, a college student sought to provoke a fight with a merchant seaman by making insulting remarks. Eventually the seaman had had enough and threw a punch that connected to the student's jaw and sent him sprawling to the floor. The seaman then told the student that he wanted no further trouble. Getting up off the floor, the student pulled a knife out of his pocket and charged at the seaman. Three other students were standing between the seaman and the exit door. The seaman tried to dodge, but was cut on the forearm by the student's knife. The seaman immediately drew a gun and shot the student, killing him. The seaman was charged with murder. Which of the following points raised in the seaman's defense will not be helpful for his defense? A The student had no reason to fear serious bodily injury when he drew the knife. B The student's drawing of the knife constituted an escalation of the fight. C Three college students were standing between the seaman and the door, so there was no clear route of retreat. D The student's comments were motivated by a desire to provoke the seaman.

(D) Even though the student's words may have been intended to provoke the seaman, this fact alone would not justify the seaman's use of deadly force. A person may use deadly force in self-defense if he: (i) is without fault; (ii) is confronted with unlawful force; and (iii) reasonably believes that he is threatened with imminent death or great bodily harm. Generally, one who is at fault for starting a confrontation has no right to use force in his own defense during that confrontation. However, if the victim of the initial aggression suddenly escalates a relatively minor fight into one involving deadly force and does not give the aggressor a chance to withdraw or retreat, the aggressor may use deadly force in his own defense. Here, although the student instigated the hostile situation by repeatedly insulting the seaman, the seaman's throwing of a punch probably calls for his being characterized as the aggressor. The student, as the victim of the initial aggression, escalated matters by using a knife, especially because the seaman had said that he wanted no further trouble. This escalation (which is the point stated in choice (B)) entitled the seaman to employ deadly force in his own defense against the imminent threat of death or great bodily harm posed by the student's use of the knife.

At a victory party after a hard-fought election, the campaign director consumed several drinks. A campaign worker who had also been drinking took the director to her hotel room for a nightcap. They later had intercourse. The worker filed a complaint with the police, claiming that the director had intercourse with her against her will, and the director was charged with rape. Which of the following is most likely to be admitted in the director's defense? A The worker has a reputation in the community as being sexually promiscuous. B Since the incident occurred, the worker has had sexual intercourse with two other campaign workers. C Two years ago during the candidate's previous campaign, the worker maintained a sexual relationship with the former campaign director. D The director and the worker had had consensual sex on two prior occasions.

(D) Evidence of prior consensual sexual relations between the director and the worker is most likely admissible. Although Federal Rule 412 generally excludes evidence of an alleged victim's sexual behavior, evidence of specific instances of sexual conduct between the alleged victim and the accused may be admitted to show consent. Thus, if the director raises consent as a defense to the rape charge, evidence of his previous consensual sexual encounters with the worker is admissible. (A) is incorrect because it presents evidence that Federal Rule 412 specifically intends to exclude; i.e., evidence of the alleged victim's sexual behavior. The Federal Rules also contain an exception for specific instances of the alleged victim's sexual conduct tending to show that someone other than the accused was the source of semen, injury, or other physical evidence. (B) does not fit this exception because the worker had intercourse with two other workers after the incident with the director occurred. Likewise, (C) is not likely to fit this exception as the worker's relationship with the previous campaign director occurred two years ago, and is therefore probably not helpful in explaining the presence of physical evidence.

A town with a population of 30,000 merged with a city of 60,000. To protect voting rights of the citizens of the former town, a proposal was made that for a period of 20 years, beginning at the date of the merger, the city council of the merged city would consist of six persons. Each formerly separate municipality would be divided into three council districts. Each district from the former town would have approximately 10,000 residents, and each district from the former city would have 20,000 residents. A mayor would be elected at large. Before this proposal was placed on the ballot, the state attorney general issued an advisory opinion stating that the proposal was not in violation of any state statutory or constitutional provision. The proposal was placed on the ballot and was carried by large majorities in both the town and the city, and the districts were carved out. Three taxpayers filed suit to enjoin the holding of an election with council districts of such disparate proportions. The suit reached the state supreme court, which ruled that the governmental formula was constitutional under both the state and United States Constitutions. The plaintiffs wish to take the case to the United States Supreme Court. How should the Supreme Court proceed? A Rely on the attorney general's opinion and not hear the case on its merits. B Not hear the case, because it was decided below on an independent state ground. C Not hear the case, but remand it to federal district court. D Hear the federal issues involved, but decline to rule on state issues.

(D) The Supreme Court may grant certiorari to review a case from the highest court in a state that can render an opinion on the matter if a state statute's validity is called into question under the federal Constitution. [28 U.S.C. �1257] The Court may decide the federal issues, but cannot rule on the state law issues. (A) is incorrect for several reasons: (i) the attorney general evaluated only the proposal's validity under the state constitution; and (ii) even if her opinion had addressed the proposal's federal constitutional validity, the Supreme Court is not bound by advisory opinions of state attorneys general. (B) is incorrect even though the state supreme court may have had an independent state ground for finding the law constitutional under its state constitution. The Supreme Court will refuse to hear the case only if the state ground is adequate by itself to support the decision as well as independent, so that the Court's review of the federal ground for the decision would have no effect on the outcome of the case (such as if the state court had found the law invalid under both the state and federal Constitutions). Here, the Supreme Court's review of the state court opinion on the law's federal constitutional status may have an outcome on the case regardless of the state court's decision on the state constitutional issue; the Court therefore will hear the federal issues involved. (C) is incorrect because 28 U.S.C. section 1257 provides that appellate review of a matter from a state's highest court is to the Supreme Court by petition for a writ of certiorari, rather than to a federal district court.

A three-year-old girl attending nursery school punched a boy in the face because he was teasing her about wearing glasses. The blow knocked out the boy's newly acquired front teeth. If the boy's parents sue the girl's parents for the injury, what is their best defense? A The boy was the initial aggressor. B The girl is too young to be responsible for her actions. C A parent cannot be liable for damages due to the child's conduct. D The parents were unaware of any potentially violent behavior by the girl.

(D) The best defense of the girl's parents is that they were unaware of any potentially violent behavior by the girl. At common law, parents are not vicariously liable for the torts of their child. (Statutes in most states allow for limited liability for intentional torts, but there is no indication of such a statute here.) Parents can be liable, however, for their own negligence, i.e., in not exercising due care under the circumstances. Thus, if the parents know their child may be violent, they could be negligent if they do not take precautions to prevent that behavior or injury from that behavior. However, if the parents have no reason to know their child could be violent, they have no duty to protect against such behavior. Here, if the girl had never done anything like this before, and her parents had no idea that she would be violent, they were not negligent in allowing her to attend nursery school. (A) is wrong because, although the boy's teasing may have provoked the girl, he did not initiate the violence. He did nothing to allow the girl a right of self-defense, and so his actions would not provide the girl's parents with a good defense. (B) is wrong because there is no general tort immunity for children. As long as the child is old enough to intend the act, she can be held liable. Here it seems that the girl intended to cause a battery. She either intended or knew with substantial certainty that swinging her fist would strike the boy in the face, i.e., would cause a harmful or offensive contact. Thus, this choice does not present the best defense for the defendants. (C) is wrong because parents can be liable for damages due to their child's conduct. As explained above, although the parents are not vicariously liable at common law, they can be liable based on their own negligence (e.g., for negligent supervision)

The defendant and an accomplice were on trial together for burglary. Both had given confessions implicating themselves and their accomplice. At trial, the defendant maintained that his confession had been obtained through improper coercion by the police. For the purpose of countering the claim of coercion, the prosecution seeks to place the accomplice's confession into evidence. After objection by the defendant's counsel, the judge agrees to issue a limiting instruction to the jury that the confession is to be considered only with regard to the question of whether the defendant's confession was coerced. May the accomplice's confession be admitted under that condition? A No, because admission of the confession violates the defendant's right of confrontation. B No, unless the accomplice takes the stand and subjects himself to cross-examination regarding the confession. C Yes, as long as all portions of the confession referring to the defendant can be eliminated. D Yes, because the judge's instruction limits consideration of the confession only to the issue of coercion.

(D) The confession is admissible with the judge's limiting instruction. Where two persons are tried together and one has given a confession implicating the other, the general rule is that the Sixth Amendment right to confront adverse witnesses prohibits the use of such a statement. This problem arises because of the inability of the nonconfessing defendant to compel the confessing co-defendant to take the stand for cross-examination at their joint trial. As exceptions to the general rule, the statement may be admitted if: (i) all portions of the statement referring to the other defendant can be eliminated (so that there is no indication of that defendant's involvement); (ii) the confessing defendant takes the stand and subjects himself to cross-examination with respect to the truth or falsity of what the statement asserts; or (iii) the confession of the nontestifying co-defendant is being used to rebut the defendant's claim that his confession was obtained coercively, in which case the jury must be instructed as to the purpose of the admission. (B) and (C) are incorrect because neither of those conditions is necessary for the confession to be admitted as long as the judge issues a limiting instruction, as discussed above.

A homeowner entered into a written agreement with a contractor whereby the contractor agreed to completely remodel the homeowner's bathroom "to her specifications" at a cost of $10,000. The homeowner's specifications were highly detailed and required custom-made fixtures that would not be usable in other bathroom remodeling jobs. The contractor ordered the custom-made fixtures and paid $4,000 for them when they were delivered to his place of business. Figuring up the cost of the fixtures and labor, the contractor estimated that he would make a total profit of $2,000 on the job after payment for materials and workers. Before the contractor began work on the project, but after he had paid for the fixtures, the homeowner told the contractor that she had had a change of heart and would probably be selling the house the following year, and so would not need a custom bathroom. The contractor made no attempt to sell the fixtures to another contractor and filed suit against the homeowner for damages. What is the contractor likely to recover? A Nothing, because he failed to mitigate damages. B His expectation damages of $2,000. C $4,000, the cost of materials as restitution. D $2,000 as expectation damages, plus $4,000 in reliance damages.

(D) The contractor can recover $2,000 as lost profits plus the $4,000 in costs he incurred before the homeowner breached the contract. The purpose of a damages remedy is to give compensation for the breach; i.e., to put the nonbreaching party where he would have been had the promise been performed. In most cases, the plaintiff's standard measure of damages will be based solely on an "expectation" measure, i.e., sufficient damages for him to buy a substitute performance. A reliance measure of damages, on the other hand, awards the plaintiff the cost of his performance, i.e., his expenditures in performing his duties under the contract. In certain situations, an award of compensatory damages will contain both an expectation and a reliance component. In a construction contract, if the owner breaches the contract after the builder has already begun his performance, the builder will be entitled to any profit he would have derived from the contract plus any costs he has incurred to date. The contractor has begun performance by ordering and purchasing the custom-made fixtures at a cost of $4,000. Because they are usable only for the homeowner's purposes, their cost, which is treated just like any other expenditure of labor and material in a partially completed construction contract, can be recovered as reliance damages. The other element of his recovery is the $2,000 profit that he would have derived from the contract—his expectation damages. His total recovery will therefore be $6,000.

A wholesale seller sent a fax to a manufacturer with whom he had done business before: "Send 500 'Madewell' chairs at your usual price." The manufacturer responded, also by fax, that the line was being discontinued, but he would ship his last 500 chairs at $75 per chair, his usual price. The manufacturer immediately began the paperwork for processing the order and started preparing and packing the chairs for shipment. Before the chairs could be delivered, the wholesaler canceled his order, noting that the price was too high. The day after receiving the wholesaler's cancellation, the manufacturer sold the chairs to another buyer for $75 each. If the manufacturer sues the wholesaler for damages, how much should he recover? A Nothing, because this was a contract between merchants and the wholesaler canceled within a reasonable time. B Nothing, because the manufacturer was able to cover by selling the chairs at the same price he would have received from the wholesaler. C $37,500, the full contract price, because the wholesaler breached the contract and $75 per chair was a fair price. D The incidental costs of preparing the paperwork and other office costs connected with preparing and packing the chairs for shipment to the wholesaler.

(D) The manufacturer will recover only his incidental damages, i.e., the costs of preparing to ship the chairs. An offer calling for shipment of goods, such as the offer here, may be accepted by prompt shipment with notice or by a promise to ship. Acceptance forms a contract. Here, the manufacturer accepted the wholesaler's offer by promising to ship, and a contract was formed. The wholesaler breached the contract by canceling his order. When a buyer breaches by repudiating his offer, as the wholesaler did here, the seller has a right to recover his incidental damages plus either the difference between the contract price and the market price or the difference between the contract price and the resale price of the goods, reduced in either case by any expenses saved as a result of the breach. Here, the manufacturer made what he would have if the sale with the wholesaler had gone through—there was no difference between the contract price and the resale price. Thus, the manufacturer would be limited to his incidental damages. Lost profits would not be available because the chairs were the last ones that the manufacturer had and would have because the line was being discontinued. Therefore, the manufacturer would not have been able to sell another set of 500 chairs to another potential buyer.

The owner of an apartment building contracted with a painter to paint the porches of the apartments for $5,000. The contract was specifically made subject to the owner's good faith approval of the work. The painter finished painting the porches. The owner inspected the porches and believed in good faith that the painter had done a bad job. The painter demanded payment, but the owner told him that the paint job was poor and refused to pay. The painter pleaded that he was desperately in need of money. The owner told the painter that she would pay him $4,500, provided he repainted the porches. The painter reluctantly agreed, and the owner gave the painter a check in the amount of $4,500. The painter went to his bank, indorsed the check "under protest" and signed his name, then deposited the check in his account. He never returned to repaint the porches. The painter sues the owner for $500, which he believes is still owed to him on his contract to paint the porches. Will he prevail? A Yes, because he indorsed the check "under protest." B Yes, but only if he repaints the porches. C Yes, because he performed the contract by painting the porches the first time. D No, even if he repaints the porches.

(D) The painter will be unable to recover the $500 because he did not satisfy the condition precedent to payment under the contract. A party does not have a duty to perform if a condition precedent to that performance has not been met. Here, the parties made the owner's satisfaction with the painter's paint job a condition precedent to the owner's duty to pay the $5,000. Because the owner was not satisfied with the paint job, her duty to pay the painter never arose. The fact that the owner offered to give the painter $4,500 if he repainted the porches has no effect on this analysis, because the offer constituted a new contract, the owner having been excused from the old one. (A) is wrong because it does not matter whether the painter indorsed under protest. The indorsement will not change the result here because the new contract did not seek to discharge any contractual duty—the owner was already excused from her duties because the condition precedent was never met. (B) is wrong because the old contract, which provided for payment of $5,000, is considered to be at an end. Under the terms of the new contract, the painter is entitled to only $4,500, provided he repaints the porches. (C) is wrong because the condition precedent to the payment of $5,000, the owner's satisfaction, was not met. The courts have held such conditions to be valid—not illusory promises—because of the promisor's duty to exercise good faith in assessing satisfaction. Here, the facts state that the owner believed in good faith that the painter had done a bad job; thus, the painter is not entitled to payment under the original contract. Note that since he has not performed under the new contract, he is in breach and not entitled to the $4,500 already paid.

A plaintiff filed a negligence action against a defendant in federal district court after a two-car accident. The plaintiff's attorney created a list of everyone he could identify who observed the accident or otherwise had information relevant to the accident. The list includes one eyewitness whom the plaintiff's attorney was able to identify only through the expenditure of several thousand dollars in investigation costs. The defendant served the following interrogatory on the plaintiff: "Please state the name of each person of whom you are aware who may know or have information relevant to this action." Must the plaintiff provide the defendant with the names of all of the people on the plaintiff's attorney's list? A No, because the names on the list are protected from discovery under the work product doctrine. B No as to the name of the eyewitness found through the plaintiff's investigation efforts, but yes as to the other names on the list. C Yes, because, while the names are subject to qualified immunity from discovery under the work product doctrine, the defendant will be able to show sufficient need to obtain a court order requiring the names' disclosure. D Yes, because the names are relevant to the claims and defenses of the parties, and they do not constitute work product.

(D) The plaintiff must provide the defendant with the names of the people on the plaintiff's attorney's list. In general, discovery may be had of any nonprivileged matter that is relevant to any party's claim or defense and proportional to the needs of the case, including the identity of individuals with knowledge of any discoverable matter. Because the Federal Rules of Civil Procedure state that the identities of people with knowledge of discoverable matters are discoverable, the plaintiff's attorney's list is not considered work product. Thus, (A) and (C) are incorrect. (B) is incorrect because the identity of the eyewitness is discoverable regardless of the extent of the plaintiff's investigation efforts.

A landlord brought suit against a tenant in federal court for overdue rent payments on a commercial lease. The landlord sought to recover on the six rent installments that were past due and unpaid at the time of the suit. The landlord won the case, and judgment was entered in her favor. The lease has an acceleration clause that states that all future rent payments become due if the tenant falls behind three months or more. The landlord now files suit against the tenant for the remaining rent payments. The tenant moves to dismiss, asserting that the landlord's claim is barred by claim preclusion (res judicata) principles. Should the tenant's motion to dismiss be granted? A No, because a landlord may choose when to sue on an acceleration clause. B No, because the two suits do not involve the same cause of action. C Yes, because a landlord may not sue on later installments of an installment contract. D Yes, because the two suits arose out of the same transaction or occurrence.

(D) The plaintiff's suit is unlikely to be successful because the two suits arise out of the same transaction or occurrence. Claim preclusion (res judicata) requires that (i) a valid, final judgment on the merits was entered in the first case; (ii) the cases were brought by the same claimant against the same defendant; and (iii) the same cause of action is involved in the later lawsuit. Generally, a claimant is required to assert all causes of action arising out of the same transaction or occurrence that is the subject matter of the claim. In the situation of installment payments, the claimant is required to sue on all installments due at the time of the suit. If there is an acceleration clause, the claimant must sue for all installments. Here, the plaintiff should have sued for all installments in the first lawsuit. Because she did not, her second suit is barred by claim preclusion. (A) is incorrect because a plaintiff may not choose when to sue on an acceleration clause. If there is a nonoptional acceleration clause in the contract, the plaintiff must sue for all installments in her original lawsuit. (B) is incorrect because the two suits do involve the same cause of action. They arise out of the same transaction or occurrence, i.e., the lease. (C) is incorrect because, although usually a plaintiff may only sue on the installments due at the time of the suit, she must sue for all installments if there is an acceleration clause.

A landowner validly conveyed a small office building to the Green Party "as long as they use it for operating quarters until the next presidential election." After the next presidential election, which was in three years, the building would go to a private organization that monitors and prepares comprehensive listings of gas prices throughout the country. A year after the conveyance, the landowner died, validly devising all of her property to her son. Although this jurisdiction is a common law jurisdiction with respect to all real property considerations, the state's probate laws provide that future interests or estates in real property may be passed by will or descent in the same manner as present or possessory interests. Last week, the Green Party and the gas monitoring organization joined together to sell the office building in fee simple absolute to a developer. The son filed suit to prevent the sale of the property to the developer. In this action, who should prevail? A The Green Party and the gas monitoring organization, because together they own a fee simple absolute in the building. B The Green Party and the gas monitoring organization, because the attempted restrictions on the use of the property violate the Rule Against Perpetuities. C The Green Party and the gas monitoring organization, because the deed restriction was an unlawful restraint on alienation. D The son, because he did not sign the contract of sale.

(D) The son may enjoin the sale because he has an interest in the property. A fee simple determinable is an estate that automatically terminates on the happening of a stated event. The Green Party's interest in the office building is a fee simple determinable because it lasts as long as the Party is using the building for operating quarters. However, the grant does not provide for the contingency of the Green Party ceasing to use the building as operating quarters before the next presidential election. This gap would be filled by a possibility of reverter retained by the landowner. Because the landowner passed that interest to her son in her will, there can be no contract to sell the property without his signature. Note: Although the gas monitoring organization appears to have an indefeasibly vested remainder (i.e., it is created in an ascertained company, is certain to become possessory, and is not subject to being defeated, divested, or diminished in size), its interest is not capable of taking on the natural termination of the preceding estate and so is characterized as a springing executory interest. (A) is wrong because the son also has an interest in the land. (B) is wrong because the interest in the office building will pass to the gas monitoring organization, if at all, within 21 years. (C) is wrong because the Green Party is not prohibited from transferring any interest; it could pass a defeasible fee.

Congress created a seven-member safety commission to investigate and make recommendations to Congress for new fireworks safety laws, to make further rules for establishing safety and performance standards, and to prosecute violations of these safety standards. The chairman of the commission was appointed by the President. Three members were selected by the Speaker of the House of Representatives, and three members were selected by the President pro tempore of the Senate. An organization with proper standing seeks to enjoin enforcement of the commission's rules. Which of the following presents the strongest constitutional argument that the organization can make against the validity of the commission? A The commerce power does not extend to activities occurring solely within a state. B Legislative power may not be delegated by Congress to an agency in the absence of clear guidelines. C The organization is denied due process of law because it is not represented on the commission. D The commission lacks authority to enforce its standards because some of its members were appointed by Congress.

(D) The strongest argument is that the commission lacks authority to enforce its standards. The Appointments Clause of the Constitution permits Congress to vest appointments of inferior officers only in the President, the courts, or the heads of departments. Enforcement is an executive act; therefore, Congress cannot appoint members of a commission that exercises enforcement powers. In these facts, the safety commission consists of some members appointed by Congress. This commission therefore violates the Appointments Clause of the Constitution because it has enforcement powers, since it can prosecute violations. Legislative power can be delegated even under vague guidelines without creating an unconstitutional delegation. Thus, (B) is incorrect. (A) is incorrect because the commerce power extends to many activities occurring solely within a particular state, if, taken as a whole, the activities in question have a substantial economic effect on interstate commerce. There is no constitutional provision requiring that a party or group to be regulated be represented on the commission that regulates it. (C) is therefore incorrect.

Shortly after a professor at a state university completed her second year of teaching, she was informed that her contract was not being renewed for the following year. By state law, a professor does not acquire tenure until after she has completed three consecutive years of teaching. Before acquiring tenure, state law does not require either a statement of reasons or a hearing when a professor's contract is not renewed, and the university administration refused to give either to the professor. Which of the following, if established, sets forth the strongest constitutional argument that the professor could make to compel the university to furnish her a statement of reasons for the failure to rehire her and an opportunity for the hearing? A She purchased a home in anticipation of renewal of her contract, because most professors who had taught two years were rehired. B She had been voted the most popular professor on campus in each of her first two years of teaching. C She was the only teacher at the university whose contract was not renewed that year. D There is evidence to indicate that the decision not to rehire the professor was not based on her ability to teach.

(D) The strongest argument the professor could make is that the decision was not based on her ability to teach. The professor is an at-will employee, and under most circumstances may be discharged "for any reason or no reason at all." Thus, normally, evidence regarding the motives for dismissal is irrelevant. The question here, however, is what the strongest argument is that the professor could make, and (D) creates at least an inference that an impermissible motive might be present (gender, free speech, etc.). (A) is a weaker answer because the professor has no property interest in continued employment; a mere expectation of continued employment is not enough, even when coupled with reliance (her buying a house). There must be a legitimate claim or entitlement—created by a contract or clear policy—that employment can be terminated only for cause. The bases alleged in (B) are arguably irrelevant; the professor's popularity may or may not have anything to do with her ability, and even if it does, she remains an at-will employee. (C) might under some circumstances offer an argument, but there could be any number of valid explanations for keeping others and letting a particular professor go, including budget constraints, subject needs, etc. (D) is, accordingly, the strongest of the possibilities.

A man and a woman were in a two-car traffic accident. Immediately after the accident, the man was treated by a physician at a nearby hospital's emergency room. The man later filed a negligence action against the woman in federal district court, seeking compensatory damages. The woman now seeks discovery regarding the emergency room physician's observations, opinions, and treatment of the man. Is the woman entitled to discovery regarding that information? A No, because such discovery is not relevant to the claim or defense of a party. B No, because the woman is not entitled to obtain discovery from persons who are not parties to the action. C Yes as to the physician's observation and treatment, but the physician's opinions are discoverable only if the man intends to call the physician as an expert witness at trial. D Yes, because the physician observed and treated the man and developed opinions about the man's injuries for purposes other than litigation or trial.

(D) The woman is entitled to discovery regarding the physician's observations, opinions, and treatment of the man because the physician developed opinions about the man's injuries for purposes other than litigation or trial. (A) is incorrect because the physician's observations, opinions, and treatment of the man are relevant to the man's claims and the driver's defenses. (B) is incorrect because parties may obtain discovery from individuals with knowledge of any discoverable matter, not just parties to the action. (C) is incorrect because it is not applicable to the situation here, where the physician was not retained in anticipation of litigation or trial.

A state law provides that all persons who have been residents of the state for more than three years shall be entitled to free tuition at the state's main university. It further provides that persons who have resided in the state for three years or less shall pay the nonresident tuition rate, which is significantly higher. A student at the state's university who had been a state resident for less than three years filed a class action in federal court on behalf of himself and other similarly situated university students, seeking a declaration that the state statute is unconstitutional. When the case came to trial, the student had been a resident of the state for more than three years and was no longer required to pay tuition. By that time, a number of amicus curiae briefs had been filed in the case, some supporting and some opposing the student's position. Nevertheless, the state moved to dismiss the case as moot. Should the state's motion to dismiss be granted? A Yes, because the student is now a three-year resident. B Yes, because the student lacks standing. C No, because amicus curiae briefs have been filed. D No, because there is a live controversy.

(D) There is a live controversy and the case is not moot. A federal court will not hear a case unless there is a real, live controversy at all stages of the proceeding, not merely when the case is filed. Because the student is no longer required to pay nonresident tuition, there is arguably no controversy and the case may seem moot. However, a class action is not moot, and the class representative may continue to pursue it—even if the representative's own controversy has become moot—because the claims of others in the class are still viable. Here, the student filed his suit as a class action for university students with less than three years' residency; undoubtedly some of those students will still have a real controversy at this time. Thus, the case is not moot. (A) is wrong although it states a true fact. (A) implies that the case should be dismissed because the student's claim is moot. As explained above, this is a class action and other members of the class have a viable case. Thus, even though the named student's case by itself would be moot, he may continue the case as a representative of the class action.

A buyer entered into a contract with a seller to purchase the seller's farm. The contract of sale referred to the farm as containing 250 acres. The agreed-on price was $1 million. Before the date on which escrow was to close, the buyer learned from a surveyor he had hired that the farm actually contained 248 acres. On the date the sale was to close, the buyer instructed the escrow agent to release all but $8,000 of the purchase money because he was not getting what he bargained for. The seller refused to proceed with the sale. The buyer brings an action for specific performance and also seeks an $8,000 reduction of the agreed-upon contract price. What will be the probable outcome of the litigation? A The seller will win, because the buyer refused to tender the contract price when the seller tendered substantially what the contract called for her to perform. B The seller will win, because both parties had seen the farm before the contract was formed. C The buyer will win, because he is not receiving what he bargained for under the contract. D The buyer will win, if the court finds that the $8,000 reduction in price is a fair reflection of the title defect.

(D) This answer states the traditional rule where the amount of land in a land sale contract is less than as agreed. When a buyer has a remedy of specific performance in a land sale contract, a court of equity will order a seller to convey the title if the buyer tenders the purchase price. If the seller cannot provide marketable title under the terms of the contract, but the buyer wishes to proceed with the transaction, the buyer can usually get specific performance with an abatement of the purchase price in an amount reflecting the title defect. A defect as to the quantity of land conveyed is usually corrected by a pro rata abatement of the price. (D) states the factors that a court of equity will look for when deciding whether to grant specific performance with abatement.A) is incorrect because the parties' contract did not merely refer to the farm as a named parcel of land; it recited that it contained 250 acres. Based on this recital, a court could readily conclude that the difference of two acres is a material change in the terms of the contract and that the seller's tender of 248 acres was not substantial performance. (B) is incorrect because viewing the property did not put the buyer on notice as to the discrepancy; the buyer is not required to visually calculate the amount of acreage a parcel of land contains.

An environmental group, wishing to stop the issuance by a federal agency of a mining permit to a coal company, commences an action in federal court against the federal agency, seeking, among other things, a permanent injunction barring the issuance of the permit to the coal company. If the coal company seeks to join the litigation as a matter of right, must the federal court grant the motion? A No, because intervention of outside parties is a matter within the sole discretion of the judge. B No, unless the coal company has been given an unconditional right to intervene by a federal statute. C Yes, because the coal company has an interest in getting the mining permit. D Yes, unless the court concludes that the coal company's interest in getting the permit is adequately protected by the federal agency.

(D) Under Rule 24, a nonparty may intervene in an action as a matter of right in two situations. First, a nonparty may intervene when it has an unconditional right to do so by a federal statute. Second, a nonparty may intervene if (i) it has an interest in the property or transaction the is the subject matter of the action; (ii) the disposition or resolution of the action may as a practical matter impair the nonparty's ability to protect its interest; and (iii) the nonparty's interest is not adequately protected by an existing party in the action. Here, although no federal statute gives the coal company the right to intervene, it does have an interest at stake in the action—its interest in getting the mining permit issued to it—and its ability to obtain the permit will as a practical matter be impaired if the environmental group succeeds in getting an injunction against its issuance. Thus, the coal company should be allowed to intervene unless the court concludes that the federal agency adequately represents the coal company's interest.

In a criminal battery case brought against the defendant, the prosecutor asked the court to take judicial notice of the fact that a car driven from Chicago to Detroit has to cross state lines. The defense attorney raised no objection, and the judge declared that she was taking judicial notice of the fact as requested by the prosecution. What is the effect of such judicial notice? (A) To raise an irrebuttable presumption. (B) To satisfy the prosecutor's burden of persuasion on that issue. (C) To shift the burden of persuasion on that issue to the defendant. (D) That the judge should instruct the jury that it may, but is not required to, accept the noticed fact as conclusively proven.

(D) The effect of the judge's noticing that a car driven from Chicago to Detroit must cross state lines is that the judge will now instruct the jury that it may, but is not required to, accept that fact as conclusively proven. Under the Federal Rules, in a civil case, the court must instruct the jury to accept the judicially noticed fact as conclusive. [Fed. R. Evid. 201(f)] Because this question deals with a prosecution for criminal battery, the applicable rule is that the jury be instructed that the fact that has been judicially noticed may be accepted by it as conclusive, but that the jury is not required to do so. (A) would be correct if this were a civil case. In such an instance, the jury would be instructed to accept as conclusive the judicially noticed fact. This would have the effect of raising an irrebuttable presumption. (B) is incorrect because, in a criminal case, the prosecution has the burden of proving every element of the crime beyond a reasonable doubt. Only the jury can decide, after all of the evidence is in, whether the burden of persuasion is satisfied.

A defendant charged with driving while intoxicated pleaded not guilty and insisted on a trial. Right before the trial began, he fired his attorney and decided to defend himself. At one point during opening arguments, the defendant began to act like a cat, meowing and chasing an imaginary squirrel out of the courtroom. If no one else raises the issue of the defendant's competency to stand trial, what is the responsibility of the trial judge here? A The trial judge has no responsibility, because the defendant decided to defend himself. B The trial judge has no responsibility, because she cannot decide whether the defendant is competent to stand trial. C The trial judge must raise the issue of competency, because the defendant is representing himself. D The trial judge must raise the issue of competency, because the Constitution obligates her to do so.

(D) The judge must raise the issue of competency. If it appears to the judge that the defendant might be incompetent, the judge has a constitutional obligation to conduct further inquiry and determine whether in fact the defendant is incompetent. If the defendant is tried and convicted but it later appears that he was incompetent to stand trial, the judge's failure to raise the issue or to request a determination of competency does not constitute a waiver of the competency issue. Therefore, if the trial judge observes the defendant acting in such a way that may indicate he is incompetent to stand trial (e.g., meowing, chasing imaginary squirrels), she should conduct further inquiry to determine the competency of the defendant. (C) is incorrect. If there is evidence that the defendant might not be competent to stand trial, the judge should conduct an independent inquiry into competency regardless of whether the defendant is representing himself or is represented by an attorney.

While cross-examining a defendant on trial for robbery and assault with a deadly weapon, the prosecutor asks him whether he was convicted of fraud within the previous year. Is this question proper? A No, because fraud is not probative of a tendency to commit violence. B No, unless the proper foundation was laid. C Yes, because fraud is a form of stealing, and so it will tend to show that the defendant could commit robbery. D Yes, because it tends to show that the defendant would lie.

(D) The question is proper. The defendant has taken the stand in his own defense, and therefore the prosecutor can attack his credibility as a witness. Under Federal Rule 609, evidence of conviction of a crime requiring proof of an act of dishonesty or false statement can always be used to attack a witness's character for truthfulness. (A) is incorrect because even if fraud were probative of the tendency to commit violence, evidence of other crimes is not admissible to prove that a person has a propensity to commit criminal acts. (C) is incorrect for the same reason. (B) is wrong because no foundation is needed to show a prior conviction for impeachment purposes.

Using Character in Criminal Cases (5)

1. Character evidence is inadmissible unless defendant opens the door [prosecutor cannot introduce evidence of defendant's bad character if it is offered to show defendant acted in propensity with this bad character] 2. Defendant is allowed to present evidence of relevant good character to show he acted in conformity with good character and did not commit the crime [reputation or opinion evidence only on direct] 3. If the defendant takes advantage of 2 to show he did not commit the crime, defendant opened the door and prosecution can rebut with evidence of defendant's bad character; 4. Evidence of prior bad acts are never admissible to show propensity, but they can be offered to show motive, identity, absence of mistake, intent, common plan/scheme. [this evidence is always subject to FRE 303 regarding unfair prejudice.] 5. If defendant testifies he automatically places his character for truthfulness/credibility at issue.

Using Character in Civil Cases (3)

1. Character evidence is inadmissible unless it is directly in issue or an essential element of a plaintiff's claim or defense. [no propensity][defamation, negligent entrustment, hiring, or retention, or child custody] 2. If litigant has some other purpose for introduction of character and it is relevant, then the rule prohibiting character will not keep it out. 3. If defendant testifies he automatically places his character for truthfulness/credibility at issue.

Five ways to impeach a witness

1. Prior inconsistent statement (if given under oath it is for both impeachment but also as substantive evidence; can use extrinsic evidence) 2. Bias or motive to misrepresent (allows admissible and even allow extrinsic evidence if refuted) 3. Prior conviction (crime involving dishonesty or false statement, it is usable to impeach in any case and no discretion by judge to exclude; felony but not exceed 10 years) 4. Specific acts of misconduct that bear on truthfulness or untruthfulness. (can be inquired into on cross-examination, but no extrinsic evidence) 5. Bad reputation for truth or veracity. (can use extrinsic evidence)

The National Park Service recently created a new personnel level for field employees, which became the highest salaried position available to Park Service field employees. The position is restricted to employees over six feet in height. A female ranger who is five feet, three inches tall seeks your advice as to whether she can challenge the validity of the height restriction in federal court. If you decide to file suit on her behalf, which of the following would be your strongest argument against validity of the restriction?

Because most women are less than six feet tall, the restriction is an invalid discrimination on the basis of gender in violation of the DP Clause of the Fifth Amendment. (R. Equal protection applies to states, not federal;

Recently enacted legislation required farmers in certain counties of a western state to use drip irrigation systems instead of traditional methods in order to conserve water for agricultural and other uses. A farmer who refused to use the drip system was charged pursuant to the enforcement provisions of the legislation. A state court enjoined him from using other irrigation methods and fined him. The farmer appealed to the state supreme court, renewing his trial court claims that the irrigation legislation violated a state constitutional provision prohibiting certain governmental intrusions into private commercial activities and that it was preempted by federal water management statutes. The state supreme court held that the state constitution prohibited the challenged legislation, and construed the relevant statutes as being within the parameters of the federal statutes, and thus preempted. If the state petitions for certiorari to the US Supreme Court, how should the Court rule on the petition?

Deny the petition, because there is no substantial federal question that is dispositive of the case. (Would be an advisory opinion if State Court has already ruled that the law violates the State Constitution and this was decided on adequate and independent state grounds) (R. When may a Supreme Court rule on a State court decision? (1) Has to be a final judgment from a state court; (2) Had to come from highest state court; (3) Substantial federal question being raised; and (4) Decision from state court did not rest on adequate and independent state grounds.)

A vendor entered into a written contract with a purchaser for the sale of a large tract of land. The contract set forth an accurate metes and bounds description of the land based on a professional survey. At closing, the purchaser discovered that the deed was incorrectly transcribed and did not agree with the description of the land in the contract. The deed described the property to be conveyed as follows: "(i) from the southwest corner of [a specified starting point], proceed South 45 degrees East 200 feet to [a specified point]; (ii) from that point, proceed South 45 degrees West 100 feet to [a specified point]; (iii) from that point, proceed North 45 degrees West 200 feet to [a specified point]; and (iv) from that point, proceed South 45 degrees East 100 feet to the starting point." The purchaser refused to proceed with the closing and brought an action to reform the deed to make it conform to the intention of the parties. Which of the following corrections should be made for the deed to properly describe the land? A Direction (i) should be changed to "South 45 degrees East 100 feet." B Direction (iii) should be changed to "North 45 degrees West 100 feet." C Direction (iii) should be changed to "North 45 degrees East 200 feet." D Direction (iv) should be changed to "North 45 degrees East 100 feet."

Direction (iv) needs to be corrected in its course but not its distance. In land contracts and deeds, property may be described in various ways as long as the description is unambiguous. From a designated starting point that can be identified by reference to a government survey or a natural or artificial monument, the boundaries of the property can be described by successive calls of courses (e.g., angles) and distances until returning to the starting point. A course is a statement of direction generally stated as some number of degrees east or west of due north or south. In each call a distance must be stated together with the course. Thus, the boundary in direction (iv) runs at an angle 45 degrees east of due south (i.e., southeast) for a distance of 100 feet. However, because direction (i) went southeast, direction (ii) went southwest, and direction (iii) went northwest, the fourth direction has to be northeast for a distance of 100 feet to bring the final boundary back to the starting point. (In this type of question, diagram the boundaries as shown below to help you visualize the property.) Therefore, the correction in choice (D) is correct. (A), (B), and (C) are incorrect because none of those proposed changes in distance or direction would be sufficient to bring the final call back to the starting point.

A father conveyed his property to his son and daughter "as joint tenants with right of surviviorship, but if they ever attempt to sell the property during their lifetimes, a right of first refusal based on the sale price is hereby granted to my sister." Unbeknownst to the son or the sister, the daughter quitclaimed her interest in the property to a purchaser. The following month, the daughter was killed in a snowmobile accident. The purchaser of the daughter's interest filed a suit for partition of the property. The son filed an appropriate counterclaim for quiet title, asserting that he was the owner of the entire parcel. The sister also filed a counterclaim, asserting that her right of first refusal was valid and that she was prepared to exercise her right to purchase the property for the contract price. In a jurisdiction in which the RAP is unmodified by statute, how should the court rule?

For the sister, because she has a valid right of first refusal. (R. rights of first refusal have to follow RAP; not an unreasonable restraint on alienation if can sell, but must first offer ROFR ) (Not violate RAP here because right of first refusal is for the life of the sister, so we will know during her life whether or not ROFR will be exercised)

To encourage minority business and foster pride in minority heritage, a state adopted legislation exempting magazines and other periodicals from the state's receipts tax if 20% of the magazine is devoted to articles concerning minorities ( a commission was set iup to sample magazines to determine on a yearly basis whether they should be exempt.) A publisher produced a sports magazine in the state that occasionally contained articles about minority athletes, but the commission determined that the publisher's magazine was not eligible for the receipts tax exemption. After paying the tax assessed on her magazine, the publisher sued for a refund.

In favor of the publisher, because the tax violates the 1st Amendment freedoms of speech and press. (R. Content based and viewpoint regulations on speech are preemptively unconstitutional and tested under a strict scrutiny standard of review and are rarely upheld)

The victim collapsed at her desk while drinking her morning coffee and was rushed to the hospital. Later that night, the victim's brother went to visit the victim in the intensive care unit. Barely conscious, the victim said "I've thought about this all day and it must have been my assistant. She brought me my coffee this morning before I could make it for myself, and she's never done that before. Don't let her get away with murder." The victim soon lost consciousness and lapsed into a coma, and she remains in this vegetative state. It was determined that she was poisoned. The assistant is arrested and charged with attempted murder. At the assistant's trial, the prosecution wishes to call the victim's brother to testify to the victim's statement about the assistant. The court should find the statement:

Inadmissible, because it is hearsay not within any exception. (R.Dying declaration 4 requirements 1. unavailability on part of the declarant at trial; 2 Belief that death is imminent; 3 statement concerning the cause or circumstances surrounding the belief of death; and 4. applies in any civil case and in criminal homicide cases.) (Therefore, here it would not count as a dying declaration because it is a criminal case that is for attempted murder, not homicide.)

The legislature of a state was concerned that the numerous and strident television, radio, and newspaper advertisements by auto dealerships annoy and mislead the public. Therefore, it enacted comprehensive legislation regulating the timing and content of such ads, limiting their duration, frequency, and the types of claims and information made and given. Which of the following statements is most accurate as to the constitutionality of the state's ad regulation?

It is constitutional if it does not prohibit the dissemination of truthful information about price and the availability of products, and is narrowly tailored to serve a substantial government interest. (R. Commercial speech allowed as long as they serve a substantial government interest; Intermediate scrutiny used for: zoning laws on adult movie theaters and bookstores, nude dancing bands, symbolic speech, content neutral time place and manner restrictions.)

A state statute prohibits leaving a child under the age of five years unattended in an automobile. A mother parked her car at a supermarket parking lot. She left her four-year-old son in the car with his seat belt fastened while she did her grocery shopping. While the mother was shopping, the son undid his seat belt, left the car, and started riding on the grocery carts that customers had left in the parking lot. The son crashed one of the carts into another shopper's car, causing damages. The shopper brought a negligence action against the mother to recover for the damage caused by the son. At trial, the shopper presented evidence of the statute and the facts stated above. At the conclusion of the shopper's case, the mother moved for a directed verdict in her favor. Should the court grant the mother's motion?

No, because the jury could find that it was foreseeable that the son would cause damage to cars in the parking lot if the mother left him unattended. (R. Negligence per se should only be picked if statute designed to prevent that type of harm and protect that particular class.)

A customer slipped and fell in a store, suffering a severe injury. Several weeks after the accident, anticipating that the customer would file an action against it, the store's attorney had the store manager interview any employees who were near the accident to determine what they saw or heard. The store manager did so, taking handwritten notes. The notes are now in the store's possession. The customer subsequently filed a civil action against the store in federal district court. The complaint alleged that the store negligently left a spill on the floor of the store, causing the customer's fall. The customer's attorney served on the store a request for production of documents, which included a request for all documents and reports prepared by the store that relate to the customer fall and injury. Must the store produce to the customer the notes taken by the store manger when he interviewed the store's employees?

No, because the manager's notes are protected by a qualified immunity from discovery under the work product doctrine (R. Work product doctrine is a qualified immunity [not absolute immunity]; not have to be produced unless it can be shown that there is a substantial need or an undue hardship)

A tourist from State A was severely injured in a bar fight in State B. The tourist filed a battery action against one of the bar's patrons, seeking $100,000 for his injuries. The defendant patron claims that the tourist is mistaken about who hit him. The patron says that he did not touch the tourist. The patron claims that it was the bar's bouncer--who looks like the patron-- who hit the tourist and then continued to pummel him. The bouncer claims that he never touched or harmed the tourist. Can the patron assert a third-party claim against the bouncer to bring him into the action?

No, because the patron has no legal basis to assert a claim against the bouncer (patron is denying liability all together) and is not seeking to recover from the bouncer any portion of the patron's liability to the tourist.

A merchant owned a skate rental business that she operated out of a specially equipped van. She would drive to various parks and public beaches within her home state and rent roller skates, related safety equipment and lightweight stereo/earphone sets to passerby on an hourly basis. She also sold skates and skating equipment. About 50% of the merchant's time is spent in a single city, and she earns about 70% of her gross rental and sale income at that city's beach areas. After receiving numerous complaints from beach goers about the sidewalks congested with roller skates, the city council passed an ordinance prohibiting roller skating on public property between the hours of 7 am and 9 pm If the merchant seeks to enjoin enforcement of the ordinance in federal district court on the basis that it is unconstitutional, what should the court do?

Reach the merits of the merchant's challenge because enforcement of the ordinance will harm her business and the rights of the public are linked to her rights (R. A seller of goods may have third party standing to challenge a law that adversely affects the rights of her customers. [standing plus this])

After leaving ceremonies at which the chief justice of a state supreme court had been named distinguished jurist of the year, an associate justice was interviewed by the press. The associate justice told a reporter that the chief justice "is a senile imbecile who lets his clerks write all his opinions. He hasn't had a lucid thought in decades, and he became a judge by being on the payroll of the mob." Enraged, the chief justice brought an action for defamation against the associate justice. Which of the following, if established by the chief justice in his defamation action, would permit recovery against the associate justice?

The associate justice made the statements knowing they were false. (R. Defamation: False statement by the defendant that injures the plaintiff's reputation; Public figure: must show published defamatory statement with actual malice [knowledge of falsity or reckless disregard of the truth] ;Public concern/matter: only have to show negligence and damages [4 per se situations where damages are presumed: 1. Loathsome disease 2. Impugning a women's chastity when it is not true 3. Accusing someone of committing a crime of moral turpitude 4. Impugning someone's profession, trade or business])

A testator executed a will, devising his land "to my son and my daughter, share and share alike." Shortly thereafter, the daughter died intestate, leaving a child as her only heir. The next year, the testator and his son were involved in a car accident The testator died immediately. The son died six days later leaving a will that bequeathed his entire estate to his wife. The jurisdiction has the following statute: "If a devisee, including a devisee of a class gift, who is a grandparent or a lineal descendant of a grandparent of the testator is dead at the time of execution of the will or fails to survive the testator, the issue of such deceased devisee shall take the deceased's share under the will. Who owns the land.

The daughter's child and the son's wife each own an undivided one-half interest in the land. (R. Under common law, if a beneficiary predeceases the testator, it would pass to testator's residuary unless there is an anti-lapse statute.)

An owner of three acres of lakefront property subdivided it and sold two acres to a buyer, retaining the one acre actually fronting on the lake. The deed for the two acres expressly included an easement over the westernmost 30 feet of the one-acre parcel retained by the owner for access to the lake. The buyer recorded his deed in the county recorder's office, which maintained an alphabetical grantor-grantee index only. Fifteen years later, the owner died, leaving the one-acre parcel to his wife. She sold it to a developer that planned to build condominiums. A month later, the buyer died, and his two acres passed by will to his nephew. Three weeks after taking title to the property, the nephew visited the property and discovered that the developer had erected a chain link fence all along the boundary between the nephew's land and the acre of lakefront land. The nephew brings an action to enjoin the developer from obstructing his easement across the acre of lakefront property. Which of the following best describes why the nephew should prevail in this litigation?

The nephew's easement is a legal interest that the developer had record notice of, even though there is no tract index. (Recorded notice, so no track index is irrelevant.) (R. Easement can be created by: express easement, adverse prescription/possession, landlocked; Easement terminated by: abandonment,

An American tourist was visiting another country when he was warned by US health authorities to go immediately to a hospital because he had a serious and extremely contagious disease that required him to be quarantined. He decided to ignore the warning and instead traveled on an airline flight back to the US. Despite the tourist's belief that he would not be discovered and his best efforts to keep a low profile, the news media were tipped off to what he had done and publicized it. When a passenger who had been sitting next to the tourist on the plane learned about it, she became extremely upset, fearing that she would contract the disease. The passenger brought a negligence action to recover for the distress she suffered but the jury rejected her claim. Why?

The passenger did not suffer physical injury from her distress. (R. Must suffer physical harm or injury for NIED [Exceptions: 1. mishandling a corpse; or 2. False report of death])

A philanthropist told his friend, who was a state governor, that he planned to build a museum. The governor thought that the museum would bolster the state's tourism industry and offered to arrange to have the state purchase land and grant it to the museum to enable the philanthropist to build a bigger museum with his money than originally planned. The philanthropist agreed, and the museum was built. The philanthropist undertook the hiring of the museum's senior staff. He was of German descent and was ashamed of Germany's actions during World War II. To assuage his own conscience, he refused to hire anyone whom he believed to be of German descent. A restoration expert applied for a job as chief curator of the museum, but the philanthropist refused to hire him because of his German background. The restoration expert discovered the philanthropist's rationale and brings suit against the museum, claiming that the hiring practice violates his constitutional rights. How is the court most likely to rule?

The policy is constitutional, because the museum is a private entity and so may constitutionally hire and fire as it desires. (Just giving land is not enough for state action) (R. If a private citizen is discriminating, it is usually not going to violate an individual's constitutional rights; only if the government is significantly involved in the the activity will you find state action, incidental involvement is not enough.)

A landowner who had owned and operated a small airport notified the electric company that he was discontinuing operations and that it should shut down the electrical current that had supplied his communications equipment. The equipment had been surrounded by a fence and signs warning of high voltage. Because the electric company had maintained a transformer next to the landowner's communications equipment that contained many valuable and reusable parts, it decided to leave the power on to prevent theft until it could schedule removal of the transformer. Three days later, a trespasser who knew that the airport had closed went onto the property looking for something to steal. He could find nothing of value except the transformer. He noticed the signs warning of the high voltage but believed that the power had since been turned off. He scaled the fence with the intent to dismantle the transformer. As soon as he touched the transformer, he was seriously injured by the electric current. If the trespasser asserts a claim against the electric company to recover damages for his injuries, will he prevail?

Yes, because the electric company used unreasonable force to protect its property. (R. You cannot use deadly force to protect property.)

The defendant was charged with embezzling $1 million from his employer, a bank, by transferring the funds to a secret offshore account in the bank's name. Only the defendant and the bank's VP were authorized to draw funds from the account. The defendant testified that he had wired $1 million to the account but had done so at the direction of the bank's VP. The defendant stated under oath that he had no intent to embezzle bank funds. The government's cross-examination of the defendant concentrated exclusively on his relationship and conversations with the VP, who has committed suicide. The defense now seeks to call a second witness, who is prepared to testify that he ad worked with the defendant for 10 years and that the defendant had a reputation in both the business and general communities as being a very honest person. Is the witness's testimony admissible.

Yes, to hep show that the defendant did not embezzle funds. (R. Normally you cannot bolster your witness's credibility before it has been attacked. Exception: if it is being offered for some other purpose.) (Here, he was charged with embezzlement, so he is allowed to present evidence of his good character reputation to show he is honest and not the type of person that would embezzle.

After picking up a load of hazardous chemical waste, a truck driver for a waste management company set out on the road to his next stop. However, he had failed to secure the latch on the back panel of the truck. Consequently, the panel opened while the truck was on the road, and a metal canister full of chemical waste fell onto the road. A car struck the canister, causing the car to veer off the road and injure the driver. The driver filed suit against the company for his injuries. The jurisdiction in which the above events took place has adopted a rule of partial comparative negligence. At trial, the driver of the car admitted that he had momentarily taken his eyes off the road to look at his speedometer. When he had looked up again, the canister was there and he could not stop in time. The jury found that the company, through its truck driver, had acted willfully and wantonly and was 90% at fault, while the driver of the car was 10% at fault. The driver filed a motion for judgment notwithstanding the verdict, seeking recovery for 100% of his damages. If the judge grants the motion, what is the most likely reason? A A plaintiff's comparative negligence is not taken into account in cases of willful and wanton conduct by the defendant. B A state ordinance mandating motorists to stay within the posted speed limit requires as a matter of law an occasional glance at the speedometer. C The company was more than 50% at fault. D The company was engaged in an abnormally dangerous activity.

(B) If the driver was effectively required by statute to take an occasional quick look at his speedometer to make sure that he was complying with appropriate speed limits, then his momentary glance at the speedometer in the instant case would, as a matter of law, not constitute negligent conduct. Because this is a matter of law, the judge would be authorized to correct this aspect of the jury's verdict. If the driver is thus found to be not negligent in this matter, his recovery will not be reduced. (A) is incorrect because in most states that have adopted comparative negligence, the plaintiff's negligence will be considered even in cases where the defendant has acted willfully and wantonly. (C) is incorrect because the fact that the defendant is more than 50% at fault does not mean that the plaintiff is entitled to receive 100% of his damages from the defendant in a partial comparative negligence jurisdiction. It only means that the plaintiff's recovery is not totally defeated. (D) is incorrect because, although the transportation of chemical waste would probably be considered an abnormally dangerous activity, liability for conducting an abnormally dangerous activity attaches only if the harm results from the kind of danger to be anticipated from such activity; i.e., the injury must flow from the normally dangerous propensity of the activity. The canister falling from the truck is not the "normally dangerous propensity" of transporting chemical waste.

Jeopardy attaches in a jury trial when: A Opening statements begin B The jury is empaneled and sworn C The first witness is sworn D The judge delivers jury instructions

(B) Jeopardy attaches in a jury trial when the jury is empaneled and sworn. Under the Fifth Amendment right to be free of double jeopardy, a defendant may not be retried for the same offense once jeopardy has attached. Jeopardy does not attach in a jury trial when the first witness is sworn. This is when jeopardy attaches in a bench trial, not in a jury trial. By the time opening statements begin or the judge delivers jury instructions, jeopardy has already attached.

A township located in a farming community was composed mostly of persons belonging to a specific religious sect. To help instill proper respect for authority in children, which was a central tenet of the sect, and to maintain order in the classroom, the local school board allowed teachers to inflict corporal punishment. Such punishment was inflicted on a fourth grader in a township school immediately after his teacher saw him pulling a girl's hair. Neither he nor his parents belonged to the religious sect. When the boy's parents learned of the incident, they hired an attorney. Rather than suing the teacher for battery as permitted under state law, the attorney brought an action against the teacher under a federal statute providing a cause of action for damages against any government employee who deprives a person of his constitutional rights. Should the court find the policy allowing corporal punishment to be constitutional? A No, because the punishment policy violates the First Amendment Establishment Clause. B No, because the boy was denied any kind of hearing, in violation of his right to procedural due process under the Fourteenth Amendment. C Yes, because under the doctrine of parens patriae states may impose any punishment they see fit. D Yes, because the punishment was not grossly disproportionate under the Eighth and Fourteenth Amendments.

(D) The punishment here is constitutional because it does not violate any constitutional provision. The best answer reflecting this reasoning is (D)—there was no Eighth Amendment violation here—because paddling students as a disciplinary measure has not been found to be cruel and unusual punishment. (A) is incorrect because there is no Establishment Clause violation here. Under the Establishment Clause, if there is no sect preference, government action generally will be upheld if the action serves a secular purpose, its primary effect neither advances nor inhibits religion, and it does not excessively entangle government with religion. There is no sect preference under the school board's corporal punishment rule here, the rule has the secular purpose of maintaining order in the classroom (the fact that this coincides with the tenets of a local religion does not change that conclusion), its main purpose neither advances nor inhibits religion, and there is no excessive entanglement. (B) is incorrect because there has been no deprivation of procedural due process. The Supreme Court has held that although corporal punishment may involve a liberty interest, no hearing is required prior to inflicting such punishment; the possibility of a common law action in tort is sufficient procedural protection. [Ingraham v. Wright (1977)] (C) is incorrect because it is too broad. The doctrine of parens patriae allows the state to stand in the shoes of a parent, but even a parent may not impose any punishment he sees fit (e.g., a parent may not break a child's arm as punishment for stealing).

Specific Intent Crimes

solicitation, conspiracy, attempt, first degree murder, assault, larceny, embezzlement, false pretenses, robbery, burglary, forgery

A state statute provided for criminal penalties for "knowingly selling alcoholic beverages in violation of the regulations of the State Liquor Commission to any person under the age of 18." One of the State Liquor Commission regulations provided that "before an alcoholic beverage is sold to any person between the ages of 17 and 24, the seller must demand some form of photo identification to determine the buyer's age." A minor who looked much older than his age of 17 walked into a tavern located in the state and asked the bartender for a beer. The bartender never asked the minor for any form of identification, as he thought that he was at least 25 years old. Had the bartender asked for identification, the minor would have shown him a fake identification card showing that he was 21 years old. The bartender served the beer to the minor, who consumed it on the premises. The bartender was subsequently charged under the state statute for selling the beer to the minor. Is the bartender guilty? A No, because he reasonably believed that the minor was older than 25 years. B No, because the minor had fake identification with which he could have obtained the beer. C Yes, because he sold an alcoholic beverage to a minor, a strict liability crime. D Yes, because he failed to ask for identification, and the regulation does not provide for a mens rea requirement.

(A) The bartender's reasonable belief that the minor is 25 years old is a mistake of fact that negates the state of mind required by the statute. Ignorance or mistake as to a matter of fact will affect criminal guilt only if it shows that the defendant did not have the state of mind required for the crime. In addition, the mistake must be reasonable unless the offense is a specific intent crime. Here, the statute requires that the defendant have acted "knowingly" with respect to each of the material elements of the offense. A person acts knowingly with respect to the nature of his conduct when he is aware that his conduct is of that nature or that certain circumstances exist. At least one of the material elements of the offense here is that the sale be to a person under the age of 18. If the bartender believed that the minor was 25 years old, the bartender has not acted knowingly with respect to the fact that the purchaser was under 18, and he cannot be convicted of violating the statute.(D) is not as good an answer as (A). Although the regulation apparently does not have a state of mind requirement, it is not entirely clear that criminal liability can result from the violation of the regulation. There would have to be a separate statute providing for criminal penalties for failing to check for identification. Furthermore, the criminal statute in question clearly has a state of mind requirement that must be satisfied, as explained above.

A plaintiff sued a defendant over a claimed debt. At the trial, the plaintiff established the existence of the debt and testified that he never received payment. In response, the defendant presents evidence sufficient to establish that she took her check to the post office and sent it to the plaintiff's proper address by certified mail. The defendant offers a certified mail receipt with an illegible signature, which she claims is the plaintiff's signature. The defendant also presents evidence that her basement flooded on March 28, and she claims that she cannot produce a canceled check because her box of canceled checks was destroyed from the water damage. Evidence is also presented that, due to a computer glitch, the defendant's bank cannot reproduce her checking account records for the months of February and March. After the defendant's testimony, which of the following is correct? A The burden of persuasion and the burden of going forward with the evidence are on the plaintiff. B The burden of persuasion is on the plaintiff, but he has no burden of going forward with the evidence. C The plaintiff has satisfied his burden of persuasion, but he has a burden of going forward with the evidence. D The plaintiff has satisfied both his burden of persuasion and his burden of going forward with the evidence.

(A) The burden of persuasion and the burden of going forward with the evidence are on the plaintiff because the defendant's testimony raises a rebuttable presumption that the check had been delivered in the mail. The burden of persuasion is the burden of a party to persuade the jury to decide an issue in its favor. If, after all the proof is in, the issue is equally balanced in the mind of the jury, then the party with the burden of persuasion must lose. The burden of persuasion does not shift from party to party during the course of a trial. Because the plaintiff sued the defendant for the debt, the plaintiff has the burden of persuasion when the time for the jury to make a decision arrives. The burden of going forward with the evidence is the burden of producing sufficient evidence to create a fact question of the issue involved. If a plaintiff makes out a prima facie case, he has met his burden of going forward with the evidence and the burden shifts to the defendant. Here, when the plaintiff made out a prima facie case of the defendant's debt, the burden of going forward with the evidence shifted to the defendant. The defendant met this burden through the use of a presumption. Federal Rule 301 provides that a presumption imposes on the party against whom it was directed the burden of going forward with the evidence to rebut the presumption. The defendant's evidence regarding the proper posting of the check raises a rebuttable presumption that the check was delivered to the plaintiff because a letter shown to have been properly addressed, stamped, and mailed is presumed to have been delivered in the due course of mail. Therefore, the burden of going forward with the evidence has shifted back again to the plaintiff, who must now produce evidence to rebut the presumption (i.e., evidence that he did not receive the check).

In an action by the plaintiff against the defendant, one of the issues is whether the defendant is a licensed physical therapist. Normally, the names of all licensed physical therapists are registered with the office of the state Department of Professional Registrations. The plaintiff wishes to introduce a certified document, signed by the chief registrar of the department (who cannot be located), stating that an examination of the department's rolls does not disclose the defendant's name. Should the document be admitted? A Yes, because a statement of absence from public record is admissible. B Yes, because the chief registrar is unavailable. C No, because the document is hearsay not within an exception. D No, because the document is not self-authenticating.

(A) The document should be admitted. Related to the exception to the hearsay rule for public records and other official writings, Federal Rule of Evidence 803(10) provides that evidence in the form of a certification or testimony from the custodian of public records that she has diligently searched and failed to find a record is admissible to prove that a matter was not recorded, or, inferentially, that a matter did not occur. Here, the defendant's status as a licensed physical therapist would normally be revealed in the records of the department. The document here at issue is admissible, under the foregoing hearsay exception, as a means of proving that the defendant is in fact not licensed. (B) is incorrect because this hearsay exception does not require unavailability of the declarant. (C) is incorrect because, as explained above, the statement of absence from public record forms an exception to the hearsay rule. (D) is incorrect because a public document that has been signed and certified is self-authenticating under Rule 902; hence, no testimonial sponsorship for the document is required.

An employee of the United States Department of Labor was instructed by his superior to solicit subscriptions to the Department's bulletin on a door-to-door basis in the city in which he worked. While doing so, the employee was arrested for violation of a city ordinance that prohibited commercial solicitation of private residences. What is the employee's best defense? A Intergovernmental immunity. B The First Amendment freedom of expression as it applies to the states through the Fourteenth Amendment. C The Equal Protection Clause as it applies to the states through the Fourteenth Amendment. D The city ordinance effectively restricts interstate commerce.

(A) The employee's best defense is intergovernmental immunity. State and local governments cannot tax or regulate the activities of the federal government. This principle is often termed "intergovernmental immunity." The arrest and prosecution of a federal employee who was on the job violates this principle, which is based on the supremacy of the federal government and federal law. (B) is not a bad answer because door-to-door solicitation is protected by the First Amendment. However, at best, (B) would subject the city's actions to strict scrutiny and allow the city to prevail if it could prove that its action was necessary to achieve a compelling government purpose. In contrast, (A) would automatically invalidate the city's enforcement of the law against the employee, and so (A) is a better answer. (C) is irrelevant because the Fourteenth Amendment's restriction on the states has to do with persons, not the federal government, and here there is no claim that the city was discriminating against the employee. The city's ordinance, as briefly described, does not seem to provide the basis for an equal protection claim. (D) is wrong because nothing in the facts shows any burden on interstate commerce. Moreover, at most such a claim would trigger heightened scrutiny; it would not automatically invalidate the enforcement of the law as would (A).

A landowner conveyed his land "to my son for life, then to my son's widow for her life, then to my son's children." At the time of the conveyance, the son was 20 years old and unmarried. The son eventually married and had two children, the landowner's grandson and granddaughter. Many years later, the landowner and the grandson were involved in a train accident. The landowner was killed instantly. The grandson died a short time later of his injuries. The landowner left his entire estate by will to his friend. The grandson's will devised his entire estate to the city zoo. The son's wife was so grief-stricken that she became ill and died the next year, leaving her entire estate to her husband. Eventually the son met and married a 21-year-old. Ten years later, the son died, leaving everything to his second wife. When the second wife moved onto the land, the granddaughter filed suit to quiet title to the land, joining all of the appropriate parties. If the jurisdiction recognizes the common law Rule Against Perpetuities, unmodified by statute, in whom will the court most likely find that title to the land is held? A One-half in the granddaughter and one-half in the city zoo, subject to the second wife's life estate. B One-half in the granddaughter and one-half in the second wife, because the second wife took the son's interest. C Entirely in the friend, subject to the second wife's life estate, because the gift to the son's children violates the Rule Against Perpetuities. D Entirely in the granddaughter, subject to the second wife's life estate, because the grandson did not survive the son.

(A) The granddaughter and the city zoo each own one-half of the land, subject to the second wife's life estate. At the time of the conveyance by the landowner, the son had a life estate, the son's widow had a contingent interest (because the son's "widow" cannot be ascertained until the son's death), and the son's children had a contingent remainder (because they have not yet been born). When the grandson and the granddaughter were born, however, their interests became vested subject to open (i.e., if the son had more children). Thus, when the grandson died, he had a vested remainder subject to open that he was free to devise by will; the city zoo took his vested remainder subject to open. At the son's death, the class of his "children" closed (because the son could not have any more children), and the granddaughter's and the zoo's vested remainders subject to open became indefeasibly vested. Also at the son's death, his widow was ascertained and her interest vested in possession. Because the second wife was the son's widow, she is entitled to the valid life estate. Thus, the granddaughter and the city zoo hold one-half interests, subject to the second wife's life estate. (D) is wrong because the gift to the son's children was not conditioned on their survival of the son. The law does not imply such a condition. The grandson's interest was vested subject to open and could be disposed of by his will.

A manufacturer of high-speed computers entered into a written agreement with a distributor whereby the distributor would purchase a specified computer from the manufacturer for $50,000. The parties had orally agreed that the delivery date would be November 4. However, when the agreement was reduced to writing, a glitch in the word processor caused the printout to show the delivery date as "12/4" instead of "11/4." Both parties signed the paper without noticing the incorrect delivery date. Before reducing their agreement to writing, the parties had also orally agreed that the agreement would not become binding unless the distributor notified the manufacturer, in writing, by October 7, that it (the distributor) had obtained a buyer for the computer. On September 25, the distributor found a buyer who needed the computer for her business and who agreed to buy it from the distributor. However, the distributor did not inform the manufacturer that it had found a buyer until October 30. In the meantime, due to a strike at the manufacturer's leading competitor, the price of high-speed computers rose rapidly during the month of October. By the end of the month, the market value of the computer in question was $70,000. Because of the increase in the value of the computer, the manufacturer does not want to deliver the specified computer to the distributor for $50,000. Which of the following provides the manufacturer the best defense if the distributor sues to enforce the contract? A The increase in value of the computer makes the contract unconscionable. B There has been a failure of a condition precedent. C A case of mutual mistake exists because of the word processing error regarding the delivery date. D There has been a failure of a condition subsequent.

(B) By oral agreement, the written agreement between the manufacturer and the distributor was not to take effect unless the distributor notified the manufacturer in writing by October 7 that it (the distributor) had obtained a buyer. The distributor did not provide notice that it had found a buyer until October 30. Generally, under the parol evidence rule, when the parties express their agreement in a writing with the intent that it embody the final expression of their bargain, any other expressions—written or oral—made prior to the writing, as well as any oral expressions contemporaneous with the writing, are inadmissible to vary the terms of the writing. However, if a party asserts that there was an oral agreement that the written contract would not become effective until a condition occurred, all evidence of the understanding may be offered and received. The rationale is that the written agreement is not being altered by parol evidence because the written agreement never came into being. Here, the notification by October 7 was a condition precedent to the effectiveness of the contract. Thus, evidence of the oral agreement is admissible. Since the contract never became binding, the manufacturer has no obligation to deliver the computer for $50,000 or otherwise. (D) is incorrect because the condition that failed was precedent rather than subsequent. The condition was both precedent to the agreement itself and phrased in condition precedent terms. A condition subsequent is one the occurrence of which cuts off an already existing absolute duty of performance. (A) is incorrect because contractual duties will be discharged by unconscionability only if, at the time the contract was made, it was one-sided and terribly unfair to one of the parties. Here, at the time the contract was made, nothing about it seemed unfair, and the subsequent increase in price does not render the contract unconscionable. (C) is incorrect because the mistake regarding the delivery date is a mere clerical error, rather than a mistake of fact going to a point that is material to the transaction. As such, this is not the type of mistake that would relieve either or both of the parties of their obligations under the contract.

A boater taking his new powerboat out on a large lake ran out of gas because of a defective seal in the gas tank. The defect was not discoverable by an ordinary inspection. His frantic signaling alerted the captain of a sightseeing boat passing by. The captain pulled up alongside to assist and attempted to restart the boat. A spark ignited a pool of gas that had leaked from the gas tank and collected in the lower part of the boat, causing an explosion and fire. The captain was severely burned and died from his injuries. The captain's estate brought a wrongful death action based on strict liability against the powerboat dealer and the manufacturer. Evidence at trial established that the dealer had sold the manufacturer's boats for years without any problems reported by customers. Can the captain's estate recover any damages from the dealer? A Yes, unless the jury finds that the boater was negligent in failing to investigate where the gas had gone. B Yes, because harm to someone in the captain's position was a foreseeable result of the gas leak. C No, because the dealer had no reason to anticipate that the manufacturer assembled the gas tank improperly. D No, because the captain did not have a sufficient relationship to the boater to make the dealer liable for the captain's death.

(B) The captain's estate can recover from the dealer on a strict products liability ground because the captain was a foreseeable bystander and the dealer is a commercial supplier. Recovery in a wrongful death action is allowed only to the extent that the deceased could have recovered in a personal injury action had he lived. The captain could have recovered from the dealer in a products liability action based on strict liability because the dealer is a commercial supplier of the boat and is liable if it sold a product that was so defective as to be unreasonably dangerous. The defect in the assembly of the gas tank was unreasonably dangerous because it allowed gas to leak out and collect where it could be ignited. The dealer would be liable to the captain, despite the fact that he was not in privity with the dealer, because he was a foreseeable plaintiff. The disabling effect of the gas leak made it foreseeable that someone passing by would come to the boater's assistance and thereby come within the zone of danger from the leak (i.e., danger invites rescue). The explosion that resulted from the leak was the actual and proximate cause of the captain's death. Therefore, the captain's estate can recover damages from the dealer. (A) is wrong because, as with proximate cause analysis in ordinary negligence actions, the negligence of a subsequent actor is foreseeable and therefore not a superseding cause that would cut off the liability of the original tortfeasor. In any products liability case, the negligent failure of an intermediary to discover the defect or the danger does not void the commercial supplier's strict liability. Hence, the boater's negligence would be irrelevant. (C) is wrong because in products liability actions based on strict liability, the retailer may be liable even if it had no reason to anticipate that the product was dangerous or had no opportunity to inspect the product for defects. While the dealer could assert that defense if the action were based on negligence, the call of the question indicates that the action is based on a strict liability theory. Under strict liability, the dealer is liable simply because it is a commercial supplier of a product with a dangerous defect. (D) is wrong because the fact that the captain was not in privity with the dealer is irrelevant in a products liability action based on strict liability. Strict liability applies not only to buyers, but also to the buyer's family, friends, and employees, and to foreseeable bystanders. As a rescuer, the captain was a foreseeable bystander to whom the dealer may be liable.

A landlord entered into a written lease of a bakery for a term of 25 years with a baker. The parties agreed to a right of first refusal if the bakery was offered for sale during the term of the lease. The lease also permitted assignments and subleases on notice to the landlord. Three years later, the baker retired and, after notifying the landlord, transferred the lease to a chocolatier. Twenty-one years later, the landlord entered into a contract with a buyer for the sale of the bakery for $100,000. The landlord had informed the buyer of the lease but had forgotten about the right of first refusal. When the chocolatier learned of the sale to the buyer, she informed both the landlord and the buyer that she wanted to exercise her option and was prepared to purchase the bakery for the contract price. The jurisdiction's Rule Against Perpetuities is unmodified by statute. Can the chocolatier enforce the option? A Yes, because an option held by a tenant on leased property cannot be separated from the leasehold interest. B Yes, because the option touches and concerns the leasehold estate. C No, because the transfer to the chocolatier made the option void under the Rule Against Perpetuities. D No, because the option was not specifically included when the lease was transferred to the chocolatier.

(B) The chocolatier can enforce the option to purchase because it is a covenant that runs with the land. When a tenant makes a complete transfer of the entire remaining term of his leasehold interest, it constitutes an assignment. The assignee and the landlord are then in privity of estate, and each is liable to the other on all covenants in the lease that run with the land. The covenant runs with the land if the original parties so intend and the covenant "touches and concerns" the leased land, i.e., burdens the landlord and benefits the tenant with respect to their interests in the property. Here, the transfer of the lease to the chocolatier was an assignment, making all covenants in the lease that run with the land enforceable by the assignee. The right of first refusal burdens the landlord's power of alienation over the bakery, and there is nothing to indicate that the parties intended the option to be personal to the baker. Hence, the chocolatier can enforce the option and purchase the property. (D) is incorrect because, as discussed above, the option is a covenant that runs with the land regardless of whether it was specified in the assignment to the chocolatier. The chocolatier, as the assignee of the leasehold, can enforce the option on privity of estate grounds.

A consumer purchased a new television set from an electronics store. When he got home, he opened the box and found an owner's manual that contained operation instructions, warnings regarding the danger of electricity, and a warranty that stated: "The store expressly warrants that this set shall be free of manufacturing defects for 30 days. If a set is defective, the store's liability shall be limited to the cost of repair or replacement of defective parts. The store "HEREBY DISCLAIMS ANY AND ALL OTHER WARRANTIES, EXPRESS OR IMPLIED, INCLUDING THE WARRANTY OF FITNESS FOR PARTICULAR PURPOSE AND THE WARRANTY OF MERCHANTABILITY."" Five weeks later, after the set was properly installed, the consumer turned on the set, heard a crackling noise, and watched as his television exploded and was destroyed. Under which of the following theories will the consumer most likely recover? A Breach of express warranty. B Breach of the implied warranty of merchantability. C Breach of the implied warranty of fitness for a particular purpose. D Breach of the warranty of reasonable workmanship.

(B) The consumer will most likely recover for breach of the implied warranty of merchantability. In every sale of goods, unless expressly disclaimed, there arises a warranty that the goods will be merchantable, which means that they will be fit for the ordinary purposes for which such goods are used. A television that explodes after five weeks of use likely breaches this warranty. The warranty will not be disclaimed because, to be effective, a disclaimer must be part of the offer and acceptance process or must be agreed to by the buyer as a modification. Here, the "disclaimer" was in the box, and the consumer did nothing to accept the disclaimer. Thus, (B) is correct. (A) is wrong because no express warranty was made as to quality at a time sufficient to become part of the bargain. An express warranty will arise from any affirmation of fact or promise made by the seller to the buyer, or from any description of the goods, and any sample or model if the statement, description, sample, or model is part of the basis of the bargain. Here, there was no such affirmation, promise, description, sample, or model indicating that the television would not explode; that is something that would be understood (implied). (C) is wrong because an implied warranty of fitness for a particular purpose arises when (i) the seller had reason to know the particular purpose for which the goods are to be used and that the buyer is relying on the seller's skill and judgment to select suitable goods; and (ii) the buyer in fact relies on the seller's skill or judgment. The facts in this case do not point to such a warranty; on the contrary, the consumer selected the television himself. (D) is wrong because the warranty of reasonable workmanship is a common law doctrine generally applied to builders. This is a contract for the sale of goods and is governed by the UCC, not common law.

A comprehensive federal health-care reform statute created a Federal Health Policy Board, which was directed to monitor the fees charged for various medical procedures covered by insurance. The board also had the power to subpoena records to determine whether fee increases were a true reflection of cost increases. Nothing in the statute provided for caps on fee increases. Because of the continuing escalation of health-care costs while the statute was being debated, several states had passed health-care legislation on their own. One state passed legislation that prohibited most fee increases of 10% or more per year for specified health-care services covered by insurance, and created a health-care review board to regulate these costs and impose monetary penalties on health-care providers or insurers that tried to circumvent the cap. Which of the following would be the best basis for finding the state provision unconstitutional? A The federal legislation was passed after the state legislation and therefore supersedes it. B The Federal Health Policy Board was constituted with many of the same powers as the state board but was not given the power to impose sanctions. C The state provision impairs existing contracts between health-care providers and insurers in violation of the Contract Clause. D Health-care fee caps create an undue burden on interstate commerce even in the absence of federal regulation.

(B) The fact that the federal board was similar to the state board but was not given the power to restrict fee increases and impose sanctions in an otherwise comprehensive bill suggests that such provisions in the state law violate the Supremacy Clause. A state law may fail under the Supremacy Clause even if it does not directly conflict with a federal statute or regulation if it interferes with the achievement of a federal objective or the federal regulations occupy the entire field. Where the federal laws are comprehensive or a federal agency is created to oversee the field, preemption will often be found. The fact that the health-care legislation was comprehensive but the federal board was not given regulatory or enforcement power suggests that Congress did not want specific restrictions in these areas and may have wanted free-market principles to determine fee increases at the outset. The state board's power to impose these restrictions may violate the Supremacy Clause under these circumstances. (A) is incorrect because the fact that the federal legislation was passed later does not automatically mean that the state legislation has been superseded. In areas of concurrent legislative power, a state regulation will be upheld if it does not conflict with and is not preempted by federal legislation. (C) is incorrect because the Contract Clause prevents only substantial impairments of existing contracts by state legislation, and only if the legislation does not serve an important and legitimate public interest or is not a reasonable and narrowly tailored means of promoting that interest. Here, the law has a prospective effect only, and even if existing contracts between health-care providers and insurers are affected by the legislation, the other requirements for the Contract Clause to apply are not likely to be satisfied. (D) is incorrect because states may regulate local aspects of interstate commerce in the absence of federal regulation as long as the regulation is nondiscriminatory and does not unduly burden interstate commerce, which is a case-by-case balancing test. Here, the legislation appears to be nondiscriminatory and there are insufficient facts to establish that it would constitute an undue burden; hence, (B) presents a stronger argument than (D).

At trial, questions have been raised as to whether the proposed testimony of the witness is relevant and whether it falls within the present sense impression exception to the hearsay rule. How should a preliminary determination of the admissibility of the witness's testimony be made? A A judge should determine whether the proposed testimony falls within the exception before it is heard by the jury, and in making that determination she is limited by the rules of evidence. B A judge should decide whether the testimony falls within the present sense impression exception, but in making that determination she is not limited by the rules of evidence other than privilege. C The jury, after being instructed on the rules of evidence by a judge, should determine whether the testimony falls within the scope of the present sense impression exception. D The jury should determine whether the testimony falls within the scope of the exception and the judge should then instruct the jury on the appropriate uses for that evidence.

(B) The judge determines whether the testimony falls within an exception to the hearsay rule, and is generally not limited by the rules of evidence in making that determination. The Federal Rules of Evidence distinguish between preliminary facts to be decided by the jury, which involve whether the proffered evidence is relevant, and preliminary facts decided by the judge, which involve whether the evidence is competent, i.e., not barred by an exclusionary rule. All preliminary fact questions that determine the applicability of an exception to the hearsay rule must be determined by the judge, because the competency of the evidence will depend on that preliminary fact determination. In making this preliminary fact determination, the trial court may consider any nonprivileged relevant evidence, even though it would not otherwise be admissible under the rules of evidence. [Fed. R. Evid. 104(a)] In this case, then, the judge should decide whether the testimony falls within the present sense impression exception, and she is not limited in making this determination by the rules of evidence other than privilege.

A 13-year-old boy who lived on a farm with his parents in a rural area had learned to drive the family's tractor when he was 11. A state statute permitted persons without a driver's license to operate farm vehicles on public roads for short distances. One morning the boy took the tractor onto a public road to reach one of the outlying fields a few hundred yards away. As he neared the field he was distracted by a girl riding by on a bicycle, and cut in front of a milk delivery truck that was starting to pass him. The truck swerved off the road, injuring the driver. If the driver sues the boy to recover damages for his injuries, which of the following statements is most correct regarding the standard of care to be applied? A The state statute replaces the general common law standard of care with a statutory standard. B The trier of fact should take into account the boy's experience at driving a tractor when considering the applicable standard of care. C Persons 13 years of age or older are held to the same standard as adults. D An adult standard of care will not be applied because it is common in that region for children of that age to be operating tractors.

(B) The most correct statement is that the trier of fact should take into account the boy's experience when considering the applicable standard of care. Regardless of the specific standard of care that is applied, someone with knowledge superior to that of the average person is required to use that knowledge. Hence, the trier of fact should take into account the fact that the boy had driven a tractor since he was 11 years old. (A) is incorrect. The precise standard of care in a common law negligence case may be established by proving the applicability to that case of a statute providing for criminal penalties, so that the statute's specific duty will replace the more general common law duty of due care, and a violation of the statute will establish duty and breach of duty. Here, nothing in the facts indicates that the boy violated any provisions in the statute, but he still may be liable to the truck driver for breach of a general duty of care. (C) is incorrect. The usual standard of conduct to which a child must conform is that of a child of like age, education, intelligence, and experience. While a child must conform to an adult standard of care when engaging in a potentially dangerous activity in which usually only adults engage, there is no blanket rule that children 13 years of age or older are held to the same standard of care as adults. (D) is incorrect because the fact that 13-year-olds commonly drive tractors in that region does not preclude the court from applying an adult standard of care when a tractor is driven on a public road.

A feed store owner agreed to purchase several tons of grain products at a specified price from a large supplier of cattle feeds. The supplier later failed to deliver the promised grains, and the owner was forced to cover by purchasing from local producers at a higher price. The owner contacted a large law firm in the city and obtained their agreement to represent him in connection with his possible claims against the supplier. Due to error, the applicable statute of limitations period passed without the filing of any action on the owner's behalf. The owner retained another lawyer and sued the large law firm for malpractice. The jurisdiction retains traditional contributory negligence. In addition to the firm's negligence, what else does the owner have to establish as part of his prima facie case? A He had a good faith claim against the supplier that was lost by the law firm's dilatoriness. B He would have recovered from the supplier if an action had been timely filed. C He did not contribute to the failure to timely file an action through his own negligence. D The losses resulting from breach of the sales agreement by the supplier severely harmed his financial situation.

(B) The owner will have to show that he would have recovered damages in his lawsuit. The following elements must be proved for a prima facie case of negligence: (i) the existence of a duty on the part of the defendant to conform to a specific standard of conduct for the protection of the plaintiff against unreasonable risk of injury, (ii) breach of that duty by the defendant, (iii) that the breach of duty was the actual and proximate cause of the plaintiff's injury, and (iv) damage to the plaintiff's person or property. Here, the owner can establish that the law firm breached its professional duty of care by failing to file a claim within the statute of limitations. He must also establish that this breach was an actual and proximate cause of his damages, which here would be the loss of the contract damages that he could have recovered from the breach by the supplier. (A) is incorrect because merely having a good faith claim that was lost because of the firm's negligence is not sufficient. The owner has to show by a preponderance of the evidence that he suffered damages because of the firm's negligence.

A seller entered into a written contract to sell a tract of land to a buyer. The buyer was to pay $1,500 per month for five years, at which time the seller would deliver a warranty deed. The contract was silent as to the quality of title to be conveyed. After making 12 payments, the buyer discovered that a neighbor had an easement of way over the land, which was not discussed at the time the seller and buyer entered into the contract. The neighbor had not used the easement over the previous year because she had been out of the country. On the basis of the easement, the buyer wishes to cancel the contract. Which party is more likely to prevail? A The seller, because the neighbor's easement has been extinguished. B The seller, because the buyer has no basis on which to rescind the contract. C The buyer, because the obligation to convey marketable title is implied. D The buyer, because the seller has breached the covenant against encumbrances.

(B) The seller is more likely to prevail because the buyer has no basis on which to rescind the contract. Absent a provision to the contrary, a contract for the sale of land contains an implied promise by the seller that she will deliver to the buyer a marketable title at the time of closing. This promise imposes on the seller an obligation to deliver a title that is free from reasonable doubt; i.e., free from questions that might present an unreasonable risk of litigation. Title is marketable if a reasonably prudent buyer would accept it in the exercise of ordinary prudence. An easement that reduces the value of the property (e.g., an easement of way for the benefit of a neighbor) generally renders title unmarketable. If the buyer determines, prior to closing, that the seller's title is unmarketable, he must notify the seller and allow a reasonable time to cure the defect. If the seller is unable to acquire title before closing, so that title remains unmarketable, the buyer can rescind, sue for damages caused by the breach, or obtain specific performance with an abatement of the purchase price. However, the buyer cannot rescind prior to closing on grounds that the seller's title is unmarketable. Where an installment land contract is used, the seller's obligation is to furnish marketable title when delivery is to occur, e.g., when the buyer has made his final payment. Thus, a buyer cannot withhold payments or seek other remedies on grounds that the seller's title is unmarketable prior to the date of promised delivery. Here, there is a valid easement on the property (see below), but the seller has four years in which to cure this defect. Thus, the buyer cannot yet rescind on grounds that title is unmarketable.

A shoe manufacturer entered into a contract to purchase all cowhide refined by a tannery for the next five years at a price set at 95% of the domestic market price at the time of delivery. The shoe manufacturer agreed to purchase no less than 500 pounds of cowhide a week. At the time this contract was signed, the shoe manufacturer gave written notice to the tannery that it intended to buy all cowhide produced by the tannery until further notice. For the first year, the shoe manufacturer continued to purchase all cowhide produced by the tannery. However, by the end of that year, the tannery doubled its production of cowhide. At a meeting between the tannery and the shoe manufacturer, the tannery's president noted that the shoe manufacturer was getting as much cowhide as it needed, and that the tannery intended to sell the extra cowhide it was producing on foreign markets at a higher price than the shoe manufacturer was paying. The shoe manufacturer agreed to maintain its purchases at the first year's level and signed an addendum to the original agreement reflecting this change. Is the modification of this contract made by the addendum enforceable? A Yes, as long as the shoe manufacturer's needs continue to be met by the first year's amount. B Yes, the modification is enforceable as agreed. C No, because there was no consideration for the shoe manufacturer's agreement to take only one-half of the cowhide produced. D No, because the contract did not state the amount of cowhide that the tannery would produce.

(B) Yes, the modification is enforceable in its entirety. Although there was no consideration for the shoe manufacturer to take only one-half of the bargained-for production, none is required because this is a contract for the sale of goods. All that is required is good faith. Here, the shoe manufacturer did not need twice as much cowhide, and the tannery could make more money selling the excess to other parties. Thus, the modification was made in good faith. (A) is wrong because the enforceability of the modification is not limited by the manufacturer's needs. (C) is wrong because, as discussed above, under the UCC, a good-faith modification is enforceable even without consideration. (D) is wrong because the agreement indicates the amount of cowhide that the shoe manufacturer will purchase—all cowhide refined by the the tannery. Although no specific quantity is mentioned, these contracts are sufficiently definite because the quantity is capable of being made certain by reference to objective, extrinsic facts (i.e., the seller's actual output).

A homeowner returned home from work one day to find a robber in her living room. After a brief physical altercation, the homeowner ran to a bedroom, hid in a closet, and called 911 on her cell phone. Police officers arrived in less than two minutes and were able to apprehend the robber as he tried to run out the front door. Once they made sure he was locked in the police car, one of the officers went to speak with the homeowner about what had happened. She was still crying and shaking when the officer found her, and she said, "Thank you for catching him! He punched me in the head as I was running away!" The robber was charged with robbery and assault. Traumatized, the homeowner left the country and cannot be traced, despite the efforts of the prosecutor. The prosecutor intends to call the officer to testify as to the homeowner's statement. Should the court allow the officer's testimony? A Yes, because the homeowner's statement is an excited utterance. B Yes, because the homeowner's statement is not testimonial. C No, because admitting the homeowner's statement would violate the defendant's constitutional rights. D No, because the homeowner's statement is hearsay not within any exception.

(C) The testimony should not be admitted. Under the Confrontation Clause, an accused has the right to be confronted by the witnesses against him. A hearsay statement will not be admitted—even if it falls within a hearsay exception—when: (i) the statement is offered against the accused in a criminal case; (ii) the declarant is unavailable; (iii) the statement was testimonial in nature; and (iv) the accused had no opportunity to cross-examine the declarant's "testimonial" statement prior to trial. The Supreme Court has established that if the primary purpose of police interrogation is to enable the police to help in an ongoing emergency, statements made in the course of the interrogation are nontestimonial. When the primary purpose of the interrogation is to establish or prove past events potentially relevant to a later criminal prosecution, statements are testimonial. Here, the homeowner's statement was hearsay because it was made out of court and is being offered for its truth—that the robber punched her in the head. It appears that the emergency had already resolved by the time the statement was made. Although only a few minutes had passed since the physical altercation and the homeowner was still upset, the robber no longer posed any danger because he was locked in the police car and the homeowner was aware of this ("Thank you for catching him!"). Therefore, the homeowner's statements to the officer were testimonial. Because the homeowner is unavailable to testify at trial and the robber has had no opportunity to cross-examine the statements, admitting them at trial through the testimony of the officer would violate the Confrontation Clause. (A) is incorrect. Under the excited utterance exception, a declaration made by a declarant during or soon after a startling event is admissible if it relates to the startling occurrence and was made under the stress of excitement produced by the startling event. Here, finding a robber in one's home is a startling event, and the homeowner was still under the stress of the excitement when she made the statement—she was crying and shaking. The statement likely qualifies as an excited utterance. However, even a hearsay statement that falls within an exception may be barred by the Confrontation Clause, as is the case here. (B) is incorrect. As stated above, when the primary purpose of the interrogation is to establish or prove past events potentially relevant to a later criminal prosecution, statements are testimonial. Because the homeowner's statement was made for this purpose and not to address an ongoing emergency, it was testimonial in nature and its admission would violate by the Confrontation Clause.

While the defendant was committing a robbery, he shot and killed the victim. The defendant is charged with first degree murder in a state that defines first degree murder as murders committed with premeditation or deliberation or during the commission of burglary, arson, rape, or robbery, and defines second degree murder as all other murders. The state also defines voluntary manslaughter as the unlawful killing of a human being with malice upon a sudden quarrel or heat of passion, and it defines involuntary manslaughter as the unlawful killing of a human being without malice in the commission of an unlawful act, not amounting to an enumerated felony, or in the commission of a lawful act that might produce death in an unlawful manner or without due caution and circumspection. Assuming evidence to support, what explanation for the shooting would best help the defendant in avoiding conviction for first degree murder? A In an act of resistance, the victim suddenly attacked the defendant and knocked him down, so the defendant pulled the trigger because he was afraid the victim was going to hit him again. B The defendant had the gun for many years, it was old and rusty, and he did not think it would fire. C The defendant had taken "angel dust" before the incident and does not remember getting a gun or holding up the victim. D When the defendant tried to hold up the victim, the victim said, "Get out of here, you dirty bum, or I'll kill you," and the defendant became so upset that he did not know what he was doing.

(C) Because the defendant was charged with first degree murder, the theory of the case is most likely felony murder, and (C) is the only choice that sets out a theory to avoid a felony murder conviction. If the defendant was so intoxicated that he could not form the intent to steal, then he is not guilty of robbery, and there would be no "felony" from which the felony murder rule is to arise. (A) and (B) are incorrect because even though the defendant could argue that no premeditation or deliberation was present, he would still be guilty of first degree murder under the felony murder rule because the felony was robbery. (D) is wrong because insulting someone is not adequate provocation that would mitigate a homicide to voluntary manslaughter; neither would this "threat" suffice, in all likelihood. At most, the circumstances might produce the sort of unreasonable anger that would negate premeditation and deliberation. However, because the defendant clearly caused the homicide while committing the felony of robbery, his crime remains first degree murder.

A thief sold some stolen goods to a dealer. Several weeks later, the police raided the dealer's store and arrested him. In this raid, the police seized the goods the thief sold to the dealer and a record book in which the dealer had recorded this transaction. However, at the dealer's subsequent trial for receiving stolen goods, the charges against him were dismissed when the court ruled that the search warrant had been improperly issued. The police were able to trace the stolen goods to the thief because of fingerprint identification and the information contained in the dealer's record book. At his trial, the thief made a motion to suppress the stolen goods and record book. What should the judge do? A Grant the motion, because the evidence is the fruit of the poisonous tree in that the search of the dealer's store was improper. B Grant the motion, because the trial court in the dealer's case has already ruled that the evidence was improper. C Deny the motion, because the thief has no standing to object to the search. D Deny the motion, because the thief's fingerprints on the stolen goods were what led to his identification.

(C) The court should deny the motion because the thief had no standing to object to the search. A person challenging the admissibility of seized evidence must have standing to do so. As a general rule, standing requires a person to have a reasonable expectation of privacy in the place being searched or the item being seized. One may not challenge a search or seizure by claiming that another person's constitutional rights have been violated. Here, the thief had no ownership interest in the dealer's store. He had no reasonable expectation of privacy with respect to it; i.e., he was not present when the search was made, and he had no ownership interest in the stolen goods. Thus, he lacks the standing to object to their illegal seizure.

Acting with probable cause, the police arrested a man in connection with the armed robbery of a liquor store. After being given Miranda warnings, the man confessed to the robbery but denied his involvement with several other recent armed robberies of businesses in the area. He was formally charged with the one robbery and put into a cell with a paid informant working undercover for the police. The informant had been instructed to find out what he could about the other robberies but not to ask any questions. The informant began talking about a convenience store robbery in which a bystander was shot and seriously injured by the robber, and he deliberately misstated how it happened. The man, unaware that his cellmate was an informant, interrupted to correct him, bragging that he knew what really happened because he was there, and proceeded to make incriminating statements about the robbery. The man was subsequently charged with armed robbery and attempted murder in the convenience store robbery. At a motion-to-suppress hearing on that charge, if the man's attorney moves to exclude the statements made to the informant, should the motion be granted? A Yes, because the informant deliberately elicited incriminating statements in violation of the man's Sixth Amendment right to counsel. B Yes, because the informant's conduct constituted custodial interrogation in violation of the man's Fifth Amendment privilege against self-incrimination. C No, because the man had not yet been charged with the robbery of the convenience store when he made the statements to the informant. D No, because the informant's conduct did not constitute interrogation.

(C) The man's motion should be denied because neither his Fifth nor Sixth Amendment rights were violated by the informant's conduct. The Sixth Amendment right to counsel applies to all critical stages of a criminal prosecution after formal proceedings have begun, but does not apply in precharge custodial interrogations. Because this right is "offense specific," the fact that the right to counsel has attached for one charge does not bar questioning without counsel for an unrelated charge. Because the man has not been charged with the convenience store robbery, his Sixth Amendment right to counsel has not been violated. The Fifth Amendment privilege against self-incrimination requires Miranda warnings and a valid waiver before any statement made by the accused during custodial interrogation can be admitted. However, this requirement does not apply where interrogation is by an informant who the defendant does not know is working for the police, because the coercive atmosphere of police-dominated interrogation is not present. [Illinois v. Perkins (1990)] Because the man was not aware of the informant's status, the informant's conduct did not constitute a police interrogation. (A) is wrong despite the fact that the informant's conduct may have been deliberately designed to elicit incriminating remarks. As discussed above, the man's right to counsel did not attach for purposes of the convenience store robbery. (B) is incorrect because, as discussed above, the Miranda warnings need not be given before questioning by a cellmate working covertly for the police. (D) is incorrect because interrogation refers not only to express questioning, but also to any words or actions on the part of the police that the police should know are reasonably likely to elicit an incriminating response from the suspect. Here, the informant, working for the police, made statements about the convenience store robbery that were intended to, and reasonably likely to, prompt a response from his cellmate. Hence, it is not the absence of "interrogation" that avoids the Miranda problem, but the fact that the man did not know that his cellmate was working for the police.

Two neighbors owned summer homes adjacent to each other on the lake. After a week-long stay by the son of one of the property owners, the neighbor called the owner and said that his boat dock had been badly damaged and was told by another resident that the owner's son and some friends had gotten drunk and accidentally crashed their boat into his dock. The owner was surprised at the accusation because he was sure that if his son had caused the damages, he would have told him. However, he did not want to get into a dispute with his neighbor, so he told his neighbor that he would have the dock repaired and pay for the repairs if the neighbor agreed not to bring a claim against his son for the damage to the dock. The neighbor agreed, and the owner hired a local carpenter to do the work. Later, however, the owner discovered that his son did not damage the dock because the damages occurred after his son had returned to college. Is the owner obligated to pay for the repairs? A No, because the owner never really believed that his son caused the damage. B No, because his son in fact did not cause the damage. C No, because the neighbor was wrong when he accused his son of causing the damage and it would be unfair to enforce an agreement when there was a mutual mistake of fact. D Yes.

(D) Modern courts would hold that a promise to forbear suit on a claim that the promisor honestly and reasonably believes to be valid is good consideration to support an agreement, even if the claim ultimately turns out not to be valid. Hence, (A) and (B) are wrong. (C) is wrong because mutual mistake is not a defense when the adversely affected party bore the risk that the parties' assumption was mistaken. To be a defense, it must be a true mistake, not merely an uncertainty. Here, the owner always had the right to investigate the truth of the facts before he agreed to pay for the dock.

employees with members of the press unless prior permission had been obtained from a supervisor. Executive Department employees were subject to dismissal for violation of the order. An employee of the United States Department of Agriculture spoke to a reporter and told the reporter that the USDA sanctioned the approval of a dangerous pesticide because of payoffs made by lobbyists to a high-ranking USDA official. The newspaper printed the story and quoted the employee by name. After a hearing in line with civil service regulations, the employee was found to have violated the executive order barring unauthorized conversations with news media reporters, and the employee was fired. The employee brought suit in federal court for reinstatement, back pay, and other benefits and also prayed that the court should strike down the executive order as unconstitutional. If the employee's case reaches the United States Supreme Court, how should the Court rule? A The executive order is constitutional, because the President has plenary power to control Executive Department employees. B The executive order is constitutional, because government employment is a privilege and not a right. C The executive order is unconstitutional, because Congress, rather than the President, has authority to set the terms of federal employment. D The executive order is unconstitutional, because the President cannot broadly limit all executive employees' freedom of speech and association.

(D) The President's order is unconstitutional because it is too broad a limit on the freedom of speech and association of government employees. No government employees were allowed to have even a conversation with the press without the permission of a supervisor. Thus, it is unconstitutional. (A) is incorrect because the President's plenary power to control executive employees is limited by the Constitution. For example, the President cannot violate the First Amendment (or other constitutional guarantees) in regulating federal employees. (B) is incorrect because the Court no longer draws a distinction between rights and privileges. Furthermore, government employees would retain their First Amendment rights even if government employment were termed a "privilege." (C) is incorrect because the President has authority to enact some regulations for federal employees. The problem with this regulation is that it unduly restricts freedom of speech.

A husband and wife were traveling in a car with the wife driving when they were in an accident with a truck. The accident occurred in a jurisdiction that followed the traditional rule as to joint and several liability among tortfeasors. The husband sued the truck driver in federal district court. The truck driver, contending that the wife was an indispensable party, filed a motion to dismiss the action because the husband did not join his wife as a party to the action. How should the court rule on the truck driver's motion? A Grant the motion, because the wife is an indispensable party. B Deny the motion and order service of process on the wife to make her a party, provided she is subject to the court's personal jurisdiction. C Deny the motion, because the wife and the truck driver cannot be joined as co-defendants in the action. D Deny the motion, because the wife is not "needed for just adjudication."

(D) The court should deny the truck driver's motion because the wife is not "needed for just adjudication." When deciding whether an absentee party is indispensable, the court considers a number of factors. However, the Supreme Court has held that a joint tortfeasor subject to joint and several liability is not a person needed for just adjudication. [Temple v. Synthes Corp., 498 U.S. 5 (1990)] Because the wife here is a joint tortfeasor subject to joint and several liability, she is not an indispensable party in this case. Therefore, (D) is correct, and (A) and (B) are incorrect. (C) is incorrect because the wife could be joined as a co-defendant if the husband chose to do so; a spouse's immunity from being sued by the other spouse has been abolished in almost all jurisdictions.

A homeowner hired a contractor to make some improvements on his house. They entered into a written contract providing that the contractor would do the improvements for $5,000. Shortly after the contract was signed, the contractor told the homeowner to give the money to his (the contractor's) daughter when the job was finished, adding, "She is getting married soon and I want her to have a nice wedding present from me." The daughter was aware that her father made this statement to the homeowner. She married, but soon thereafter the contractor told the homeowner to pay him the $5,000, and not the daughter, because his son-in-law had a gambling problem and would probably use the money to bet at the racetrack. What is the best argument in favor of the daughter's being able to enforce a contract for $5,000 in her favor? A Statute of Frauds. B Parol evidence rule. C The daughter was an intended third-party beneficiary. D The daughter married in reliance on the promise.

(D) The daughter's best argument to enforce the contract in her favor is that she married in reliance on the contract (detrimental reliance), although she will probably be unsuccessful. Here, the daughter was a gratuitous assignee (because she gave no consideration) and her rights under the contract were revoked. Thus, her strongest argument will be one that nullifies the revocation. Under the doctrine of detrimental reliance, a promise will be enforced to the extent necessary to prevent injustice if it was made with a reasonable expectation that it would induce reliance, and such reliance was in fact induced. The problem with this argument here is that it is not clear that the daughter relied on the promise to give her $5,000, because she already had planned to get married. However, none of the other choices is a possible argument, so (D) is her best choice. (C) does not help her because she was not an intended third-party beneficiary. If a contract between two parties contemplates performance to a third party, that third party may have rights to enforce the contract. To do so, the third party must be an intended beneficiary at the time the contract was made (e.g., designated in the contract). An assignment, on the other hand, is a contract that does not contemplate performance to a third party when the contract is made. Rather, later one of the parties transfers his rights to another. Here, the contractor and the homeowner signed their contract and later the contractor assigned his rights to his daughter. Thus, the daughter was not an intended third-party beneficiary who could enforce the agreement, but merely an assignee who gave no consideration for the assignment. As such, the contractor was free to revoke the assignment, and his daughter cannot recover the $5,000.

The defendant was arrested, given Miranda warnings, and charged with burglary. At the police station, he telephoned his mother and asked her to come to the station to post bail. Instead, his mother immediately called the family attorney. In the meantime, the police had begun questioning the defendant. Although he never told the police to stop the questioning, his answers were at first vague or clearly unresponsive. During the course of the questioning, the family attorney phoned the station and told the police that she had been hired to represent the defendant and would be there in half an hour. The police did not inform the defendant of the attorney's call. Ten minutes later, the defendant admitted to committing the burglary, and signed a statement to that effect prepared by the police. The attorney arrived a few minutes later and advised the defendant to remain silent, but he told her that he had already signed a confession. How should the court rule on the attorney's pretrial motion to exclude the confession as evidence at trial? A Grant the motion, because the police had a duty to inform the defendant that an attorney was coming to represent him. B Grant the motion, because the defendant has been deprived of his Sixth Amendment right to counsel. C Deny the motion, because the defendant's statement admitting the crime was voluntary. D Deny the motion, because the defendant waived his Miranda rights.

(D) The defendant's confession should be admitted because he waived his Fifth Amendment privilege against compelled self-incrimination after receiving Miranda warnings. Miranda v. Arizona requires that a person in custody be informed of his right to remain silent and his right to the presence of an attorney during questioning. A suspect may subsequently waive his rights by making a confession, as long as the waiver was knowing and voluntary. In this case, the defendant received proper Miranda warnings, and there is no indication that he did not understand what his rights were. Although his answers during questioning were initially unresponsive, he never asked for an attorney or indicated that he wished to remain silent, and he voluntarily confessed after a relatively short period of interrogation. Hence, he validly waived his Miranda rights.(C) is incorrect even though it is true that the defendant made a voluntary statement. Due process requires that for confessions to be admissible, they must be "voluntary," based on the totality of the circumstances, and here all of the circumstances indicate that the defendant's confession was voluntary. However, even a voluntary confession will be inadmissible if it was obtained in violation of Miranda rights. (D) is therefore a better choice than (C).

It was common practice in a particular state for a security interest in land to be structured as a deed absolute, which gave a lender absolute title to the borrower's property as security for the loan. The lender would reconvey only on complete payment of the loan by the debtor party, and could dispose of the land immediately without a foreclosure sale on default. A new governor of the state whose campaign platform was built around abolishing the deed absolute mortgage encouraged the legislature to enact a bill that immediately outlawed use of the deed absolute, declaring that all such deeds would be considered mere liens against the secured property. The law applied not only to loans made in the future, but also to the thousands of such loans in existence at the time the legislation was passed. As soon as the governor signed the legislation, lending institutions and individuals who had loaned money secured through deeds absolute challenged the constitutionality of the new law. What is the strongest argument that the challengers can make?

As applied to the loans outstanding at the time the bill was enacted, the law impairs the contract rights of the lenders and such rights are guaranteed by the Contracts Clause of the federal Constitution. (R. Contracts clause only applies to state laws that retroactively invalidate already existing contracts; cannot do so unless the government act serves an important government interest and state law is narrowly tailored to that government interest.)

A developer contracted with a general contractor to build an office building, and completion of the building was two years late. The developer filed a breach of contract action in federal district court against the general contractor, seeking damaged caused by the delay. The general contractor filed a third-party claim against a major subcontractor, claiming that the subcontractor caused any delay and should be liable to the general contractor for anything the general contractor has to pay the developer. The subcontractor believes that the developer interfered with the subcontract and that the developer's interference caused not only the delay but also substantial cost overruns for the subcontractor. May the subcontractor assert a claim in the pending action against the developer seeking payment for the cost overruns?

Yes, because the subcontractor's claim against the developer arises form the same transaction or occurrence as the developer's original claim, but the subcontractor may assert the claim in an independent action if it prefers.


Related study sets

ADV 3008 Exam 2 Learnsmarts (CH 7-CH 9)

View Set

Chapter 42: Management of Patients With Musculoskeletal Trauma

View Set

Biology - Chapter 24 (The Body's Defenses)

View Set

Psych 211 Exam 4 Applied Questions

View Set

Philos: Final Study Guide: Part 5 Sartre

View Set